eureka 2009

135
CONTEÚDO XIV OLIMPÍADA DE MAIO 2 XLVIII OLIMPÍADA INTERNACIONAL DE MATEMÁTICA Enunciados e Resultado Brasileiro 5 XLIX OLIMPÍADA INTERNACIONAL DE MATEMÁTICA Enunciados e Resultado Brasileiro 7 XXII OLIMPÍADA IBEROAMERICANA DE MATEMÁTICA Enunciados e Resultado Brasileiro 9 XXIII OLIMPÍADA IBEROAMERICANA DE MATEMÁTICA Enunciados e Resultado Brasileiro 12 ARTIGOS 0,999... OU “COMO COLOCAR UM BLOCO QUADRADO EM UM BURACO REDONDO” Pablo Emanuel 14 ALGORITMO DE GOSPER E APLICAÇÕES Humberto Silva Naves 21 DOMINGO REGADO A REPUNITS Valberto Rômulo Feitosa Pereira 27 HOMOTETIAS, COMPOSIÇÃO DE HOMOTETIAS EO PROBLEMA 6 DA IMO 2008 Carlos Yuzo Shine 32 COMO É QUE FAZ? 44 OLIMPÍADAS AO REDOR DO MUNDO 48 SOLUÇÕES DE PROBLEMAS PROPOSTOS 52 PROBLEMAS PROPOSTOS 62 AGENDA OLÍMPICA 63 COORDENADORES REGIONAIS 64

Upload: demi-de

Post on 28-Mar-2016

350 views

Category:

Documents


0 download

DESCRIPTION

ARTIGOS 0,999... OU “COMO COLOCAR UM BLOCO QUADRADO EM UM BURACO REDONDO” Pablo Emanuel HOMOTETIAS, COMPOSIÇÃO DE HOMOTETIAS EO PROBLEMA 6 DA IMO 2008 Carlos Yuzo Shine ALGORITMO DE GOSPER E APLICAÇÕES Humberto Silva Naves XXII OLIMPÍADA IBEROAMERICANA DE MATEMÁTICA Enunciados e Resultado Brasileiro XXIII OLIMPÍADA IBEROAMERICANA DE MATEMÁTICA Enunciados e Resultado Brasileiro XLIX OLIMPÍADA INTERNACIONAL DE MATEMÁTICA Enunciados e Resultado Brasileiro 12 14 27 32 21 5 7 9

TRANSCRIPT

Page 1: Eureka 2009

CONTEÚDO XIV OLIMPÍADA DE MAIO

2

XLVIII OLIMPÍADA INTERNACIONAL DE MATEMÁTICA Enunciados e Resultado Brasileiro

5

XLIX OLIMPÍADA INTERNACIONAL DE MATEMÁTICA Enunciados e Resultado Brasileiro

7

XXII OLIMPÍADA IBEROAMERICANA DE MATEMÁTICA Enunciados e Resultado Brasileiro

9

XXIII OLIMPÍADA IBEROAMERICANA DE MATEMÁTICA Enunciados e Resultado Brasileiro

12

ARTIGOS

0,999... OU “COMO COLOCAR UM BLOCO QUADRADO EM UM BURACO REDONDO” Pablo Emanuel

14

ALGORITMO DE GOSPER E APLICAÇÕES Humberto Silva Naves

21

DOMINGO REGADO A REPUNITS Valberto Rômulo Feitosa Pereira

27

HOMOTETIAS, COMPOSIÇÃO DE HOMOTETIAS EO PROBLEMA 6 DA IMO 2008 Carlos Yuzo Shine

32

COMO É QUE FAZ?

44

OLIMPÍADAS AO REDOR DO MUNDO

48

SOLUÇÕES DE PROBLEMAS PROPOSTOS

52

PROBLEMAS PROPOSTOS

62

AGENDA OLÍMPICA

63

COORDENADORES REGIONAIS 64

Page 2: Eureka 2009

Sociedade Brasileira de Matemática

EUREKA! N°29, 2009

2

XIV OLIMPÍADA DE MAIO PRIMEIRO NÍVEL

PROBLEMA 1 Quantos números distintos de 6 algarismos e múltiplos de 45 podem ser escritos colocando um dígito à esquerda e outro à direita de 2008? PROBLEMA 2 No colégio Olímpico as provas são avaliadas com números inteiros, a menor nota possível é 0, e a maior é 10. Na aula de Matemática o professor aplica as provas. Este ano a turma tem 15 alunos. Quando um dos alunos tira na primeira prova uma nota menor que 3 e na segunda prova uma nota maior que 7, o aluno é chamado de aluno superado. O professor, ao terminar de corrigir as provas, fez uma média com as 30 notas e obteve 8. Qual é a maior quantidade de alunos superados que pode ter tido a turma? PROBLEMA 3 Num quadro negro estão escritos os números inteiros de 1 a 2008 inclusive. Apagam-se dois números e escreve-se a diferença entre eles. Por exemplo, se apagamos o número 5 e 241, escrevemos 236. Assim continuamos, apagando os números e escrevendo a diferença, até que fica somente um número. Determine se o número que ficou por último pode ser 2008. E 2007? Em cada caso, se a resposta é afirmativa indique uma seqüência com esse número final, e se é negativa, explique o porquê. PROBLEMA 4 Sobre o lado AB de um quadrado ABCD é desenhado exteriormente o triângulo retângulo ABF, de hipotenusa AB. Sabe-se que AF = 6, e que BF = 8. Chamamos de E o centro do quadrado. Calcule o comprimento de EF. PROBLEMA 5 Num tabuleiro de 16 × 16 colocamos 25 moedas, como na figura abaixo. É permitido selecionar 8 linhas e 8 colunas e retirar do tabuleiro todas as moedas que se encontram nessas linhas e colunas. Determine se é possível retirar todas as moedas do tabuleiro. Se a resposta é afirmativa, indique as 8 linhas e as 8 colunas selecionadas, e se é negativa, explique o porquê.

Page 3: Eureka 2009

Sociedade Brasileira de Matemática

EUREKA! N°29, 2009

3

SEGUNDO NÍVEL PROBLEMA 1 Num quadro negro está escrita a seguinte expressão:

2 3 4 5 6 7 8 9 101 2 2 2 2 2 2 2 2 2 2 .− − − − − − − − − − Juan distribui parêntesis de distintas maneiras e efetua o cálculo que fica. Por exemplo:

2 3 4 5 6 7 8 9 101 2 (2 2 ) 2 (2 2 2 ) 2 (2 2 ) 403− − − − − − − − − − = ou 2 3 4 5 6 7 8 9 101 (2 2 ( 2 2 ) (2 2 2 )) (2 2 ) 2 933.− − − − − − − − − − = −

Quantos resultados diferentes pode obter Juan? PROBLEMA 2 No retângulo ABCD de lados AB, BC, CD e DA, seja P um ponto do lado AD tal que µ 90BPC = ° . A perpendicular a BP traçada por A corta BP em M e a perpendicular a CP traçada por D corta CP em N. Demonstre que o centro do retângulo está no segmento MN. PROBLEMA 3 Nos números 1010...101 estão alternados uns e zeros; se há n uns, há n – 1 zeros ( 2).n ≥ Determine os valores de n para os quais o número 1010...101, que tem n uns, é primo. PROBLEMA 4 No plano há 16 retas tais que não há duas paralelas nem três concorrentes. Sebastián tem que pintar os 120 pontos que são interseção de duas retas de modo que em cada reta todos os pontos pintados sejam de cor diferente. Determine o número mínimo de cores que Sebastián precisa para concluir a tarefa. E se as retas são 15 (neste caso, os pontos são 105)?

Page 4: Eureka 2009

Sociedade Brasileira de Matemática

EUREKA! N°29, 2009

4

PROBLEMA 5 Matias cobriu um tabuleiro quadrado de 7 × 7, dividido em casas de 1 × 1, com peças dos três tipos a seguir

Tipo 1 Tipo 2 Tipo 3

sem buracos nem superposições, e sem sair do tabuleiro. Cada peça do tipo 1 cobre exatamente 3 casas e cada peça do tipo 2 ou do tipo 3 cobre exatamente 4 casas. Determine a quantidade de peças do tipo 1 que Matias pode ter utilizado. (As peças podem girar e ser viradas). RESULTADO BRASILEIRO 2008: Nível 1 (até 13 anos) Nome Cidade - Estado Pontos Prêmio

Rafael Kazuhiro Miyazaki São Paulo – SP 45 Medalha de Ouro

Débora Ornellas Salvador – BA 36 Medalha de Prata

Nicolas Seoane Miquelin Mauá – SP 34 Medalha de Prata

Guilherme Renato Martins Unze São Paulo – SP 33 Medalha de Bronze

Ana Beatrice Bonganha Zanon Santo André – SP 26 Medalha de Bronze

Paula Dias Garcia Brasília – DF 24 Medalha de Bronze

Lara Timbó Araújo Fortaleza – CE 24 Medalha de Bronze

Francisco Markan Nobre de Souza Filho Fortaleza – CE 24 Menção Honrosa

Henrique Gasparini Fiúza do Nascimento Brasília – DF 23 Menção Honrosa

Henrique Vieira G. Vaz São Paulo – SP 23 Menção Honrosa

2008: Nível 2 (até 15 anos)

Nome Cidade - Estado Pontos Prêmio

João Lucas Camelo Sá Fortaleza – CE 37 Medalha de Ouro

Guilherme da Rocha Dahrug Santo André – SP 30 Medalha de Prata

Matheus Barros de Paula Taubaé – SP 29 Medalha de Prata

Rafael Ferreira Antonioli S.B. do Campo – SP 23 Medalha de Bronze

Nara Gabriela de Mesquita Peixoto Fortaleza – CE 22 Medalha de Bronze

Rodrigo Nagamine Santo André – SP 22 Medalha de Bronze

Henrique Lopes de Mello Rio de Janeiro – RJ 21 Medalha de Bronze

Victorio Takahashi Chu São Paulo – SP 21 Menção Honrosa

Jonas Rocha de Lima Amaro Fortaleza – CE 17 Menção Honrosa

Deborah Barbosa Alves São Paulo – SP 16 Menção Honrosa

Page 5: Eureka 2009

Sociedade Brasileira de Matemática

EUREKA! N°29, 2009

5

XLVIII OLIMPÍADA INTERNACIONAL DE MATEMÁTICA Enunciados e Resultado Brasileiro

A XLVIII Olimpíada Internacional de Matemática foi realizada na cidade

de Hanói, Vietnã no período de 19 a 31 de julho de 2007. A equipe brasileira foi liderada pelos professores Carlos Gustavo Moreira do Rio de Janeiro – RJ e Onofre Campos da Silva Farias de Fortaleza – CE. RESULTADOS DA EQUIPE BRASILEIRA BRA1 Régis Prado Barbosa Medalha de Prata BRA2 Henrique Pondé de Oliveira Pinto Medalha de Prata BRA3 Ramón Moreira Nunes Medalha de Bronze BRA4 Rafael Sampaio de Rezende Medalha de Bronze BRA5 Rafael Tupynambá Dutra Medalha de Bronze BRA6 Guilherme Phillipe Figueiredo Menção Honrosa PRIMEIRO DIA PROBLEMA 1: Sejam 1 2, ,..., na a a números reais. Para cada (1 )i i n≤ ≤ definimos

max :1 min : i j jd a j i a i j n= ≤ ≤ − ≤ ≤

e max :1 .id d i n= ≤ ≤

(a) Prove que para quaisquer números reais 1 2 ... ,nx x x≤ ≤ ≤

max :1 .2i i

dx a i n− ≤ ≤ ≥ (*)

(b) Prove que existem números reais 1 2 ... nx x x≤ ≤ ≤ para os quais vale a igualdade em (*). PROBLEMA 2: São dados cinco pontos A, B, C D e E tais que ABCD é um paralelogramo e BCED é um quadrilátero cíclico (e convexo). Seja l uma recta que passa por A.

Page 6: Eureka 2009

Sociedade Brasileira de Matemática

EUREKA! N°29, 2009

6

Suponhamos que l intersecta o segmento DC num ponto interior F e a recta BC em G. Suponhamos também que .EF EG EC= = Prove que l é a bissectriz do ângulo DAB. PROBLEMA 3: Numa competição de Matemática alguns participantes são amigos. A amizade é sempre recíproca. Dizemos que um grupo de participantes é um clique se dois quaisquer deles são amigos (em particular, qualquer grupo com menos de dois participantes é um clique). O tamanho de um clique é o número de seus elementos. Sabe-se que nesta competição o tamanho máximo dos cliques é par. Prove que os participantes podem ser distribuídos em duas salas, de modo que o tamanho máximo dos cliques contidos numa sala é igual ao tamanho máximo dos cliques contidos na outra sala. SEGUNDO DIA PROBLEMA 4: No triângulo ABC a bissectriz do ângulo BCA intersecta a circunferência circunscrita em ( ),R R C≠ a mediatriz de BC em P e a mediatriz de AC em Q. O ponto médio de BC é K e o ponto médio de AC é L. Mostre que os triângulos RPK e RQL têm áreas iguais. PROBLEMA 5: Sejam a e b inteiros positivos tais que 4ab – 1 divide 2 2(4 1) .a − Prove que a = b. PROBLEMA 6: Seja n um número inteiro positivo. Considere o conjunto

( , , ) : , , 0,1,..., , 0S x y z x y z n x y z= ∈ + + >

de 3( 1) 1n + − pontos no espaço tridimencional. Determine o menor número possível de planos cuja união contém todos os pontos de S mas não contém (0, 0, 0).

Page 7: Eureka 2009

Sociedade Brasileira de Matemática

EUREKA! N°29, 2009

7

XLIX OLIMPÍADA INTERNACIONAL DE MATEMÁTICA Enunciados e Resultado Brasileiro

A XLIX Olimpíada Internacional de Matemática foi realizada na cidade de

Madri, Espanha no período de 10 a 22 de julho de 2008. A equipe brasileira foi liderada pelos professores Luciano Monteiro de Castro do Rio de Janeiro – RJ e Carlos Yuzo Shine de São Paulo – SP. RESULTADOS DA EQUIPE BRASILEIRA BRA1 Davi Lopes Alves de Medeiros Medalha de Prata BRA2 Henrique Pondé de Oliveira Pinto Medalha de Prata BRA3 Marcelo Matheus Gauy Medalha de Bronze BRA4 Rafael Tupynambá Dutra Medalha de Prata BRA5 Régis Prado Barbosa Medalha de Prata BRA6 Renan Henrique Finder Medalha de Prata PRIMEIRO DIA PROBLEMA 1: Seja ABC um triângulo acutângulo e seja H o seu ortocentro. A circunferência de centro no ponto médio de BC e que passa por H intersecta a reta BC nos pontos

1A e 2 .A Analogamente, a circunferência de centro no ponto médio de CA e que passa por H intersecta a reta CA nos pontos 1B e 2 ,B e a circunferência de centro no ponto médio de AB e que passa por H intersecta a reta AB nos pontos 1C e 2C . Mostre que 1 2 1 2 1 2, , , , ,A A B B C C estão sobre uma mesma circunferência. PROBLEMA 2: (a) Prove que

2 2 2

2 2 2 1( 1) ( 1) ( 1)

x y zx y z

+ + ≥− − −

(*)

para todos os números reais x, y, z, diferentes de 1, com xyz = 1. (b) Prove que existe uma infinidade de ternos de números racionais x, y, z, diferentes de 1, com xyz = 1, para os quais ocorre a igualdade em (*).

Page 8: Eureka 2009

Sociedade Brasileira de Matemática

EUREKA! N°29, 2009

8

PROBLEMA 3: Prove que existe um número infinito de inteiros positivos n tais que 2 1n + tem um divisor primo maior que 2 2 .n n+ PROBLEMA 4: Determine todas as funções ] [ ] [0, 0,f ∞ → ∞ (ou seja, f é uma função dos reais positivos para os reais positivos) tais que

( ) ( )2 2 2 2

2 2 2 2

( ) ( )( ) ( )

f w f x w xf y f z y z

+ +=

+ +

para todos os números reais positivos w, x, y com wx = yz. PROBLEMA 5: Sejam n e k números inteiros positivos tais que k n≥ e k n− é um número par. São dadas 2n lâmpadas numeradas de 1 a 2n, cada uma das quais pode estar acesa ou apagada. Inicialmente todas as lâmpadas estão apagadas. Uma operação consiste em alterar o estado de exatamente uma das lâmpadas (de acessa para apagada ou de apagada para acesa). Consideremos seqüências de operações. Seja n o número de seqüências com k operações após as quais as lâmpadas de 1 a n estão todas acessas e as lâmpadas de n + 1 a 2n estão todas apagadas. Seja M o número de seqüências com k operações após as quais as lâmpadas de 1 a n estão todas acesas e as lâmpadas de n + 1 e 2n estão todas apagadas, e durante as quais todas as lâmpadas de n + 1 a 2n permanecem sempre apagadas.

Determine a razão .NM

PROBLEMA 6: Seja ABCD um quadrilátero convexo cujos lados BA e BC tam comprimentos diferentes. Sejam 1w e 2w as circunferências inscritas nos triângulos ABC e ADC, respectivamente. Suponhamos que existe um circunferência w tangente à reta BA de forma que A está entre B e o ponto de tangência, tangente à reta BC de forma que C está entre B e o ponto de tangência, e que também seja tangente às retas AD e CD. Prove que as tangentes comuns exteriores a 1w e 2w se intersectam sobre w.

Page 9: Eureka 2009

Sociedade Brasileira de Matemática

EUREKA! N°29, 2009

9

XXII OLIMPÍADA IBEROAMERICANA DE MATEMÁTICA Enunciados e Resultado Brasileiro

A XXII Olimpíada Iberoamericana de Matemática foi realizada na cidade

de Coimbra, Portugal no período de 6 a 16 de setembro de 2007. A equipe brasileira foi liderada pelos professores Eduardo Wagner e Edmilson Motta da cidade do Rio de Janeiro – RJ e São Paulo – SP respectivamente. RESULTADOS DA EQUIPE BRASILEIRA BRA1 Guilherme Rodrigues Nogueira de Souza Medalha de Ouro BRA2 Henrique Pondé de Oliveira Pinto Medalha de Prata BRA3 Ramon Moreira Nunes Medalha de Ouro BRA4 Régis Prado Barbosa Medalha de Ouro PRIMEIRO DIA PROBLEMA 1: Dado um inteiro positivo m, define-se a sucessão na da seguinte maneira:

1 ,2m

a = 1 ,n n na a a+ = si 1.n ≥

Determinar todos os valores de m para os quais 2007a é o primeiro inteiro que aparece na sucessão. Nota: Para um número real x se define x como o menor inteiro que é maior ou igual a x. Por exemplo, 4, 2007 2007.π = =

PROBLEMA 2: Sejam ABC um triângulo com incentro I e Γ uma circunferência de centro I, de raio maior que o da circunferência inscrita e que não passa por nenhum dos vértices. Sejam 1X o ponto de intersecção de Γ com a reta AB mais perto de

2,B X e 3X os pontos de intersecção de Γ com a recta BC sendo 2X o mais perto de B e 4X o ponto de intersecção de Γ com a recta CA mais perto de C. Seja K o ponto de intersecção das rectas 1 2X X e 3 4.X X Demonstre que AK corta o segmento 2 3X X no seu ponto médio.

Page 10: Eureka 2009

Sociedade Brasileira de Matemática

EUREKA! N°29, 2009

10

PROBLEMA 3: Duas equipes, A e B disputam o território limitado por uma circunferência. A tem n bandeiras azuis e B tem n bandeiras brancas ( 2,n ≥ fixo). Jogam alternadamente e A começa o jogo. Cada equipe, na sua vez, coloca uma das suas bandeiras num ponto da circunferência que não se tenha usado numa jogada anterior. Cada bandeira, uma vez colocada, não se pode mudar de lugar. Uma vez colocadas as 2n bandeiras reparte-se o território entre as duas equipes. Um ponto do território é da equipe A se a bandeira mais próxima dele é azul, e é da equipe B se a bandeira mais próxima dele é branca. Se a bandeira azul mais próxima de um ponto está à mesma distância que a bandeiras branca mais próxima deste ponto, então o ponto é neutro (não é de A nem de B). Uma equipe ganha o jogo se seus pontos cobrem uma área maior que a área coberta pelos pontos da outra equipe. Há empate se ambos cobrem áreas iguais. Demonstre que, para todo n a equipe B tem estratégia para ganhar o jogo. SEGUNDO DIA PROBLEMA 4: Em um tabuleiro quadriculado de tamanho 19 × 19, uma ficha chamada dragão dá saltos da seguinte maneira: desloca-se 4 casas numa direcção paralela a um dos lados do tabuleiro e 1 casa em direção perpendicular à anterior. X X

X X

D

A partir de D, o dragão pode saltar para uma das quatro posições X. Sabe-se que, com este tipo de saltos, o dragão pode mover-se de qualquer casa a qualquer outra. A distancia dragoniana entre duas casas é o menor número de saltos que o dragão deve dar para mover-se de uma casa a outra. Seja C uma casa situada num canto do tabuleiro e seja V a casa vizinha a C que a toca num único ponto.

Page 11: Eureka 2009

Sociedade Brasileira de Matemática

EUREKA! N°29, 2009

11

Demonstre que existe alguma casa X do tabuleiro tal que a distância dragoniaana de C a X é maior que a distãncia dragoniana de C a V. PROBLEMA 5: Um número natural n é atrevido se o conjunto dos seus divisores, incluindo 1 e n, pode ser dividido em três subconjuntos tais que a soma dos elementos de cada subconjunto é a mesma nos três. Qual é a menor quantidade de divisores que pode ter um número atrevido?

PROBLEMA 6: Seja F a família de todos os hexágonos convexos H que satisfazem as seguintes condições: a) os lados opostos de H são paralelos; b) quaisquer três vértices de H podem ser cobertos por uma faixa de largura 1. Determine o menor número real l tal que cada um dos hexágonos da família F pode ser coberto com uma faixa de largura l . Nota: Uma faixa de largura l é a região do plano compreendida entre duas rectas paralelas que estão à distancia l (incluídas ambas as rectas paralelas).

Page 12: Eureka 2009

Sociedade Brasileira de Matemática

EUREKA! N°29, 2009

12

XXIII OLIMPÍADA IBEROAMERICANA DE MATEMÁTICA Enunciados e Resultado Brasileiro

A XXIII Olimpíada Iberoamericana de Matemática foi realizada na cidade

de Salvador, Bahia no período de 20 a 28 de setembro de 2008. A equipe brasileira foi liderada pelos professores Eduardo Wagner e Fábio Dias Moreira, ambos da cidade de Rio de Janeiro – RJ. Com este resultado a equipe brasileira obteve também a maior pontuação total da competição ficando em primeiro lugar com 155 pontos. RESULTADOS DA EQUIPE BRASILEIRA BRA1 Henrique Ponde de Oliveira Pinto Medalha de Ouro BRA2 Renan Henrique Finder Medalha de Prata BRA3 Ramon Moreira Nunes Medalha de Ouro BRA4 Régis Prado Barbosa Medalha de Prata PRIMEIRO DIA PROBLEMA 1: Os números 1, 2, 3,..., 20082 são distribuídos num tabuleiro 2008 × 2008, de modo que em cada casa haja um número distinto. Para cada linha e cada coluna do tabuleiro calcula-se a diferença entre o maior e o menor dos seus elementos. Seja S a soma dos 4016 números obtidos. Determine o maior valor possível para S. PROBLEMA 2: Sejam ABC um triângulo escaleno e r a bissectriz externa do ângulo .ABC∠ Sejam P e Q os pés das perpendiculares à recta r que passam por A e C, respectivamente. As rectas CP e AB intersectam-se em M e as rectas AQ e BC intersectam-se em N. Demonstre que as rectas AC, MN e r têm um ponto em comum. PROBLEMA 3: Sejam m e n inteiros tais que o polinômio 3( )P x x mx n= + + tem a seguinte propriedade: se x e y são inteiros e 107 divide ( ) ( ),P x P y− então 107 divide x – y. Demosntre que 107 divide m.

Page 13: Eureka 2009

Sociedade Brasileira de Matemática

EUREKA! N°29, 2009

13

SEGUNDO DIA PROBLEMA 4: Demonstre que não existem inteiros positivos x e y tais que

2008 2008! 21 .yx + = PROBLEMA 5: Seja ABC um triângulo e X, Y, Z pontos interiores dos lados BC, AC, AB respectivamente. Sejam A´, B´, C´ os circuncentros dos triângulos AZY, BXZ, CYX, respectivamente. Demonstre que

( ) ( )´ ´ ´

4ABC

A B C ≥

e que a igualdade ocorre se, o somente se, as rectas AA´, BB´, CC´ têm um ponto em comum. Observação: Para um triângulo qualquer RST, denotamos a sua área por (RST). PROBLEMA 6: Numa partida de biribol enfrentam-se duas equipes de quatro jogadores cada uma. Organiza-se um torneio de biribol em que participam n pessoas, que formam equipes para cada partida (as equipes não são fixas). No final do torneio observou-se que cada duas pessoas disputaram exactamente uma partida em equipes rivais. Para que valores de n é possível organizar um torneio com tais características?

Page 14: Eureka 2009

Sociedade Brasileira de Matemática

EUREKA! N°29, 2009

14

0,999... OU “COMO COLOCAR UM BLOCO QUADRADO EM UM BURACO REDONDO”

Pablo Emanuel

• Nível Intermediário

Quando um jovem estudante de matemática começa a estudar os números reais, é difícil não sentir certo desconforto e estranhamento. De repente, algumas coisas que faziam sentido param de funcionar tão bem assim. Com certeza, uma das mais estranhas pode ser resumida na igualdade

0,9999..... = 1,0000... Como assim? No mundo dos números inteiros (com a exceção muito razoável de 0 = –0), qualquer número tem uma única representação decimal (os zeros à esquerda não são um problema sério, ou definimos que nunca podemos começar com 0 – o que deixa o próprio zero em uma situação meio desconfortável de ter uma representação decimal vazia – ou definimos que sempre vamos completar com infinitos zeros à esquerda), e esta se comporta bem – i.e. qualquer número da forma 8xxxxxxx é menor que qualquer número da forma 9yyyyyyy com o mesmo número de dígitos. Como pode haver um número começado por 0,9 que não seja menor que um número começado por 1,0 e, pior, que seja igual!? A resposta explica, mas não convence: se definirmos as seqüências: an = 0.999999...9 (n dígitos 9) e bn = 1.000000...0 (n dígitos 0) temos que an < 0.999... ⇐ 1.000... < bn. No entanto, como bn – an = 1/10n, que converge para 0, a diferença entre 1.000... e 0.999..., que é menor que qualquer das diferenças bn – an só pode ser 0, logo os números são iguais. OK, entendido, mas ainda tem caroço neste angu. Por que a representação decimal, que se comporta tão bem para números inteiros tem este tipo de esquisitice para números reais? A verdade é que usar a representação decimal para números reais é enfiar um bloco quadrado em um buraco redondo. Em primeiro lugar, até agora estamos usando a representação decimal (base 10) apenas porque é a representação com que estamos mais acostumados. Será que o

Page 15: Eureka 2009

Sociedade Brasileira de Matemática

EUREKA! N°29, 2009

15

problema está com o número 10? Infelizmente não. Mesmo que usemos outras bases, o problema continua: 0,1111.... = 1,000.... em base 2 0,2222... = 1,000... em base 3 e o mesmo problema acontece em qualquer base que escolhamos (e a demonstração é exatamente a mesma, mutatis mutandis, que fizemos lá em cima). Antes de entrarmos a fundo em por que a representação decimal (ou em qualquer base N) tem estes problemas, é bom ver que outras opções nós temos para representar os números reais. Em primeiro lugar, nossas notações foram feitas para trabalhar com números inteiros, que é o que nos é familiar (a ponto de Pitágoras afirmar que os números inteiros são a fundação do universo). Dos números inteiros conseguimos de forma natural derivar os números racionais (os números da forma a/b, onde a e b são inteiros). O pulo para os números reais irracionais, no entanto, é menos natural, e as únicas maneiras que temos para tentar nomear os números irracionais são através das aproximações por números racionais. A própria representação decimal é a aproximação por números racionais da forma A/10n (ou A/Nn, em base N). Duas outras formas de representar os números reais são as frações contínuas e os cortes de Dedekind. A representação por frações contínuas tem a forma N1 + 1 _________________ N2 + 1 ___________ N3 + ... O algoritmo para construir a representação por frações contínuas de um número x é bastante simples. Em primeiro lugar, N1 é a parte inteira de x (i.e. o maior número inteiro que é menor ou igual a x). Se x – N1 = 0, acabou, senão, x – N1 é maior que 0 e menor que 1, portanto y = 1/(x – N1) é um número maior que 1. Seja N2 então a parte inteira deste número, o que equivale a dizer que 1/(N2 + 1) < x – N1 ≤ 1/N2. Agora repita o processo com y no lugar de x (i.e. N3 é o número tal que 1/(N3 + 1) < y – N2 ⇐ 1/N3), e assim por diante até chegar a um número inteiro ou continuando para sempre.

Page 16: Eureka 2009

Sociedade Brasileira de Matemática

EUREKA! N°29, 2009

16

A representação por frações contínuas não tem o mesmo problema da representação decimal (cada número real tem uma e somente uma representação por frações contínuas), mas tem outros inconvenientes. O primeiro, que não é tão sério assim, é que os números Ni, que seriam equivalentes aos dígitos, podem ser arbitrariamente grandes, ao contrário dos dígitos decimais que só podem ser de 0 a 9. O segundo, muito mais sério, é que é terrivelmente difícil fazer contas com frações contínuas. Este foi o principal motivo de a representação decimal ter substituído a numeração romana na Europa no século XV – era imensamente mais simples fazer contas com a representação decimal do que com a representação romana. (E este também é o principal motivo de a representação binária ter tomado espaço da representação decimal no século XX, é ainda mais fácil fazer contas em binário). Outra representação dos números reais é através dos cortes de Dedekind. A diferença do corte de Dedekind para a representação decimal ou de frações contínuas é que, enquanto as últimas usam uma seqüência convergente de números racionais, os cortes usam conjuntos sem uma seqüência definida. Um corte de Dedekind é um conjunto de números racionais limitado, que não possui um maior elemento e fechado inferiormente. Ou seja, um conjunto de racionais A tal que:

1) existe um racional X tal que X > a para todo a pertencente a A 2) para todo a pertencente a A, existe um b pertencente a A tal que b > a. 3) Se a pertence a A, e um racional c < a, então c pertence a A.

Em outras palavras, cada um destes conjuntos é o conjunto de todos os números racionais menores que um número real determinado. Esta representação é útil teoricamente para provar algumas propriedades dos números reais, mas é praticamente impossível de ser utilizada na prática para as tarefas corriqueiras do dia-a-dia. A conclusão é que, apesar dos seus defeitos, a representação de base N é a representação mais prática que temos para os números reais. Então vamos entendê-la mais a fundo. Em primeiro lugar, a parte antes da vírgula (ou ponto decimal, dependendo de em qual lugar do mundo você vive) é apenas a parte inteira do número, com a nossa velha e bem comportada representação decimal de números inteiros. Vamos nos concentrar então na parte à direita da vírgula, os números entre 0 e 1.

Page 17: Eureka 2009

Sociedade Brasileira de Matemática

EUREKA! N°29, 2009

17

0,0 0,1 0,2 0,3 0,4 0,5 0,6 0,7 0,8 0,9

O primeiro dígito decimal diz em qual dos dez intervalos acima o número está. Para descobrir o segundo dígito, basta subdividir cada intervalo novamente em 10 pedaços, ou, equivalentemente, multiplicar o número por 10 e ver em qual dos intervalos a parte fracionária deste novo número (10x) cai, de acordo com o gráfico abaixo:

0,9

0,8

0,7

0,6

0,5

0,4

0,3

0,2

0,1

0,00,0 0,1 0,2 0,3 0,4 0,5 0,6 0,7 0,8 0,9

Os pontos onde nasce o nosso problema são justamente estes pontos marcados na nossa linha (0; 0,1; 0,2; ...; 1). Repare que o gráfico acima é descontínuo nestes pontos – por exemplo, as imagens dos pontos se aproximando de 0,2 pela esquerda tendem a 1, enquanto pela direita tendem a 0. Dito de outra forma, os números se aproximando pela esquerda de 0,2 vão ter uma seqüência longa de 9’s depois do 0,1 e pela direita uma seqüência longa de 0’s depois do 0,2. Antes de prosseguir com a análise, já vimos que não existe nada de especial no número 10, portanto, por pura preguiça, a partir de agora vou mudar para base 2, mas deixo ao leitor desconfiado a tarefa de reescrever tudo o que se segue em base 10. Apenas para começar do ponto onde paramos, o gráfico acima para base 2 fica assim:

Page 18: Eureka 2009

Sociedade Brasileira de Matemática

EUREKA! N°29, 2009

18

0,1

0,00,0 0,1

Relembrando, o problema está no ponto da descontinuidade, neste caso o 0,1 (também conhecido como ½). A origem do problema é o fato de haver números começados por 0,0 arbitrariamente próximos de 0,1, quando, naturalmente, números começados por 0,0 deveriam ser distintamente menores do que 0,1. Vamos tentar corrigir este problema dando um pequeno espaço entre os intervalos no nosso desenho.

0,1

0,00,0 0,1

Vamos passar agora para o segundo dígito.

0,1

0,00,00 0,01 0,10 0,11

Page 19: Eureka 2009

Sociedade Brasileira de Matemática

EUREKA! N°29, 2009

19

O espaço que demos entre os intervalos do primeiro dígito naturalmente se propaga para o segundo dígito (senão teríamos o mesmo problema nos números 0,01 – ¼ - e 0,11 – ¾). [Em base 10 seriam 9 buracos na primeira rodada e 90 na segunda, entenderam a minha preguiça?] Neste ponto, já não é difícil imaginar o processo para o terceiro dígito – criar mais um buraco no meio de cada um dos 4 intervalos restantes, e assim por diante. O conjunto dos números que sobram depois de fazermos este processo infinitas vezes é um conjunto em que podemos usar a nossa representação de base 2 (ou base 10, se fizermos o processo no desenho original lá de cima) sem medo – cada número tem apenas uma representação, e elas respeitam a ordem e as distâncias que esperaríamos: por exemplo, se um número começa por 0,11010 ele é estritamente menor que qualquer número que comece por 0,11011. O único problema é que o conjunto que temos no final deste processo é tão esfarelado que ele não contém nem um intervalinho que seja. Se escolhêssemos um número entre 0 e 1 aleatoriamente, ele teria 0% de chance de estar no nosso conjunto – repare que eu disse 0% não uma em um quaquilhão elevado a um googleplex, não a chance de ganhar na megasena todas as semanas pelo resto da vida, mas zero! Este conjunto é tão importante que tem um nome – conjunto de Cantor, em homenagem ao matemático Georg Cantor que, entre outras coisas criou a teoria dos conjuntos (todos os conjuntos então são um pouquinho conjuntos de Cantor) e provou que existiam mais números irracionais do que números racionais. Dado que acabamos de ver que o próprio sistema de representação decimal que usamos todos os dias na verdade é um conjunto de Cantor, é de se admirar que apenas no final do século XIX tenhamos passado a conhecê-lo. Pelo que acabamos de ver, o nosso sistema de numeração é um mapeamento entre um conjunto de Cantor e o intervalo [0,1]. O cerne da nossa questão é que esta função não é injetiva, ou seja, existem elementos distintos a e b no conjunto de Cantor que são mapeados para o mesmo número real (a diferente de b, f(a) = f(b)), por exemplo, 0,19999... e 0,2. [Parênteses: outra forma de ver isto é afirmar que o intervalo [0,1] é o espaço quociente do conjunto de Cantor pela relação de equivalência entre os números terminados por 999... e os seus correspondentes terminados por 000..., ver o artigo Egalité] no endereço:

http://www.impa.br/~gugu/pablo-egalite.doc Vamos tentar desenhar o gráfico desta função:

Page 20: Eureka 2009

Sociedade Brasileira de Matemática

EUREKA! N°29, 2009

20

Os dois extremos do buraco do meio são os pontos 0,01111.... e 0,10000... que representam o mesmo número ½, portanto a função é constante igual a ½ neste intervalo. Da mesma forma cada um dos buracos de “gerações” maiores corresponde a diferentes platôs no gráfico. O mais surpreendente é que, depois de criarmos os infinitos platôs, o gráfico final é um gráfico contínuo, que é chamado de função de Cantor, ou de “escada do diabo” (porque sempre existem infinitos degraus entre quaisquer dois degraus da escada, você nunca conseguiria sair do lugar – assim como Aquiles correndo atrás da tartaruga). Infelizmente, é o que se tem quando se tenta botar um bloco quadrado em um buraco redondo, ou neste caso, botar um intervalo dentro de um conjunto de Cantor.

Page 21: Eureka 2009

Sociedade Brasileira de Matemática

EUREKA! N°29, 2009

21

ALGORITMO DE GOSPER E APLICAÇÕES Humberto Silva Naves

• Nível Avançado

Continuando com as idéias do artigo “Integrais discretas” (de Eduardo

Poço na Eureka número 27), vamos tentar descobrir “fórmulas fechadas” para alguns somatórios da forma:

1

0

(*)n

kk

z−

=∑

Algumas considerações devem ser feitas antes de continuarmos:

1- Vamos assumir que kz é uma seqüência hipergeométrica, isto é, a razão

1( ) /k kr k z z+≡ é uma função racional de k. 2- Por “fórmula fechada”, entendemos que existe uma seqüência

hipergeométrica ns tal que 10

nn k ks z−

== ∑ (essa definição não é tão

restritiva assim, pois veremos que a classe das seqüências hipergeométricas é bastante ampla).

Por exemplo, vamos tentar achar uma fórmula fechada para:

1

0

,n

n kk

s z−

=

= ∑ onde

2

2

2

( 1)4k k

kk

zk

=+

Claramente kz é hipergeométrica, uma vez que 2

1

(2 1)( ) /

4( 1)( 2)k k

kr k z z

k k+

+= =

+ + é

uma função racional de k. Por (*), vale:

11 1n n

n n nn n

z sz s s

s s+

+= − ⇔ = −

A nossa fórmula fechada ns é hipergeométrica, logo 1

1 1n

n

ss

+ −

é uma função

racional y(n), portanto:

1( ) ( 1) ( )n n n n ns y n z y n z y n z z+= ⋅ ⇔ + − = ⇔

Page 22: Eureka 2009

Sociedade Brasileira de Matemática

EUREKA! N°29, 2009

22

( ) ( 1) ( ) 1r n y n y n⇔ + − =

onde: 2(2 1)

( )4( 1)( 2)

nr n

n n+

=+ +

Como y(n) é racional, existem polinômios ( ), ( )P n Q n com ( ), ( ) 1mdc P n Q n =

tais que ( )

( )( )

P ny n

Q n= e substituindo na equação anterior, vale:

2(2 1) ( 1) ( ) 4( 1)( 2) ( 1) ( )n P n Q n n n Q n P n+ + − + + + =

4( 1)( 2) ( 1) ( ), 0n n Q n Q n n= + + + ∀ ≥

Disto concluí-se que:

1- ( ) | 4( 1)( 2) ( 1)Q n n n Q n+ + +

2- 2 2( 1) (2 1) ( ) ( ) (2 1) ( 1)Q n n Q n Q n n Q n+ + ⇔ − −

(Observação: todas as relações de divisibilidade e os mdc´s referencem-se aos polinômios em si e não aos seus valores em um dado ponto)

Por (1), temos que se α é uma raiz de Q então: α = –1 ou α = –2 ou α + 1 é raiz de Q. E por (2) se α é raiz de Q então: 1

2α = ou α – 1 também é raiz de Q. Portanto,

como Q não pode ter infinitas raízes, Q não possui raiz alguma, isto é, Q(n) é constante (sem perda de generalidade, Q(n) = 1).

Disto concluí-se que: 2(2 1) ( 1) 4( 1)( 2) ( ) 4( 1)( 2)n P n n n P n n n+ + − + + = + +

Logo: ( 1)( 2) ( 1) ( 1) ( )n n P n n n P n+ + + ⇔ + , daí ( ) ( 1) ( )P n n n P n= + , então vale:

2(2 1) ( 1) 4 ( 1) ( ) 4,n P n n n P n+ + − + = onde ( )P n é um polinômio

Seja 0deg , ( ) ... ;ddd P P n a n a= = + + substituindo na equação anterior, vale (para o

caso d > 0): 2 2

0 0(4 4 1)( ( 1) ... ) (4 4 )( ... ) 4d dd dn n a n a n n a n a+ + + + + − + + + = ⇔

2 11(4 4 1)( ( ) ...)d d

d d dn n a n a da n −−+ + + + +

2 11(4 4 )( ...) 4d d

d dn n a n a n −−− + + + = ⇔

Page 23: Eureka 2009

Sociedade Brasileira de Matemática

EUREKA! N°29, 2009

23

14 ... 4ddda n + + =

Mas, se 0,d > 14 ... 4ddda n + + = é um polinômio de grau d + 1. Portanto

0 ( ) 4d P n= ⇒ = (que de fato é solução da equação anterior), donde concluímos que: ( ) 4 ( 1),y n n n= + logo:

2 2 2

1

2 2 2 10

2 2 2

4 ( 1)( 1)4 ( 1)4 4

n

k n nk

k n nn

k n nn n

k n

−=

= + =+ +∑

(é fácil ver que a fórmula vale para n = 0 e n = 1, e, pela identidade que provamos, vale sempre).

Vamos agora generalizar essa idéia para qualquer seqüência hipergeométrica kz . O leitor atento deve ter notado que na conclusão de que Q(n) = 1, implicitamente usamos o fato que: mdcA(n), B(n + h) = 1, para qualquer inteiro 0h ≥ , onde

2( ) (2 1) ,A n n= + e ( ) 4( 1)( 2)B n n n= + + , são o numerador e o denominador de r(n) respectivamente.

Mas nem sempre é possível escrever 1( ) n

n

zr n z+= como a razão de dois

polinômios satisfazendo as condições:

( ), ( ) 1, .mdc A n B n h h ++ = ∀ ∈¢

O leitor pergunta: “Então essa técnica não se aplica para todos os casos?!?!?”.

O autor responde: “Não se desesperem!”, uma vez que é possível escrever r(n) da seguinte forma:

( ) ( 1)(**) ( )

( ) ( )A n C n

r nB n C n

+=

onde A(n), B(n), C(n) são polinômios tais que: ( ), ( ) 1, .mdc A n B n h h ++ = ∀ ∈¢

Não vamos demonstrar esse fato aqui (pois é um dos exercícios deste artigo), mas

vamos exibir um exemplo bem ilustrativo! Como escrever 2

( 4)( 3)( )

( 1)n n

r nn n+ +

=+

da

forma (**)?

Note que:

Page 24: Eureka 2009

Sociedade Brasileira de Matemática

EUREKA! N°29, 2009

24

1 4 3 1 4 3 2 1 3 2 ( ) ( 1)( ) ,

1 3 2 1 2 1 ( ) ( )n n n n n n n n A n C n

r nn n n n n n n n n n B n C n

+ + + + + + + + += ⋅ ⋅ = ⋅ ⋅ ⋅ ⋅ ⋅ ⋅ =

+ + + + + +

onde A(n) = 1, B(n) = n e 2 2( ) ( 1) ( 2) ( 3).C n n n n n= + + +

De forma geral se α é uma raiz do denominador de r(n) e α – h é uma raiz do numerador de r(n), onde h +∈¢ , então vale:

( ) 1 1 ( 1)1 2 ( )

n h n h n h n T nn n h n h n T n

α α α αα α α α

− − − + − + − − + += ⋅ ⋅ ⋅ =

− − + − − + − −…

Onde ( ) ( 1)( 2) ( )T n n h n h nα α α= − + − − + − −L

Vamos tentar usar a fórmula (**) em

r(n) y(n + 1) – y(n) = 1 ⇔

( ) ( 1)( 1) ( ) 1

( ) ( )A n C n

y n y nB n C n

+⇔ + ⋅ − =

Como y(n) é racional em n, podemos fazer a substituição ( 1)

( ) ( ),( )

B ny n y n

C n−

=

portanto:

( ) ( 1) ( 1) ( ) ( ) (***)A n y n B n y n C n+ − − =

Agora o milagre acontece! Se ( )y n é uma função racional que satisfaz (***), então

( )y n é um polinômio! Se ( )

( ) ,( )

P ny n

Q n= com ( ), ( ) 1,mdc P n Q n = então:

( ) ( 1) ( ) ( 1) ( ) ( 1) ( ) ( ) ( 1)A n P n Q n B n P n Q n C n Q n Q n+ − − + = +

Logo:

1- ( ) ( 1) ( 1)Q n B n Q n− +

2- ( 1) ( ) ( ) ( ) ( 1) ( 1)Q n A n Q n Q n A n Q n+ ⇔ − +

Logo se α é raiz de Q , então:

1- α é raiz de B(n – 1) ou α é raiz de ( 1)Q n +

Page 25: Eureka 2009

Sociedade Brasileira de Matemática

EUREKA! N°29, 2009

25

2- α é raiz de A(n – 1) ou α é raiz de ( 1)Q n −

Como ( ), ( ) 1,mdc A n B n h+ = , ( ) 1.h Q n+∀ ∈ =¢

Vamos agora resolver a seguinte equação polinomial:

( ) ( 1) ( 1) ( ) ( ),A n y n B n y n C n+ − − = onde 0( ) ...ddy n a n a= + +

Temos casos a considerar

1- deg degA B≠ 2- deg degA B= e A Bδ δ≠ ( Aδ é o coeficiente líder de A):

Nesses casos, pela equação (***), vale: deg deg deg ,deg C y max a b= + ⇔

deg deg ,deg .d C max A B= − 3- deg degA B m= = e A B kδ δ= =

Se d > 0, então: 1

0( ...)( ( 1) )m m ddkn an a n a−+ + + + +…

10( ...)( ) ( )m m d

dkn bn a n a C n−− + + + + = ⇔… 1 1

1( ...)( ( ) )m m d dd d dkn an a n a da n− −

−+ + + + +… 1 1

1( ...)( ) ( )m m d dd dkn bn a n a n C n− −

−− + + + + = ⇔… 1

1 1[( ( )) ( )] ( )m dd d d d da a k a da b a ka n C n+ −

− −⋅ + + − ⋅ + + =…

Se ( ) 0 ,d d

b aa b a kda d

k−

− ⋅ + = ⇒ = mas se

( ) 0 deg 1 deg deg 1d da b a kda C m d d C A− ⋅ + ≠ ⇒ = + − ⇒ = − + Em todos os casos, é possível calcular o valor de d e uma vez calculado o valor de d, a equação polinomial se transforma em um sistema linear com d variáveis que pode ser resolvido (quando possível) usando técnicas básicas de álgebra linear.

Exercícios:

1) Calcule os seguintes somatórios

a) 10 2

15 1

nk

k k−= + −

b) 4

10

42

knk

kk

k

−=

Page 26: Eureka 2009

Sociedade Brasileira de Matemática

EUREKA! N°29, 2009

26

c) 1 20

n kk k a−

=∑

2) Prove que qualquer função racional r(n), pode ser escrita como: ( ) ( 1)

( )( ) ( )

A n C nr n

B n C n+

= com ( ), ( ) 1,mdc A n B n h h ++ = ∀ ∈¢ e

( ), ( ) ( ), ( 1) 1mdc A n C n mdc B n C n= + = (o leitor atento novamente notará que adicionamos duas novas restrições).

3) Prove que a menos de multiplicações por constantes, A(n), B(n) e C(n) são únicos no exercício anterior (a forma acima chama-se “forma canônica”).

4) Dizemos que duas seqüências hipergeométricas são similares, quando a razão dos duas é uma função racional e neste caso, escrevemos .n ns t∼ Prove:

a) Se ns é não constante, então ,n ns s∆ ∼ onde 1 .n n ns s s+∆ = −

b) Se ns e nt são hipergeométricas, e 0n ns t+ ≠ então n ns t+ é hipergeométrica se e somente se .n ns t∼ (Esse resultado pode levar o leitor a indagar sobre a nossa suposição inicial do que seria uma “fórmula fechada” ser algo bem restritivo).

c) Se (1) (2) ( ), ,..., mn n nt t t são hipergeométricas e vale ( )

1 0,m ii nt= =∑ então ( ) ( )i j

n nt t∼ para

algum i, j com 1 .i j m≤ < ≤

d) Se vale 1 (1) (2) ( )0 ...n m

k k n n nz t t t−= = + + +∑ , onde (1) (2) ( ), ,..., m

n n nt t t e nz são

hipergeométricas, então 10

nk kz−

=∑ é hipergeométrica! (Ufa! De fato nossa definição

de fórmula fechada não é tão restritiva assim).

Page 27: Eureka 2009

Sociedade Brasileira de Matemática

EUREKA! N°29, 2009

27

DOMINGO REGADO A REPUNITS Valberto Rômulo Feitosa Pereira

Cefetce – Uned Cedro • Nível Iniciante

No final do ano de 2007 fui convidade pelo professor e amigo Onofre

Campos, por quem tenho admiração, para ministrar aulas para um jovem que não podia se locomover, pelo valor que eu iria receber pelas aulas, pensei que o rapaz pertencia a uma família muito rica; grande foi a minha surpresa ao perceber justamente o contrário: em segunda conversa com o supra-citado professor, soube da frágil situação financeira do jovem aluno, mas também, por outro lado, do seu incrível potencial e de sua força de vontade, fatos que me entusiasmaram em conhecê-lo. Este aluno era Ricardo Oliveira, o qual havia conquistado duas medalhas de Ouro na OBMEP.

No último encontro que tive com Ricardo, em sua residência, ainda promovido pelo projeto de iniciação científica, deparamo-nos com o seguinte problema:

“O inteiro A é formado por 666 algarismos iguais a 3, e o número B por 666 algarismos iguais a 6. Que algarismos apareceram no produto AB?”

Enquanto Ricardo fazia uma atividade, eu folhava uma apostila que continha as colunas semanais – Olimpíada de Matemática – do jornal O Povo em parceria com o Departamento de Matemática da UFC. Neste momento, vi o problema acima e falei:

- Olha Ricardo que belo problema!

Nesse instante Ricardo para sua atividade, lê o problema e passa a resolvê-lo. Eu também caio na tentativa de resolvê-lo, lembrei que:

algarismos 1

10 1111111...11 .

9

n

n

−=14243

Minha solução com o uso desta informação saiu; Ricardo sem esta informação errou por um algarismo. Expliquei a Ricardo minha solução, percebemos que a informação que eu havia usado era importante. A aula continuou, mas ainda fiquei pensando como esta igualdade daria para resolver belos problemas.

No dia seguinte tive uma conversa com meu amigo Secco, olímpico do Rio de Janeiro. Perguntei-lhe se conhecia problemas que em sua solução usava esta

Page 28: Eureka 2009

Sociedade Brasileira de Matemática

EUREKA! N°29, 2009

28

igualdade; Secco falou que conhecia e mais ainda: estes números eram chamados de Repunits e indicou [4]. Com a dica de Secco e o entusiasmo de Ricardo, cataloguei cinco problemas da antiga coluna, os quais passaremos a resolver. Também apresentei aos meus alunos do projeto OBMEP 2008, realizado no Cefet, Uned de Cedro-Ce.

1. REPUNITS

Os Repunits são números que só têm algarismos 1, por exemplo:

11, 111, 1111, 11111, ...

Estes números podem ser escritos de outra forma, vejamos:

999...9 1000...0 1 10 1111...1 .

9 9 9

k k

k

k

− −= = =

678 678123

A beleza destas informações é poder resolver problemas interessantes sem usar técnicas sofisticadas.

2. EXEMPLOS

Exemplo 1: O inteiro positivo n é formado de k algarismos 9. Mostre que a soma de todos os algarismos de n2 é igual a 9k. Demonstração: Pelas hipóteses temos

10 1999...9 9(111...1) 9 10 1.

9

kk

k

N−

= = = = −123

Calculemos N2 da seguinte forma: 2

2

2

2

1 1

.

(999...9).(10 1)

999...9000...0 999...9

999...98000...01

k

k k k

k k

N N N

N

N

N− −

=

= −

= −

=

123 123 123

123 123

A soma dos algarismos é: 9( 1) 8 1 9 .k k− + + =

Page 29: Eureka 2009

Sociedade Brasileira de Matemática

EUREKA! N°29, 2009

29

Exemplo 2: Mostre que os números 49, 4489, 444889, ..., obtidos colocando o número 48 no meio do número anterior, são quadrados de números inteiros. Demonstração: Vejamos as igualdades:

1

2

3

49 4.1.10 8.1 1

4489 4.11.10 8.11 1

444889 4.111.10 8.111 1

= + +

= + +

= + +

De modo geral temos: 1

444...488...89 4.111...1.10 8.111...1 1.n

nn n n

N−

= = + +123 123 123

Substituindo 10 1

11...119

n

n

−=123 na expressão acima ficamos:

2

2

4 8(10 1).10 (10 1) 1

9 94 4 8 8

10 10 10 19 9 9 9

2.10 13

n n n

n n n

n

N

N

N

= − + − +

= − + − +

+=

O número 2.10 1n + é múltiplo de 3, portanto N é um quadrado perfeito. Exemplo 3: Para cada inteiro positivo n, sejam A(n) e B(n) dois números inteiros formados por 2n algarismos iguais a 1 e n algarismos iguais a 2 respectivamente. Mostre que A(n) – B(n) é um quadrado perfeito. Demonstração. Pelas hipóteses temos:

2

( ) ( ) 111...1 222...2n n

A n B n− = −123 123

Como 10 1

222...2 29

n

n

−=123 e

2

2

10 1111...1 ,

9

n

n

−=123 substituindo teremos:

210 1 10 1( ) ( ) 2

9 9

n n

A n B n− −

− = −

(10 1)(10 1) 10 1( ) ( ) 2

9 9

n n n

A n B n− + −

− = −

(10 1)[(10 1) 2]( ) ( )

9

n n

A n B n− + −

− =

2(10 1)( ) ( )

9

n

A n B n−

− =

Page 30: Eureka 2009

Sociedade Brasileira de Matemática

EUREKA! N°29, 2009

30

2

2

(10 1)( ) ( ) .

3

n

A n B n−

− =

Assim ( ) ( )A n B n− é quadrado perfeito. Exemplo 4: Sem efetuar a multiplicação, calcule o valor de 2(999.999.999) .

Solução: Vamos escrever a expressão 2(999.999.999) da seguinte maneira: 29

2 18 9

18 zeros 9 zeros

10 1(999.999.999) 9. 10 2.10 1 1000...0 2000...0 1

9 − −

−= = − + = − +

123 123

fazendo as contas ficamos: 2

8 noves 8 zeros

(999.999.999) 999...98000...01− −

= 123 123

Finalmente o problema motivdor do nosso trabalho. Exemplo 5: O inteiro A é formado por 666 algarismos iguais a 3, e o número B por 666 algarismos iguais a 6. Que algarismos apareceram no produto AB?

Solução: Como 666 666

666...6 6.111...1A = =123 123 e 666

666 666

10 1333...3 3.111...1 3. ,

9B

−= = =123 123 vamos

calcular AB mas usando alguns artifícios, como segue abaixo: 66610 1

3.6.(111...1).9

AB−

=

6662.(111...1)(10 1)AB = − 666(222...2)(10 1)AB = − 666(222...2).10 222...2AB = −

666 666 666

222...2000...0 222...2AB = −123 123 123

665 665

222...21777...78AB = 123 123

Logo apareceram no produto AB: - Um algarismo 1; - Um algarismo 8; - 665 algarismos 2; - 665 algarismos 7.

Page 31: Eureka 2009

Sociedade Brasileira de Matemática

EUREKA! N°29, 2009

31

3. PROBLEMAS PROPOSTOS

1. Achar a soma: 2 22 ... 222...2+ + +

se a última parcela tem n algarismos iguais a 2. 2. Prove que: 2

2

111...1 222...2 (333...3) .n n n

= +123 123 123

3. Prove que se 111...1n

123 divisível por 41 se e somente se n é divisível por 5.

4. Mostre que nenhum inteiro da seqüência: 11,111,1111,11111,... é um quadrado perfeito.

5. Mostrar que os inteiros: 1111,111111,..., cada um dos quais é formado por

um número par de algarismos 1, são compostos. Nota dos editores: Não é difícil mostrar que se 111...1

n123 é primo então n é primo

(exercício!) . Os únicos valores de n para os quais se sabe provar atualmente que 111...1

n123 é primo são 2, 19, 23, 317 e 1031. Recentemente (entre 1999 e 2007) foram

descobertos os seguintes valores de n tais que 111...1n

123 é provavelmente primo (i.e.,

passa por diversos testes probabilísticos de primalidade): 49081, 86453, 109297 e 270343. De acordo com os testes já realizados, qualquer outro repunit primo deve ter mais de 400.000 algarismos.

REFERÊNCIAS

[1] Emanuel Carneiro, Francisco Antonio M. de Paiva, Onofre Campos, Olimpíadas Cearenses de Matemática do Ensino Fundamental, Edições Realce Editora e Indústria Gráfica, Fortaleza, 2006.

[2] Alencar Filho, Edgar de, Teoria Elementar dos Números, Nobel, São Paulo, 1988. [3] Coluna Semanal Olimpíadas de Matemática, Jornal O Povo em parceria com o

Departamento de Matemática da UFC, No. 01, ao No. 200. [4] Titu Andreescu, Razvan Gelca, Mathematical Olympiad Challenges, 2000.

Page 32: Eureka 2009

Sociedade Brasileira de Matemática

EUREKA! N°29, 2009

32

HOMOTETIAS, COMPOSIÇÃO DE HOMOTETIAS E O PROBLEMA 6 DA IMO 2008

Carlos Yuzo Shine • Nível Avançado

Antes de começar a discussão, vamos enunciar o problema 6 da IMO 2008,

que é a motivação principal desse artigo. Problema 6, IMO 2008. Seja ABCD um quadrilátero convexo cujos lados

BA e BC têm comprimentos diferentes. Sejam 1w e 2w as circunferências inscritas nos triângulos ABC e ADC, respectivamente. Suponhamos que existe uma circunferência w tangente à reta BA de forma que A está entre B e o ponto de tangência, tangente à reta BC de forma que C está entre B e o ponto de tangência, e que também seja tangente às retas AD e CD. Prove que as tangentes comuns exteriores a 1w e 2w se intersectam sobre .w

É claro que um problema de geometria não pode ficar sem um bom desenho. É razoavelmente difícil desenhar a figura do problema e sugerimos que o leitor tente fazê-lo por conta própria (dica: comece com o círculo w ). Não se perca: queremos provar que o ponto Z está sobre a circunferência .w

B

O

w

Z D O2

C

A

K O1 w1

w2 L

Quem já estudou homotetia já deve ter enxergado diversas homotetias entre as circunferências, mas muitos dos mais poderosos olímpicos do mundo foram derrotados por esse problema. De fato, dos 535 estudantes que participaram da IMO 2008, somente 13 resolveram (um deles fez 6 pontos) e 53 conseguiram pelo menos um ponto. Isto quer dizer que mais de 90% dos estudantes zeraram o problema!

Page 33: Eureka 2009

Sociedade Brasileira de Matemática

EUREKA! N°29, 2009

33

Isso é sinal de que esse problema deve ter algo novo para ser explorado. De fato, uma transformação geométrica que esteve em voga nos anos 80 e desapareceu nos anos 90 foi a homotetia. E ela voltou, discretamente em 2007 e com tudo em 2008! Vamos definir homotetia, ver algumas de suas propriedades e expandir as adéias envolvidas nessa transformação. 1. Homotetia: definição Você vai ver que homotetia nada mais é do que “fazer sombrinha”. Aparecem muitos paralelismos, mas o mais interessante são as colinearidades que irão aparecer. No início parece mágica; mas um bom matemático sempre revela seus truques! Vamos começar com a definição de homotetia com razão positiva ou homotetia direta: Definição 1.1. Homotetia de uma figura F com centro O e razão k, um número real positivo, é uma transformação geométrica que associa a cada ponto P de F o ponto P´ sobre a semi-reta OP, de origem O, tal que ´ .OP k OP= ⋅

O

A

B

F

Talvez com vetores seja mais interessante: sendo O o centro da homotetia, o ponto P é transformado no ponto P´ de modo que ´ .OP k OP= ⋅

uuur uuur Note que a homotetia é

uma função σ que leva pontos do plano (ou do espaço, se você estiver trabalhando em dimensões maiores) a pontos do plano (espaço). De fato, podemos fazer

´ ( ),P Pσ= tal que ( ) ( ) ( ) ( ).P O k P O P O k P Oσ σ− = ⋅ − ⇔ = + ⋅ − Com isso, podemos definir homotetias para k negativo também, obtendo as chamadas homotetias de razão negativa ou homotetias inversas:

Page 34: Eureka 2009

Sociedade Brasileira de Matemática

EUREKA! N°29, 2009

34

Definição 1.2. Homotetia de uma figura F com centro O e razão k, sendo k um número real negativo, é uma transformação geométrica que associa a cada ponto P de F o ponto P´sobre a reta OP, de origem O, tal que ´ .OP k OP= ⋅

uuur uuur

A

C

O B´

2. Propriedades da homotetia As principais propriedades de homotetias têm a ver com colinearidade e concorrência. Algumas têm a ver com paralelismo. 2.1. Colinearidade

• O centro de homotetia, o ponto e seu transformado são colineares. Em outras palavras, , e ´ ( )O P P Pσ= são colineares. Isso decorre diretamente da definição, mas homotetias não vêm de graça!

Normalmente as encontramos nos problemas e, com essa propriedade, obtemos pontos colineares.

2.2. Concorrência • O centro de homotetia pertence a todas as retas que ligam pontos a seus

transformados. Em outras palavras, O pertence a toda reta do tipo ´ ( ).PP P Pσ= Novamente, uma propriedade que decorre diretamente da definição (na verdade, é a mesma da colinearidade!), mas que aparece quando descobrimos alguma homotetia.

2.3. Paralelismo

• A reta que liga dois pontos é paralela à reta que liga os seus tranformados. Em outras palavras, PQ e ´ ´ ( ) ( )P Q P Qσ σ= são paralelas. A demonstração desse fato vem da semelhança entre OPQ e ´ ´OP Q (pelo caso )LAL .

Page 35: Eureka 2009

Sociedade Brasileira de Matemática

EUREKA! N°29, 2009

35

• Dois triângulos com lados respectivamente paralelos são homotéticos. Para provar isso, sendo ABC e DEF os triãgulos com AB, DE, AC, DF e BC, EF respectivamente paralelos, use o teorema de Desargues para provar que esses triângulos são perspectivos.

Em particular, algumas figuras são sempre semelhantes: os círculos! Com isso, temos a seguinte propriedade:

2.4. Círculos

• Dois círculos são sempre homotéticos. Na maioria dos casos, eles admitem

duas homotetias, uma direta e uma inversa. No caso de círculos disjuntos, os centros de homotetias são fáceis de encontrar: são as interseções das tangentes comuns internas (inversa) e das tangentes comuns externas (direta).

A B O– O+

Com isso, podemos resolver alguns problemas. Homotetia esteve bastante na moda na IMO durante o início dos anos 80, como você vai ver nos exemplos e nos exercícios.

Exemplo 2.1.

Problema 5, IMO 1981. Três círculos congruentes têm um ponto comum O e estão no interior de um triângulo. Cada círculo é tangente a dois lados do triângulo. Prove que o incentro e o circuncentro do triângulo e o ponto O são colineares. Resolução: O nome do ponto dado não é O por acaso: sejam A, B e C os centros dos três círculos congruentes e ´ ´ ´A B C o triângulo cujos lados tangenciam esses três

Page 36: Eureka 2009

Sociedade Brasileira de Matemática

EUREKA! N°29, 2009

36

círculos. Note que os raios dos círculos congruentes são ,OA OB OC= = isto é, O é circuncentro de ABC. Além disso, das tangências dos círculos com os lados temos que AA´, BB´ e CC´são as bissetrizes do triângulo A´B´C´ e se interceptam no incentro I do triângulo.

A

B

C

O´ O

I

As distâncias de A e B a A´B´ são iguais aos raios dos círculos congruentes a são, portanto, iguais. Então AB e A´B´ são paralelos. Analogamente, AC é paralelo a A´C´ e BC é paralelo a B´C´, de modo que os triângulos ABC e A´B´C´ são homotéticos. O centro de homotetia é I. Essa homotetia leva O ao circuncentro O´ de A´B´C´. Assim, I, O e O´ são colineares. Note que a dificuldade foi achar a homotetia; depois bastou aplicar a propriedade de colinearidade. Exercícios: 01. (Problema 2, IMO 1982) Seja 1 2 3A A A um triângulo escaleno com lados 1 2 3, e a a a ( ia é o lado oposto a ).iA Seja iM o ponto médio do lado ia e iT o ponto onde o incírculo do triângulo toca o lado ,ia para i = 1, 2, 3. Seja iS o simétrico de iT em relação à bissetriz interna do ângulo .iA Prove que as retas 1 1 2 2,M S M S e 3 3M S são concorrentes.

Page 37: Eureka 2009

Sociedade Brasileira de Matemática

EUREKA! N°29, 2009

37

02. (Problema 2, IMO 1983) Seja A um dos dois pontos de interseção dos círculos 1C e 2 ,C de centros 1O e 2 ,O respectivamente. Uma das tangentes comuns aos círculos toca 1C em 1P e 2C em 2 ,P e a outra toca 1C em 1Q e 2C em 2Q . Seja

1M o ponto médio de 1 1PQ e 2M o ponto médio de 2 2 .P Q Prove que

1 2 1 2 .O AO M AM∠ = ∠ 03. (Prova de seleção 2008, Banco da IMO 2007) As diagonais do trapézio ABCD cortam-se no ponto P. O ponto Q está na região determinada pelas retas paralelas BC e AD tal que AQD CQB∠ = ∠ e a reta CD corta o segmento PQ. Prove que

.BQP DAQ∠ = ∠ 3. O Fenômeno Homotético Circular Algumas aplicações de certos teoremas são tão conhecidos quanto os próprios. Para homotetias, é o caso com o fenômeno homotético circular, que mostra uma colinearidade bastante interessante envolvendo incírculo e ex-incírculo. Fenômeno Homotético Circular. Seja ABC um triângulo e sejam K e L os pontos de tangência do incírculo e ex-incírculo relativo a A em BC. Então A, L e o ponto K´ diametralmente oposto a K no incírculo são colineares. Demonstração:

A

B K

I

C

B´ C´

L

I A

Basta traçar a reta B´C´ paralela a BC que tangencia o incírculo de ABC em K´. ABC e AB´C´ são homotéticos com centro em A. Para terminar, o incículo de ABC é ex-incírculo de AB´C´, de modo que os pontos K´e L são correspondentes na homotetia e estão, portanto, alinhados com A.

Page 38: Eureka 2009

Sociedade Brasileira de Matemática

EUREKA! N°29, 2009

38

Vale a pena lembrar também que, na figura acima, BK = LC. Exercícios: 04 (Problema 4, IMO 1992) No plano, considere uma circunferência C, uma reta L tangente à circunferência e M um ponto da reta L. Encontre o lugar geométrico dos pontos P com a seguinte propriedade: existem dois pontos Q, R da reta L tais que M é o ponto médio de QR e C é a circunferência inscrita no triângulo PQR. 4. Composição de Homotetias A principal inovação na IMO 2008 no problema 6 foi explorar o seguinte fato: Composição de Homotetias. Se 1σ é uma homotetia de centro 1O e 2σ é uma homotetia de centro 2O então a composição de homotetias 2 1σ σ σ= o é uma homotetia de centro O, e 1O , 2O e O estão alinhados. A única exceção é quando a composição σ é uma translação. Demonstração Utilizaremos vetores para provar esse fato Seja P um ponto qualquer e sejam 1k e 2k as razões de homotetia de 1σ e 2σ , respectivamente. Então 1 1 1 1( ) ( )P O k P Oσ = + ⋅ − e, portanto,

2 1 2 1 2 2 1 2( ) ( ) ( ( )) ( ( ) )P P P O k P Oσ σ σ σ σ σ= = = + ⋅ −o

2 2 1 1 1 2( ( ) )O k O k P O O= + ⋅ + ⋅ − −

2 1 1 2 2 1 2(1 ) (1 )k k O k O k k P= − ⋅ + − ⋅ + ⋅ (*)

Primeiro, se σ é uma homotetia, então sua razão é 1 2k k (as figuras são “multiplicadas” por 1k e depois por 2 ;k ou seja, são “multiplicadas” por 1 2 ).k k Assim, para provarmos que σ é uma homotetia, temos que provar que existe um ponto O tal que

1 2 1 2 1 2( ) ( ) (1 )P O k k P O k k O k k Pσ = + ⋅ − = − ⋅ + ⋅ (**) Comparando os coeficientes em (*) e (**) concluímos que

1 2 2 1 1 2 2(1 ) (1 ) (1 ) .k k O k k O k O− = − ⋅ + − ⋅ Se 1 2 1, k k σ= é uma translação

(verifique!). Caso contrário, 2 1 1 2 2

1 2

(1 ) (1 )1

k k O k OO

k k− ⋅ + − ⋅

=−

e, como

2 1 2 2 1 2 2 1 2(1 ) (1 ) 1 1 ,k k k k k k k k k− + − = − + − = − O é uma média ponderada de 1O e

2.O Em outras palavras, O pertence à reta 1 2.O O

Page 39: Eureka 2009

Sociedade Brasileira de Matemática

EUREKA! N°29, 2009

39

Os partidários da geometria sintética devem estar sentindo falta de uma demonstração sintética. Vamos provar a parte da colinearidade sinteticamente. Demonstração sintética da colinearidade Considere os pontos P e Q e seus transformados

1 1 1 1 2 2 1( ), ( ), ( ) ( )P P Q Q P P Pσ σ σ σ= = = = e 2 2 1( ) ( ).Q Q Qσ σ= =

O

O2

Q

P

O1 Q1

Q2

P1 P2

Note que, das homotetias, PQ 1 1PQ e 2 2P Q são paralelos. Em termos projetivos, eles são concorrentes em um ponto do infinito. Isto quer dizer que os triângulos

1 2PPP e 1 2QQ Q são perspectivos e podemos aplicar o teorema de Desargues: as interseções entre lados correspondentes, 1 1 1 1 2 1 2 2 , PP QQ O PP Q Q O∩ = ∩ = e

2 2 PP QQ O∩ = são colineares. 4.1 Detalhe técnico Geralmente, trabalhamos com homotetias sinteticamente, e aparecem homotetias diretas e inversas. Homotetias inversas “multiplicam” figuras por fatores negativos, de modo que a composição de duas homotetias do mesmo tipo é direta e a composição de duas homotetias de tipos diferentes é inversa. Para facilitar, a homotetia inversa faz o papel do sinal de menos e a homotetia direta, do sinal de mais. Na composição de homotetias, seguimos a regra dos sinais da multiplicação. Agora estamos prontos para resolver o problema 6 da IMO 2008. Vamos reenunciar o problema e resolvê-lo. Exemplo 4.1 Problema 6, IMO 2008. Seja ABCD um quadrilátero convexo cujos lados BA e BC têm comprimentos diferentes. Sejam 1w e 2w as circunferências inscritas nos triângulos ABC e ADC, respectivamente. Suponhamos que existe um

Page 40: Eureka 2009

Sociedade Brasileira de Matemática

EUREKA! N°29, 2009

40

circunferância w tangente à reta BA de forma que A está entre B e o ponto de tangência, tangente à reta BC de forma que C está entre B e o ponto de tangência, e que também seja tangente às retas AD e CD. Prove que as tangentes comuns exteriores a 1w e 2w se intersectam sobre .w Resolução Vamos começar trabalhando com segmentos tangentes.

B

E

O

G

Z

C w

D K

L

A

O1 w1

O2 w2

F

H

Temos BE = BF, AF = AG, CE = CH e DG = DH. Então AB = BF – AF = BE – AG = BC + CE – (AD + DG) = BC – AD + (CH – DH) = BC – AD + CD ⇒ AB + AD = BC + CD. Note que esse fato depende somente de w ser tangente aos prolongamentos dos lados do quadrilátero ABCD (guarde esse fato, ele pode ser útil em outros problemas!). Isso implica

.2 2

AC CD AD AC AB BCCK AL

+ − + −= ⇔ =

Essa igualdade é simples, mas abre muitas portas para nós! De fato, ela quer dizer que os ex-incírculos 3w e 4w relativos a AC dos triângulos ABC e ADC tocam AC em K e L, respectivamente. Isso nos dá muitas, mas muitas homotetias, e pelo menos duas oportunidades de utilizar o fenômeno hometético circular! Desenhemos as circunferências:

Page 41: Eureka 2009

Sociedade Brasileira de Matemática

EUREKA! N°29, 2009

41

O

w E

G

F

H

Z D

A

B

w3 w4

O2 O1 w1

L

C

w2 K

Vamos compor homotetias para descobrir colinearidades, utilizando 3w e 4w como “intermediários”!

• 24 412 4 1w w wσ σ→ → e 21

2 1.w wσ→ O centro K da homotetia (direta)

24σ , o centro B da homotetia (direta) 41σ e o centro Z da homotetia (direta)

21σ estão alinhados. Isso quer dizer que Z pertence à reta BK.

• 23 312 3 1w w wσ σ→ → e 21

2 1.w σ σ→ O centro D da homotetia (direta)

23 ,σ o centro L da homotetia (direta) 31σ e o centro Z da homotetia (direta) 21σ estão alinhados. Isso quer dizer que Z pertence à reta DL.

Com isso, concluímos que Z é a interseção de BK e DL.

Note que até agora não envolvemos o círculo w nas homotetias. Agora é hora, mas vamos provar colinearidades de outra forma. Seja W a interseção de BK e w. Provaremos que W = Z, resolvendo o problema.

O

w E

G

F

H

Z

D

A

B

w3 w4

O2 O1

w1 L

C

w2

K

r

T

s

Page 42: Eureka 2009

Sociedade Brasileira de Matemática

EUREKA! N°29, 2009

42

Primeiro, note que a homotetia direta 4σ que leva 4w a w tem centro B e, portanto, leva K a W. Mais ainda: como AC é tangente a 4w em K, a reta r paralela a AC que passa por W é tangente a w, pois a reta AC é levada a r por 4 .σ Agora, considere a homotetia inversa 2σ que leva w a 2w . Essa homotetia tem centro em D, leva W a T e r a s, que é paralela a r e AC e é tangente a 2w . Assim, D, W e T estão alinhados, ou seja, W pertence à reta DT. Falta ainda identificar melhor o ponto T. Na verdade, ele é bem conhecido: como s e AC são paralelos, T e K são diametralmente opostos. Podemos, assim, aplicar o fenômeno homotético circular: D, T e L são colineares e L também pertence à reta DT. Portanto D, L e W são colineares, de modo que W pertence a DL. Como W pertence, por definição, à reta BK, W é a interseção de BK e DL, e só pode ser igual a Z. Observação: Note que a condição AB ≠ AC é importante para que as retas BK e DL não coincidam. Exercícios 05. (Banco da IMO 2007) O ponto P pertence ao lado AB do quadrilátero convexo ABCD. Seja w o incírculo do triângulo DPD e I o seu incentro. Suponha que w é tangente aos incírculos dos triângulos APD e BPC em K e L, respectivamente. As retas AC e BD se encontram em E e as retas AK e BL se encontram em F. Prove que os pontos E, I e F são colineares. 06. (Romênia) Seja ABC um triâgulo e , ,a b cw w w círculos dentro de ABC tangentes exteriormente dois a dois, tais que aw é tangente a AB e AC, bw é tangente a AB e BC e cw é tangente a AC e BC. Sejam D o ponto de tangência entre bw e ,cw E o ponto de tangência entre aw e cw e F o ponto de tangência entre aw e .bw Prove que as retas AD, BE e CF têm um ponto em comum. 07. (Irã) Sejam w e Ω o incírculo e o circuncírculo do triângulo ABC. w toca BC, CA e AB em D, E e F respectivamente. Os três círculos ,a bw w e cw tangenciam w em D, E e F, respectivamente, e Ω em K, L e M, respectivamente. (a) Prove que DK, EL e FM têm um ponto P em comum. (b) Prove que o ortocentro do triângulo DEF pertence à reta OP.

Page 43: Eureka 2009

Sociedade Brasileira de Matemática

EUREKA! N°29, 2009

43

08. Seja Γ uma circunferência e A, B e C pontos em seu interior. Construa as seguintes três circunferências: 1Γ tangente a , ABΓ e AC; 2Γ tangente a , ABΓ e BC; 3Γ tangente a , ACΓ e BC. Sendo 1 2 3, e C C C os respectivos pontos de tangência de 1 2 3, ,Γ Γ Γ com Γ , prove que 1 2,AC BC e 3CC passam por um mesmo ponto.

Você sabia… Que 33661⋅27031232 + 1 é primo? Esse foi o décimo primeiro primo descoberto pelo projeto "seventeen or bust" e foi encontrado por Sturle Sunde em 17 de outubro de 2007. Isso mostra que 33661 não é um número de Sierpinski (números de Sierpinski são naturais ímpares k tais que k ⋅ 2n + 1 é composto para todo n ∈ N; veja a Eureka! 18, pág. 61 e a Eureka! 25 página 56), reduzindo para 6 o número de naturais menores que 78557 (que é o menor número de Sierpinski conhecido), sobre os quais não se sabe se são ou não números de Sierpinski: 10223, 21181, 22699, 24737, 55459 e 67607. Veja www.seventeenorbust.com para mais informações (inclusive sobre como participar do projeto).

Page 44: Eureka 2009

Sociedade Brasileira de Matemática

EUREKA! N°29, 2009

44

COMO É QUE FAZ? PROBLEMA PROPOSTO POR MARCEL MENEZES DE ANDRADE PRADO Seja ABC um triângulo e P um ponto em seu interior tal que AP, BP e CP intersectam os lados BC, CA e AB nos pontos D, E e F, respectivamente. Se

,AP a= ,BP b= ,CP c= 3PD PE PF= = = e a + b + c = 43, determine abc. SOLUÇÃO: Escrevemos P em coordenadas baricêntricas:

A

P

B C 2 3

2 3

t B t CD

t t+

=+

1 3

1 3

t A t CE

t t+

=+

1 2

1 2

t A t BF

t t+

=+

2 3, ,P t A t B t C= + + com 1 2 3, , (0,1)t t t ∈ tais que 1 2 3 1.t t t+ + = Devemos ter então

2 3

2 3

t B t CD

t t+

=+

(note que ( ) ( ) )2 31 1 1 1

1

1 1 ,1

t B t CP t A t t A t D

t +

= + − = + − − 1 3

1 3

t A t CE

t t+

=+

e 1 2

1 2

.t A t C

Ft t

+=

+

Temos

( ) 2 32 3 1 1 1

2 3 2 3

13 1

t B t CPD D P t B t C t A t A t D A

t t t t +

= = − = − + − = − = − = + +

Page 45: Eureka 2009

Sociedade Brasileira de Matemática

EUREKA! N°29, 2009

45

( )1 1 1( 3),t AD t AP PD t a= = + = + donde 1

3.

3t

a=

+ Analogamente, 2

33

tb

=+

e

3

3.

3t

c=

+ Como 1 2 3 1,t t t+ + = temos

3 3 31,

3 3 3a b c+ + =

+ + + donde

( )( ) ( )( ) ( )( )( ) ( )( )( )3 3 3 3 3 3 3 3 3 3 ,b c a c a b a b c+ + + + + + + + = + + + e logo

( )3 6 6 6 27 9 9 9 27,a b c abc a b c+ + + = + + + + e finalmente

( )9 54 9 43 54 441.abc a b c= + + + = ⋅ + = PROBLEMA PROPOSTO POR WILSON CARLOS DA SILVA RAMOS Determine todas as funções ( ): 0,F +∞ → ¡ que sejam deriváveis em x = 1 e que satisfaçam:

( ) ( ) ( ), , (0, ).F xy xF y yF x x y= + ∀ ∈ +∞ SOLUÇÃO: Fazendo y = x, obtemos 2( ) 2 ( ), (0, ).F x xF x x= ∀ + +∞ Com x = 1 temos (1) 2 (1),F F= donde (1) 0.F = Vamos mostrar por indução que

2 12 2( ) 2 ( ),

k

k kF x x F x−

= para todo inteiro positivo k e todo (0, ).x∈ +∞ Para k = 1 isso é a afirmação anterior e, supondo que isso vale para um certo k,

1

12 1 2 1

2 2 2 2 2 2 2 12 2( ) (( ) ) 2 ( ) 2 2 ( ) 2 ( ),k k

k k k k k k kF x F x x F x x x F x x F x+

+− −

+= = = ⋅ ⋅ ⋅ ⋅ = ⋅ ⋅ como queríamos mostrar.

Fazendo 2 ,khx e= obtemos 1

2 122( ) 2 ( ), 1.

k

kkh

h k hF e e F e k+−

⋅= ∀ ≥

Como 12 1 2

2 2 2 22 22

( ) (1)lim 2 ( ) lim2 ( ) lim2 ( 1)lim

1

k kk kk k

k

h hhk h h k h h k

hk k k k

F e Fe F e e F e e e

e+−

→+∞ →∞ →∞ →∞

−= = − =

2 (1),hhe F= temos 2( ) ,h hF e che= onde (1),c F= para todo h > 0.

Fazendo ,hx e= obtemos ( ) (log ) , (0, ).F x c x x x= ⋅ ∀ ∈ +∞ Todas as funções desse tipo satisfazem a equação funcional. De fato, ( )( ) log log ( ) ( ), , (0, ).F xy c x y xy xF y yF x x y= + = + ∀ ∈ +∞ Resolvemos a seguir, a pedido de Mauro Felix de Sousa, três problemas da seção “Olimpíadas ao redor do mundo”, propostos nas Eureka! No. 9 e 10.

Page 46: Eureka 2009

Sociedade Brasileira de Matemática

EUREKA! N°29, 2009

46

47 (Irã – 1999) Determine todas as funções :f →¡ ¡ que satisfazem

2( ( ) ) ( ) 4 ( )f f x y f x y f x y+ = − + para todos os números reais x e y. SOLUÇÃO: Fazendo x = y = 0, obtemos ( (0)) (0).f f f=

Fazendo 0, (0),x y f= = − obtemos 2(0) ( (0)) 4 (0) ,f f f f= − donde 24 (0) 0,f− = e

logo (0) 0.f = Fazendo agora y = 0, obtemos 2( ( )) ( ), .f f x f x x= ∀ ∈ ¡ Se ( ) ( )f a f b= com ,a b≠ teremos

2 2( ) ( ( ) ) 4 ( ) ( ( ) ) 4 ( ) ( ), ,f b y f f b y f b y f f a y f a y f a y y− = + − = + − = − ∀ ∈ ¡donde, se 2 2 , ( ) ( ), .t b a f z t f z z= − + = ∀ ∈¡ Note agora que, se 0,t ≠ fazendo y = t obtemos ( ( )) ( ( ) )f f x f f x t= + =

2 2( ) 4 ( ) ( ) 4 ( ) ( ( )) 4 ( ) , ,f x t f x t f x f x t f f x f x t x= − + = + = + ∀ ∈ ¡ donde 4 ( ) 0, ,f x t x= ∀ ∈¡ e logo ( ) 0, ,f x x= ∀ ∈¡ o que é uma solução (claramente). Por outro lado, fazendo x = 0 na equação funcional, obtemos

( ) ( ), .f y f y y= − ∀ ∈ ¡ Assim, se f não é identicamente nula,

( ) ( )f a f b a b= ⇔ = ou .a b= − Em particular, como 2( ( )) ( ), ,f f x f x x= ∀ ∈¡

para todo x ∈¡ devemos ter 2( )f x x= ou 2( ) .f x x=− Se para algum 20, ( ) ,x f x x≠ = − temos 2 2 2( ) ( ) ( ( ) ) ( ) 4 ( ) , ,f x y f y x f f x y f x y f x y y− = − = + = − + ∀ ∈¡

donde 24 4 ( ) 0, ,x y f x y y− = = ∀ ∈¡ absurdo. Assim, devemos ter 2( ) , .f x x x= ∀ ∈ ¡

Isso é outra solução, pois, de fato, 2 2 2 2 2( ) ( ) 4 , , .x y x y x y x y+ = − + ∀ ∈¡ 71 (Belarus – 2000) Determine todos os pares de inteiros positivos (m, n) que satisfazem a equação

2 2 2( ) ( ) 4 .m n n m m n− − = SOLUÇÃO: Seja ( , ).d mdc m n= Temos , ,m da n db= = com ( , ) 1,mdc a b = e a equação

equivale a 2 2 2( ) ( ) 4 .a b db a a b− − = Como ( , ) 1,mdc b a b− = segue que 2| ,b db a−

donde | ,b a e logo b = 1. Temos então 2 2( 1) ( ) 4 .a d a a− − = Como ( , 1) 1mdc a a − =

e 2 2( 1) | 4 ,a a− segue que 2( 1) | 4,a − donde a – 1 = 1 ou a – 1 = 2. No primeiro caso, temos a = 2, d – a = 16, donde d = 18, e no segundo a = 3, d – a = 9, donde d = 12. Assim, as duas soluções são (m, n) = (36, 18) e (m, n) = (36, 12).

Page 47: Eureka 2009

Sociedade Brasileira de Matemática

EUREKA! N°29, 2009

47

89 (Balcânica – 2000) Determinar todas as funções :f →¡ ¡ que possuem a propriedade:

2( ( ) ( )) ( ) ,f x f x f y f x y+ = + para todos os números reais x e y. SOLUÇÃO: Fazendo x = 0, obtemos 2( ( )) (0) , ,f f y f y y= + ∀ ∈¡ donde f é uma bijeção, e logo existe a tal que f(a) = 0. Fazendo x = a, obtemos ( ( )) , ,f f y y y= ∀ ∈¡ e

logo (0) 0.f = Fazendo y = 0 obtemos então 2( ( )) ( ) , ,f xf x f x x= ∀ ∈¡ donde 2( ) ( ( ) ),xf x f f x= ou seja, 2( ) ( ),f z zf z z= ∀ ∈¡ (basta fazer x = f(z), donde

( )f x z= ). Aplicando f dos dois lados da equação obtemos 2 2( ( ) ) ( ( ( ) ( ))) ( ) ( ) ( ( ) ) ( ),f f x y f f xf x f y xf x f y f f x f y+ = + = + = + donde,

fazendo 2( ) 0,f x t= ≥ obtemos ( ) ( ) ( ), , 0f y t f y f t y t+ = + ∀ ∈ ≥¡ (lembramos

que f é sobrejetiva, donde 2( )t f x= pode assumir qualquer valor não–negativo).

Assim, 2 2 2 22 ( ) (1) (2 1) (( 1) ) (( 1) ) ( )f x f f x f x x f x f x+ = + = + − = + − = ( 1) ( 1) ( ) (1) ( ) (1), ,x f x xf x xf f x f x= + + − = + + ∀ ∈¡ donde ( ) (1), .f x xf x= ∀ ∈¡ .

Como 2(1) ( (1)) 1,f f f= = isso implica que as únicas soluções são ( ) ,f x x x= ∀ ∈¡ e ( ) ,f x x x= − ∀ ∈¡ (é fácil ver que essas funções satisfazem o

enunciado).

Você sabia… Que 243112609-1 e 237156667-1 são primos? Eles têm 12978189 e 11185272 dígitos respectivamente e são os dois maiores primos conhecidos no momento. Foram descobertos em 23 de agosto de 2008 e 6 de setembro de 2008 por Edson Smith e Hans – Michael Elvenich respectivamente, dois participantes do GIMPS. O GIMPS é um projeto cooperativo na internet que já encontrou 12 primos de Mersenne. Veja www.mersenne.org para mais informações, inclusive como ajudar a achar outros primos de Mersenne.

Page 48: Eureka 2009

Sociedade Brasileira de Matemática

EUREKA! N°29, 2009

48

OLIMPÍADAS AO REDOR DO MUNDO

ü Neste número apresentamos algumas soluções enviadas pelos leitores da nossa seção.

Bruno Holanda

Carlos Augusto David Ribeiro Obs. Nas Eurekas 25 e 27 apareceram, por descuido na edição, problemas na seção Olimpíadas ao redor do mundo com numeração repetida. Nesses casos, procuraremos mencionar o exemplar em que o problema foi publicado, ao nos referirmos a um desses problemas.

üüü 224. (Balcânica Júnior – 2007) Eureka! No. 27 Seja a um real positivo tal que

( )3 6 1 .a a= + Prove que a equação 2 2 6 0x ax a+ + − = não possui solução real. SOLUÇÃO DE JOSÉ DO NASCIMENTO PANTOJA JÚNIOR (FORTALEZA – CE)

3 2 2 66 6 ( 6) 6 6,a a a a a

a= + ⇒ − = ∴ = + e como 0a > concluímos que 2 6,a > daí:

( )2 2 26 6 6 0,a a a a a− = < ⇒ − − < ou seja, o valor de a é tal que:

( ) ( )2 21 1 4 1 6 1 1 4 1 62 3,

2 1 2 1a

− − ⋅ ⋅ − + − ⋅ ⋅ −− = < < =

⋅ ⋅ e como já sabíamos que

6 2,a > > − fica: 6 3.a< <

Substituindo na equação em x, ( )2 6a − por ( )6 / ,a teríamos como raízes

2 64 1

,2

a aax

− ± − ⋅ ⋅= mas estas raízes não são reais, pois 2 24

0,aa

− < devido ao

fato de ( ) ( )3 6 1 6 3 1 24.a a= + < + = 226. (Inglaterra – 2007) Eureka! No. 27 Seja ABC um triângulo acutângulo com AB AC> e 60 .BAC∠ = ° Seja O o circuncentro e H o ortocentro. A reta OH encontra AB em P e AC em Q. Prove que PO = HQ.

Page 49: Eureka 2009

Sociedade Brasileira de Matemática

EUREKA! N°29, 2009

49

SOLUÇÃO DE EMERSON RAMOS BARROSO (FORTALEZA – CE) A

P O Q

C M B

H

Seja M o ponto médio de BC. Sabemos inicialmente que

AH = 2OM (1) e que

QAH OAP∠ = ∠ (2) Temos que

2 120 60 cos60 cos60BOC BAC BOM MOC OM OB AO∠ = ∠ = ° ⇒ ∠ = ∠ = ° ⇒ = ° = ° ⇒2 ,OM AO= e por (1):

AO AH= (3) AHO AOH⇒ ∠ = ∠ e como e AHO AOH∠ ∠ são ângulos externos dos triângulos AQH e AOP, temos

HQA QAH OPA PAO∠ + ∠ = ∠ + ∠ e por (2): AQH APO∠ = ∠

Logo o triâgulo AQP é isósceles, onde .AP AQ= Também por (2) e (3), temos que os triângulos APO e AQH são congruentes (A.L.A.) e assim:

,PO HQ= como queríamos demonstrar. 229. (Bielorússia – 2001) Eureka! No. 27 No losango ABCD, 60A∠ = ° . Os pontos F, H e G estão sobre os segmentos , e AD CD AC de modo que DFGH é um paralelogramo. Prove que FBH é um triângulo equilátero.

Page 50: Eureka 2009

Sociedade Brasileira de Matemática

EUREKA! N°29, 2009

50

SOLUÇÃO DE ANTONIO MARCOS S. ALMEIDA (MANAUS – AM) A

l 30º

60º

l

F b

D

a

l

G

?

?

B

30º 30º

l

b

H

l

b

C Sendo // ,AD GH pois DFGH é paralelogramo, temos µ µCGH HCG≡ e o triângulo HCG é isósceles, logo .HC GH b= = Mas ,FD b= logo os triângulos BCH e

BDF são congruentes, pois BC BD l= = e µ 60 ,F DB = ° já que ABCD é losango.

Portanto BH BF≡ e µ µ .CBH DBF≡

Observe que µ µ µ µ µ µ 60 .CBD CBH H BD H BD DBF H BF≡ + ≡ + ≡ = ° Desta forma conclui-se que o triângulo BFH é equilátero, pois é isósceles e tem o ângulo do vértice de 60 .° 230. (Rússia – 2007) Eureka! No. 27

Sejam a, b, c números reais. Prove que pelo menos uma das três equações 2

2

2

( ) ( ) 0,

( ) ( ) 0

( ) ( ) 0

x a b x b c

x b c x c a

x c a x a b

+ − + − =

+ − + − =

+ − + − =

possui solução real. SOLUÇÃO DE JEAN PIERRE YOUYOUTE (RIO DE JANEIRO - RJ) Analisaremos o caso em que as duas primeiras equações não tem solução real e provaremos que a terceira possui solução real. Os demais casos são análogos.

21

22

( ) 4( ) 0

( ) 4( ) 0

a b b c

b c c a

∆ = − − − <

∆ = − − − <2 2( ) ( ) 4( ).a b b c b a⇒ − + − < −

Page 51: Eureka 2009

Sociedade Brasileira de Matemática

EUREKA! N°29, 2009

51

Como a, b, c são reais, então: 2 20 ( ) ( ) 4( ) 4( ) 0 4( ) 0a b b c b a b a a b< − + − < − ⇒ − > ⇔ − <

23 ( ) 4( )c a a b∆ = − − −

23 0 ( ) 4( ).c a a b∆ > ⇔ − > − Mas 2 2( ) 0 4( ) ( ) 4( ).c a a b c a a b− ≥ > − ⇒ − > −

Portanto, a terceira equação possui solução real. 235. (Olimpíada Checa e Eslovaca – 2007) Eureka! No. 27 Se x, y, z são números reais no intervalo (–1, 1) satisfazendo 1,xy yz zx+ + = mostre que

2 2 2 236 (1 )(1 )(1 ) 1 ( )x y z x y z− − − ≤ + + + SOLUÇÃO DE JEAN PIERRE YOUYOUTE (RIO DE JANEIRO - RJ)

2 2 2 231 ( ) 6 (1 )(1 )(1 )x y z x y z+ + + ≥ − − − ⇔ 2 2 2 2 2 231 2( ) 6 (1 )(1 )(1 )x y z xy yz zx x y z+ + + + + + ≥ − − − ⇔ 2 2 2 2 2 233 6 (1 )(1 )(1 ).x y z x y z+ + + ≥ − − −

Aplicando a desigualdade das médias obtemos: 2 2 23(1 )(1 ) (1 )(1 ) (1 )(1 ) 3 (1 )(1 )(1 )x y y z z x x y z+ − + + − + + − ≥ − − − ⇔

2 2 231 1 1 3 (1 )(1 )(1 )x x xy y z yz z x zx x y z+ − − + + − − + + − − ≥ − − − ⇔ 2 2 233 ( ) 3 (1 )(1 )(1 )xy yz zx x y z− + + ≥ − − − ⇔

2 2 233 1 6 (1 )(1 )(1 ).x y z+ ≥ − − −

Mas 2 2 2 2 2 2 1.x y z xy yz zx x y z+ + ≥ + + ⇔ + + ≥ Logo,

2 2 2 2 2 233 3 1 6 (1 )(1 )(1 )x y z x y z+ + + ≥ + ≥ − − − ⇒

2 2 2 2 2 233 6 (1 )(1 )(1 ).x y z x y z+ + + ≥ − − −

üüü

Page 52: Eureka 2009

Sociedade Brasileira de Matemática

EUREKA! N°29, 2009

52

SOLUÇÕES DE PROBLEMAS PROPOSTOS ! Publicamos aqui algumas das respostas enviadas por nossos leitores. 113. 1 2 3, , ,...a a a formam uma seqüência de inteiros positivos menores que 2007

tais que m n

m n

a aa +

+ é inteiro, para quaisquer inteiros positivos m, n.

Prove que a seqüência (an) é periódica a partir de um certo ponto. SOLUÇÃO DE ZOROASTRO AZAMBUJA NETO (RIO DE JANEIRO – RJ) Seja 2007c < o maior inteiro positivo tal que a seqüência ( )n ra ∈¥ tem infinitos termos iguais a c. Existe 0n ∈¥ tal que 0, .na c n n≤ ∀ > Note que, se ,N n∈¥ são tais que

0n N n N< − < e ,N na a c= = então, como ( )N N n nc a a − += = divide ,N n n N na a a c− −+ = +

segue que | N nc a − e, como ,N na c− ≤ devemos ter .N na c− = Precisaremos agora do seguinte Lema: Para quaisquer inteiros positivos 1 2, ,..., ,kb b b existe 0m ∈¥ tal que qualquer múltiplo de 1 2( , ,..., )kd mdc b b b= que seja maior que 0m pode ser escrito como

1 1 2 2 ... k kr b r b r b+ + + com 1 2, ,..., .kr r r ∈¥ Prova: Vamos usar indução em k. Para k = 1 o resultado é óbvio. Suponhamos que exista °

0m tal que, se °1 2( , ,... ),kd mdc b b b= então todo múltiplo de

°d maior que °0m se escreva como 1 1 ... ,k kr b r b⋅ + + com , .ir i k∈ ∀ ≤¥ Sejam agora

°1 2 1 1( , ,..., , ) ( , )k k kd mdc b b b b mdc d b+ += = e ° °

00 1 .km m b d+= + Sabemos que d pode ser

escrito como °1 ,kdx b y++ com , .x y ∈¢ Se °

1 0( ) ( ) ,kt d d tx b ty m+⋅ = ⋅ + > tomamos

,q s ∈¢ com °ty q d s= ⋅ + e °0 .s d≤ <

Temos então ° ° °1 1 1 1( ) ( ) ( ) .k k k kt d d tx q b b ty q d d tx q b b s+ + + +⋅ = ⋅ + ⋅ + − ⋅ = ⋅ + ⋅ + ⋅

Temos ,s ∈¥ e ° ° °01 1 0 1 0 1( ) ,k k k kd tx q b td b s m b s m b d m+ + + +⋅ + ⋅ = − ⋅ > − ⋅ > − ⋅ =

donde, pela hipótese de indução, existem 1 2, ,..., kr r r ∈¥ com °

1 1 1 2 2( ) ... ,k k kd tx q b rb r b r b+⋅ + ⋅ = + + + e, definindo 1 ,kr s+ = ∈¥ teremos

1 1 2 2 1 1... ,k k k kt d rb r b r b r b+ +⋅ = + + + + o que termina a prova do Lema. Sejam agora | ,nX n a c= ∈ =¥ e d o máximo divisor comum dos elementos de X. Existem 1 2, ,..., kb b b X∈ com 1 2( , ,..., ) ,kmdc b b b d= e existe 0m ∈¥ tal que todo múltiplo de d maior que 0m se escreve como 1 1 2 2 ... k kr b r b r b+ + + com

Page 53: Eureka 2009

Sociedade Brasileira de Matemática

EUREKA! N°29, 2009

53

1 2, ,..., ,kr r r ∈¥ pelo Lema. Afirmamos que, se se 0n n> é múltiplo de d então .na c= De fato, como vimos anteriormente, se 0iN N b n> − > e Na c= então

,iN ba c− = para todo .i k≤

Tomando °0N n m> + tal que ° ,

Na c= temos °N múltiplo de d, donde °

0N n m− >

também é múltiplo de d, e logo se escreve como 1 1 2 2 ... ,k kr b r b r b+ + + com

1 2, ,..., .kr r r ∈¥ Aplicando a observação acima várias vezes, concluímos que

1 1 2 2 ,..., k kn N r b r b r ba a − −= é igual a c, o que mostra nossa afirmação.

Seja agora 0 .n m> Seja t inteiro positivo tal que n + t é múltiplo de d. Temos então que n tc a += divide ,n ta a+ e, analogamente, n d tc a + += divide

,n d ta a+ + donde c divide ( ) ( ) .n d t n t n d na a a a a a+ ++ − + = − Como na e n da + são

inteiros positivos menores ou iguais a c, devemos ter .n d na a+ = Assim, n d na a+ = para todo 0 ,n n> o que prova o resultado.

115. Suponha que ABC é um triângulo com lados inteiros a, b e c com µ 60BCA = ° e ( , ) ( , ) ( , ) 1.mdc a b mdc a c mdc b c= = =

Prove que 1(mod 6)c ≡ . SOLUÇÃO DE EDEL PÉREZ CASTILLO (PINAR DEL RIO – CUBA)

2 2 2 2 22 cos(60 )c a b ab a b ab= + − ° = + − Demonstremos que c é ímpar. Como ( , ) 1mdc a b = então a é ímpar ou b é ímpar. Suponhamos que b é ímpar. Se a é par ou ímpar então c é ímpar. Demonstremos que c não é divisível por 3. Se 23 | 9 |c c⇒ então 2 2 2 2( ) 3c a b ab a b ab= + − = + −

( ) ( ) ( )2 23 | 3 | 9 | 9 | 3 3 |a b a b a b ab ab+ ⇒ + ⇒ + ⇒ ⇒

Então 3 | a ou 3 | ,b o que em qualquer caso contradiz que ( , ) ( , ) ( , ) 1.mdc a b mdc a c mdc b c= = =

Logo c não é divisível por 3. Demonstremos que 1(mod3).c ≡

( ) ( )( )22 2 2 2 2 2 2 24 4 4 4 2 3 2 2 2 2 3 .c a b ab c a ab b a b b c a b c a b b= + − ⇒ = − + = − + ⇒ − + + − =

Os números ( )2 2c a b− + e ( )2 2c a b+ − são ímpares porque b é ímpar.

Page 54: Eureka 2009

Sociedade Brasileira de Matemática

EUREKA! N°29, 2009

54

Seja d um divisor comum de ( )2 2c a b− + e ( )2 2 .c a b+ − Então | 4d c e | 2( )d a b− porém como d é ímpar temos que |d c e | (2 ).d a b− Já demonstramos

que ( ),3 1mcd c = então ( ,3) 1.mcd d =

Da equação ( )22 24 2 3c a b b= − + deduzimos que 2 2 2 2| 3 | | 1d b d b d b d⇒ ⇒ ⇒ =

(porque |d c e ( ), 1mcd b c = ).

Como ( )2 2 ,2 2 1mcd c a b c a b− + + − = e seu produto é 23b temos que: 22 2 3c a b x− + =

22 2c a b y+ − =

Ou também poderia ser: 22 2c a b y− + =

22 2 3c a b x+ − = Os valores de x, y são ímpares porque b é ímpar.

Em qualquer caso temos que ( )2 23 4c x y= +

( )2 22 2 2

30(mod3) (mod3)

4

x yc y c y c y

−− = ⇒ − ≡ ⇒ ≡

Logo 1(mod3),c ≡ porque c não é divisível por 3.

Como 1(mod3)c ≡ então 1(mod6)c ≡ ou 4(mod6),c ≡ porém c é ímpar, logo 1(mod6).c ≡

119. Mostre que não existem inteiros positivos a e b tais que ( )( )36 36a b b a+ + seja uma potência de 2.

SOLUÇÃO DE MARCILIO MIRANDA DE CARVALHO (TERESINA – PI)

Se ( )( )36 36a b b a+ + é uma potência de 2, então ( )36a b+ e ( )36b a+ são

potências de 2 maiores que 36. Daí ( )36a b+ e são ( )36b a+ são pares, logo a e b

são pares. Agora seja S o conjunto de pares (a, b) tais que ( )( )36 36a b b a+ + seja uma potência de 2. Se S não é vazio então S possui um par (a, b) tal que a é mínimo. Como a e b são pares então a = 2c e b = 2d, logo ( )( )4 36 36c d d c+ + é

uma potência de 2, daí ( )( )36 36c d d c+ + é uma potência de 2, onde c < a, absurdo!

Page 55: Eureka 2009

Sociedade Brasileira de Matemática

EUREKA! N°29, 2009

55

120. Sejam a, b, c números reais e soma nS definida como ,n n nnS a b c= + + para

qualquer n inteiro não negativo. Sabe-se que 1 2,S = 2 6S = e 3 14.S = Mostre que 2

1 1 8n n nS S S− +− ⋅ = para todo inteiro n > 1.

SOLUÇÃO DE MARIA CLARA MENDES SILVA (PIRAJUBA – MG) Vamos definir como:

1 a b cΩ = + +

2 ab ac bcΩ = + +

3 abcΩ = Veja que:

( )( ) ( )( )1 1 1 2 2 2n n n n n n n n na b c a b c a b c ab bc ac a b c− − − − − −+ + = + + + + − + + + +

( )3 3 3 .n n nabc a b c− − −+ + +

Isso pode ser verificado diretamente abrindo os produtos. Substituindo com a nossa notação:

1 1 2 2 3 3.n n n nS S S S− − −= Ω − Ω + Ω Assim podemos definir nS por recorrência. Falta descobrir 1 2,Ω Ω e 3Ω . Inicialmente note que 1 2.S a b c= + + = Assim 1 2.Ω = Elevando a + b + c ao quadrado:

( ) ( )2 2 2 22 2 24 2 2 6 2 .a b c a b c ab bc ac S= + + = + + + + + = + Ω = + Ω

Resolvendo achamos 2 1.Ω = − Finalmente:

( )( )3 3 3 2 2 214 3a b c a b c a b c ab ac bc abc= + + = + + + + − − − + =

3 3 32 (6 1) 3 14 3 0.= ⋅ + + Ω = + Ω ⇒ Ω = Na equação de recorrência que tínhamos obtido anteriormente:

1 22 .n n nS S S− −= + Agora usaremos indução em n: Para n = 2, o que queremos demonstrar é claramente verdadeiro, uma vez que

2 22 1 3 6 28 8.S S S− = − =

Suponhamos que essa propriedade seja verdadeira para todo natural menor que um certo n, queremos provar que também vale para n.

( )2 2 2 21 1 1 1 1 12 2n n n n n n n n n n nS S S S S S S S S S S− + − − − −− ⋅ = − ⋅ + = − − =

Page 56: Eureka 2009

Sociedade Brasileira de Matemática

EUREKA! N°29, 2009

56

( )21 12 .n n n nS S S S− −= − − −

Como 1 22 ,n n nS S S− −+ =

1 22 .n n nS S S− −− = − Substituindo:

2 21 1 1 2.n n n n n nS S S S S S− + − −− ⋅ = − + ⋅

Essa expressão tem o mesmo módulo do que a obtida se trocarmos todos seus sinais, ou seja 2

1 2 .n n nS S S− −− ⋅ Mas pela hipótese de indução o módulo desta é 8.

Assim, o módulo de 21 1n n nS S S− +− é 8. Por indução provamos que essa sentença é

válida para todo n inteiro positivo. 121. Na figura abaixo o lado do quadrado vale 4, obter o valor da altura h para que a área da região 1 seja igual a área da região 2.

4

4

h

4

4

1

2

3

SOLUÇÃO DE BRUNO SALGUEIRO FANEGO (VIVEIRO – ESPANHA) A união das regiões 1 e 3 é um quadrante de um semicírculo de raio 4 (de área

244

π ⋅ ) e a união das regiões 2 e 3, um retângulo de lados 4 e h (de área 4 ⋅ h),

donde, sendo iA a área da região i, 1 3,i≤ ≤ teremos 2

1 2 1 3 2 34

4 .4

A A A A A A h hπ

π⋅

= ⇔ + = + ⇔ = ⋅ ⇔ =

122. Dado um triângulo ABC tal que AB AC a b= = + e BC a= , traça-se uma ceviana partindo de B determinando em AC um ponto D tal que DA a= e DC b= . Sabendo que µ 10ABD = ° , determine os ângulos internos desse triângulo.

Page 57: Eureka 2009

Sociedade Brasileira de Matemática

EUREKA! N°29, 2009

57

SOLUÇÃO DE CARLOS ALBERTO DA SILVA VICTOR (NILÓPOLIS – RJ)

A

C B

D

?

10°

AD BC aAB AC

= = =

Tomando BAC θ∠ = e, usando que AD = BC e a lei dos senos nos triângulos ABD

e BDC, encontramos: ( )10 2 10 .2

sen sen senθ

θ ° = ⋅ + °

Observe que 20θ = ° é solução. Vamos mostrar que ela é única para o triângulo em questão. Não é difícil de verificar que 80 ,θ < ° pois não poderíamos ter AD = BC para

80θ ≥ ° (BC será o maior lado); conseqüentemente, 10 90 ,θ + < ° e 2

senθ

e

( 10 )sen θ + ° serão crescentes no primeiro quadrante.

I) Suponha que ( )20 10 2 10 2 10 30 102

sen sen sen sen sen senθ

θ θ > ° ⇒ ° = ⋅ + ° > °⋅ ° = °

(absurdo).

II) Suponha que ( )20 10 2 10 2 10 30 102

sen sen sen sen sen senθ

θ θ < ° ⇒ ° = ⋅ + ° < °⋅ ° = °

(absurdo). Conclusão: 20θ = ° é a única solução. 127. Determine todos os inteiros positivos k tais que existem inteiros positivos x, y,

z com 2 2 2

.x y z

kxyz

+ +=

SOLUÇÃO DE EDEL PÉREZ CASTILLO (PINAR DEL RIO – CUBA) Se 1x y z= = = obtemos k = 3; se x = y = z = 3 obtemos k = 1. Falta provar que estes são os únicos valores que pode tomar k. Provemos que k deve ser ímpar.

Page 58: Eureka 2009

Sociedade Brasileira de Matemática

EUREKA! N°29, 2009

58

Seja 2 2 2

3( , , ) ( *) :x y z

S x y z kxyz

+ += ∈ =¥ onde k é par

Provemos que este conjunto é vazio. Se não é vazio existem 0 0 0, ,x y z S∈ tais que 0 0 0x y z+ + é mínimo.

Temos 2 2 20 0 0 0 0 0x y z kx y z+ + = . Como o membro direito é par , há dois casos:

(1) Um dos números é par e os outros dois números são ímpares. (2) Os três são pares. No caso (1), se 0x é par, 0 0,y z ímpares, então 2 2 2

0 0 0 2(mod 4)x y z+ + ≡ e no outro membro temos que 0 0 0 0(mod 4)kx y z ≡ porque k e 0x são pares. Contradição. No caso (2) 0 1 0 1 0 12 , 2 , 2x x y y z z= = = facilmente comprovamos que:

2 2 21 1 1

1 1 12

x y zk

x y z+ +

= , logo ( )1 1 1, , .x y z S∈

( )1 1 1 0 0 0 0 0 02 .x y z x y z x y z+ + = + + < + + Isto é absurdo porque 0 0 0x y z+ + é mínimo. Logo o conjunto S é vazio. Provemos que 3,k ≤ o que resolve o problema.

Seja 2 2 2

3( , , ) ( *) : 4x y z

C x y z kxyz

+ += ∈ = ≥¥

Provemos que este conjunto em C é vazio. Se não é vazio existem 0 0 0, ,x y z tais que 0 0 0x y z+ + é mínimo. Podemos supor que 0 0 0.x y z≤ ≤ Então:

0 0 0

0 0 0 0 0 04

x y zy z x z x y

+ + ≥

Mas 0

0 01

xy z

≤ e 0

0 01,

yx z

≤ donde temos que 0

0 01 1 4,

zx y

+ + ≥ o que implica que

0 0 02 .z x y≥

Consideremos a equação 2 2 20 0 0 0 0t kx y t x y− + + = que tem a 0z como uma das suas

soluções. Seja 1z a outra solução da equação; então temos: (1) 0 1 0 0z z kx y+ =

(2) 2 20 1 0 0z z x y= +

De (1) deduzimos que 1z é inteiro, de (2) deduzimos que 1z é positivo. Logo

0 0 1( , , )x y z C∈ mas 0 0 0 0 0 1x y z x y z+ + ≤ + + porque 0 0 0x y z+ + é mínimo.

Page 59: Eureka 2009

Sociedade Brasileira de Matemática

EUREKA! N°29, 2009

59

Então 0 0 0 12 ,x y z z≤ ≤ e substituindo em (2) obtemos 2 2 2 2 20 0 0 0 04 2z y x y y≤ + ≤ donde se

obtém que 204 2x ≤ , o que é absurdo. Portanto o conjunto C é vazio.

128. Barango Joe era um sapo de mútiplos talentos que habitava a Terra das Chances Diminutas, localizada no alto de uma montanha. Após sua maioridade, Barango Joe decidiu tentar a vida no Reino das Grandes Oportunidades, localizado no cume da montanha vizinha. Para isso, ele atravessaria a extensa ponte de madeira por cima do Desfiladeiro da Morte. Entretanto, a ponte era guardada pela Esfinge Vegas, exímia jogadora que sempre desafiava os viajantes para algum jogo. O viajante vitorioso tinha a passagem franqueada; e o perdedor era lançado ao abismo. Assim chegando à cabeceira da ponte, Barango Joe foi desafiado a uma partida de “Pachang” jogo que lembra o “Black Jack” ou “Vinte e um”, mas é jogado por 2 oponentes da seguinte maneira: Os jogadores, designados por “banca” e “apostador”, utilizam um dado gerador de números aleatórios reais uniformemente distribuídos no intervalo [0,1]. Inicialmente, a banca sorteia um número X. Se não estiver satisfeita com o número obtido, pode descartá-lo e então sortear um novo número. Este procedimento pode ser executado 2 vezes, Isto é, pode haver até 3 sorteios na definição do número X da banca. Então, o apostador sorteia quantos números forem necessários até que a soma de seus números ultrapasse o número X da banca. Neste momento, se esta soma for inferior a 1, o apostador ganha; caso contrário, perde. Ou seja, para ganhar, o apostador precisa “chegar mais próximo” de 1 que a banca, sem no entanto “estourar o limite” de 1. Após explicar as regras do Pachang, a Esfinge Vegas deu uma opção ao sapo: - Você prefere ser a banca ou o apostador? O que o Barango Joe deveria responder? Obs. Utilize lápis, papel, e uma calculadora científica simples. SOLUÇÃO DE RAFAEL TUPYNAMBÁ DUTRA (BELO HORIZONTE – MG) Primeiramente, supondo que X já tenha sido escolhido, vamos determinar a função

)(Xf que dá a probabilidade de o apostador ganhar. Seja )(xpn a função densidade de probabilidade para a soma dos números do apostador (ou seja, após n números sorteados pelo apostador, a probabilidade de a soma dos n – 1 ser menor ou igual a X e a soma desses números estar entre a e b é dada por

∫b

an dxxp )( ). Definimos )(xqn igual a )(xpn para ],0[ Xx ∈ e )(xqn igual a 0

Page 60: Eureka 2009

Sociedade Brasileira de Matemática

EUREKA! N°29, 2009

60

caso contrário. Seja também ∫=1

)(X

nn dxxpg a probabilidade de o apostador

ganhar após n números sorteados. Temos 1( ) 1, [0,1]p x x= ∀ ∈ .

Lema: )!1(

)()(1

−==

nx

xpxqn

nn para ],0[ Xx ∈ e )!1(

)(1

−=

nX

xpn

n para

]1,(Xx ∈ . Prova por indução: o caso inicial 1=n é trivial. Hipótese de indução:

)!1()(

1

−=

nx

xqn

n para ],0[ Xx ∈ . Caso o jogo continue, a densidade de

probabilidade da soma dos n primeiros números é dada por )(xqn (afinal, para o

jogo continuar, essa soma deve estar no intervalo ],0[ X ). E a densidade de

probabilidade do )1( +n -ésimo número é igual a ]1,0[1)(1 ∈∀= xxp . Assim,

somando, obtemos ∫−

+ =x

xnn dttqxp

11 )()( . Dessa forma, para ],0[ Xx ∈ , temos

!)!1()()(

0

1

11 n

xdt

nt

dttqxpnx nx

xnn =

−== ∫∫

−+ . E para ]1,(Xx ∈ ,

!)!1()()(

0

1

11 n

Xdt

nt

dttqxpnX nx

xnn =

−== ∫∫

−+ , c.q.d.

Pelo lema, temos )!1(

)1()(11

−−==

∫ nX

Xdxxpgn

Xnn e a probabilidade de o

apostador ganhar é ∑ ∑∞

=

=

−=−

−==1 1

1

)1()!1(

)1()(n n

Xn

n eXnX

XgXf .

Agora imagine que a banca tenha k sorteios disponíveis para determinar X . Seja kP a probabilidade de o apostador ganhar nesse caso. Queremos calcular 3P .

Vamos calcular kP recursivamente. Primeira situação: imagine que a banca tem apenas 1 sorteio disponível para determinar X . Sabendo que, dado X , a probabilidade de o apostador ganhar é

XeXXf )1()( −= , descobrimos que 71828,02)(1

01 ≅−== ∫ edXXfP .

Page 61: Eureka 2009

Sociedade Brasileira de Matemática

EUREKA! N°29, 2009

61

Segunda situação: imagine agora que a banca tem até 2 sorteios para determinar X . Se, no primeiro sorteio, ela obteve um número x , ela pode ficar com esse

número ou descartá-lo. A probabilidade de o apostador ganhar será xexxf )1()( −= no 1º caso e será 71828,01 ≅P no 2º caso. Assim, a banca deve

descartar x somente se 1)1( Pex x >− . Seja a a raiz positiva da equação

transcendente 1)1( Pea a =− . Temos 60954,0≅a . Dessa forma, a banca deve descartar o número x se ax < e mantê-lo se ax > . Se a banca usar essa estratégia, a probabilidade de o apostador vencer será

59823,0)2()( 1

1

12 ≅−−+=+= ∫ a

a

eaeaPdXXfaPP .

Terceira situação: agora considere o problema original (a banca tem até 3 sorteios para determinar X ). Se, no primeiro sorteio, ela obteve o número x , ela pode mantê-lo ou descartá-lo. A probabilidade de o apostador ganhar será

xexxf )1()( −= no 1º caso e será 59823,02 ≅P no 2º caso. Assim, a banca deve

descartar x somente se 2)1( Pex x >− . Seja b a raiz positiva da equação

transcendente 2)1( Peb b =− . Temos 70416,0≅b . Dessa forma, a banca deve descartar o número x se bx < e mantê-lo se bx > . Se a banca usar essa estratégia, a probabilidade de o apostador vencer será

21

51915,0)2()( 2

1

23 >≅−−+=+= ∫ b

b

ebebPdXXfbPP . Assim, o apostador tem

probabilidade maior que 21 de ganhar, mesmo quando a banca usa a melhor

estratégia possível. Barango Joe deve responder que prefere ser o apostador. Agradecemos o envio de soluções e a colaboração de: Alexandre Salim Saud de Oliveira Niterói – RJ Alixanzito R. S. Costa Fortaleza – CE André Felipe M. da Silva Rio de Janeiro – RJ Doraci Gabriel da Rosa Fartura – SP Evandro Makiyama de Melo São Paulo – SP Flávio Antonio Alves Amparo – SP Glauber Moreno Barbosa Rio de Janeiro – RJ Kellem Corrêa Santos Rio de Janeiro – RJ Larissa Brito Sousa Fortaleza – CE Marcel Menzes de Andrade Prado Brasília – DF Oswaldo Mello Sponquiado São Paulo – SP Renato Carneiro de Souza Belo Horizonte – MG Samuel Liló Abdalla Sorocaba – SP Wallace Alves Martins Rio de Janeiro – RJ Continuamos aguardando soluções para os problemas 123, 124, 125, 126 e 129.

Page 62: Eureka 2009

Sociedade Brasileira de Matemática

EUREKA! N°29, 2009

62

PROBLEMAS PROPOSTOS * Convidamos o leitor a enviar soluções dos problemas propostos e sugestões de novos problemas para próximos números. 130) Suponha que , ,a b c ∈¡ e a equação ( )2 ( ) 0x a b c x ab ac bc− + + + + + = não tem raízes reais. Prove que a, b e c têm todos o mesmo sinal e existe um triângulo

de lados ,a b e .c

131) a) Considere o seguinte jogo: no início um jogador A entrega um número

2k ≥ ao jogador B . Quando A entrega um número 2m ≥ a B, B pode devolver m – 1 ou 1m + a A. Quando A recebe um número 2n ≥ deve, se n for ímpar

devolver 3n a B; se n for par mas não múltiplo de 4, pode devolver 2n

ou 3n a B, e,

se n for múltiplo de 4, pode devolver ,4 2n n

ou 3n a B. Qualquer jogador ganha o

jogo se devolver 1 ao adversário. Caso algum jogador devolva ao adversário um número maior que 1000k, o jogo empata. Determine, para cada valor de 2k ≥ , se algum dos jogadores tem estratégia vencedora, e, nesses casos, qual deles. b) Resolva o item anterior supondo que A, ao receber um número 2,n ≥ deve

devolver 3n a B se n for ímpar, deve devolver 2n

a B se n for par mas não múltiplo

de 4 e deve devolver 4n

a B se n for múltiplo de 4.

132) a) Considere uma família ℑ de 2000 círculos de raio 1 no plano tal que dois círculos de ℑ nunca são tangentes e cada círculo de ℑ intersecta pelo menos dois outros círculos de ℑ . Determine o número mínimo possível de pontos do plano que pertencem a pelo menos dois círculos de ℑ .

Problema 130 proposto por Wílson Carlos da Silva Ramos (Belém – PA), problema 131 proposto por Benedito Tadeu Vasconcelos Freire (Natal – RN), problema 132 proposto por Juan Manuel Conde Calero (Alicante – Espanha).

Page 63: Eureka 2009

Sociedade Brasileira de Matemática

EUREKA! N°29, 2009

63

AGENDA OLÍMPICA

XXXI OLIMPÍADA BRASILEIRA DE MATEMÁTICA

NÍVEIS 1, 2 e 3 Primeira Fase – Sábado, 06 de junho de 2009

Segunda Fase – Sábado, 12 de setembro de 2009 Terceira Fase – Sábado, 17 de outubro de 2009 (níveis 1, 2 e 3)

Domingo, 18 de outubro de 2009 (níveis 2 e 3 - segundo dia de prova).

NÍVEL UNIVERSITÁRIO Primeira Fase – Sábado, 12 de setembro de 2009

Segunda Fase – Sábado, 17 e Domingo, 18 de outubro de 2008

XV OLIMPÍADA DE MAIO 09 de maio de 2009

XX OLIMPÍADA DE MATEMÁTICA DO CONE SUL 14 a 20 de abril de 2009 Mar del Plata – Argentina

L OLIMPÍADA INTERNACIONAL DE MATEMÁTICA 10 a 22 de julho de 2009

Bremen – Alemanha

XVI OLIMPÍADA INTERNACIONAL DE MATEMÁTICA UNIVERSITÁRIA 25 a 30 de julho de 2009

Budapeste, Hungria

XXIV OLIMPÍADA IBEROAMERICANA DE MATEMÁTICA Setembro de 2009 Mérida – México

XII OLIMPÍADA IBEROAMERICANA DE MATEMÁTICA UNIVERSITÁRIA

Page 64: Eureka 2009

Sociedade Brasileira de Matemática

EUREKA! N°29, 2009

64

COORDENADORES REGIONAIS

Alberto Hassen Raad (UFJF) Juiz de Fora – MG Américo López Gálvez (USP) Ribeirão Preto – SP Amarísio da Silva Araújo (UFV) Viçosa – MG Andreia Goldani FACOS Osório – RS Antonio Carlos Nogueira (UFU) Uberlândia – MG Benedito Tadeu Vasconcelos Freire (UFRN) Natal – RN Carlos Alexandre Ribeiro Martins (Univ. Tec. Fed. de Paraná) Pato Branco – PR Carmen Vieira Mathias (UNIFRA) Santa María – RS Claus Haetinger (UNIVATES) Lajeado – RS Cleonor Crescêncio das Neves (EDETEC) Manaus – AM Cláudio de Lima Vidal (UNESP) S.J. do Rio Preto – SP Denice Fontana Nisxota Menegais (UNIPAMPA) Bagé – RS Edson Roberto Abe (Colégio Objetivo de Campinas) Campinas – SP Eduardo Tengan (USP) São Carlos – SP Élio Mega (Faculdade Etapa) São Paulo – SP Eudes Antonio da Costa (Univ. Federal do Tocantins) Arraias – TO Fábio Brochero Martínez (UFMG) Belo Horizonte – MG Florêncio Ferreira Guimarães Filho (UFES) Vitória – ES Francinildo Nobre Ferreira (UFSJ) São João del Rei – MG Genildo Alves Marinho (Centro Educacional Leonardo Da Vinci) Taguatingua – DF Ivanilde Fernandes Saad (UC. Dom Bosco) Campo Grande – MS Jacqueline Rojas Arancibia (UFPB)) João Pessoa – PB Janice T. Reichert (UNOCHAPECÓ) Chapecó – SC João Benício de Melo Neto (UFPI) Teresina – PI João Francisco Melo Libonati (Grupo Educacional Ideal) Belém – PA Jose de Arimatéia Fernandes (UFPB) Campina Grande – PB José Luiz Rosas Pinho (UFSC) Florianópolis – SC José Vieira Alves (UFPB) Campina Grande – PB José William Costa (Instituto Pueri Domus) Santo André – SP Krerley Oliveira (UFAL) Maceió – AL Licio Hernandes Bezerra (UFSC) Florianópolis – SC Luciano G. Monteiro de Castro (Sistema Elite de Ensino) Rio de Janeiro – RJ Luzinalva Miranda de Amorim (UFBA) Salvador – BA Mário Rocha Retamoso (UFRG) Rio Grande – RS Marcelo Rufino de Oliveira (Grupo Educacional Ideal) Belém – PA Marcelo Mendes (Colégio Farias Brito, Pré-vestibular) Fortaleza – CE Newman Simões (Cursinho CLQ Objetivo) Piracicaba – SP Nivaldo Costa Muniz (UFMA) São Luis – MA Osnel Broche Cristo (UFLA) Lavras – MG Osvaldo Germano do Rocio (U. Estadual de Maringá) Maringá – PR Raul Cintra de Negreiros Ribeiro (Colégio Anglo) Atibaia – SP Ronaldo Alves Garcia (UFGO) Goiânia – GO Rogério da Silva Ignácio (Col. Aplic. da UFPE) Recife – PE Reginaldo de Lima Pereira (Escola Técnica Federal de Roraima) Boa Vista – RR Reinaldo Gen Ichiro Arakaki (UNIFESP) SJ dos Campos – SP Ricardo Amorim (Centro Educacional Logos) Nova Iguaçu – RJ Sérgio Cláudio Ramos (IM-UFRGS) Porto Alegre – RS Seme Gebara Neto (UFMG) Belo Horizonte – MG Tadeu Ferreira Gomes (UEBA) Juazeiro – BA Tomás Menéndez Rodrigues (U. Federal de Rondônia) Porto Velho – RO Valdenberg Araújo da Silva (U. Federal de Sergipe) São Cristovão – SE Vânia Cristina Silva Rodrigues (U. Metodista de SP) S.B. do Campo – SP Wagner Pereira Lopes (CEFET – GO) Jataí – GO

Page 65: Eureka 2009

CONTEÚDO

XXX OLIMPÍADA BRASILEIRA DE MATEMÁTICA Problemas e Soluções da Primeira Fase

2

XXX OLIMPÍADA BRASILEIRA DE MATEMÁTICA Problemas e Soluções da Segunda Fase

16 XXX OLIMPÍADA BRASILEIRA DE MATEMÁTICA Problemas e Soluções da Terceira Fase

33 XXX OLIMPÍADA BRASILEIRA DE MATEMÁTICA Problemas e Soluções da Primeira Fase Nível Universitário

51 XXX OLIMPÍADA BRASILEIRA DE MATEMÁTICA Problemas e Soluções da Segunda Fase Nível Universitário

57 XXX OLIMPÍADA BRASILEIRA DE MATEMÁTICA Premiados

66 AGENDA OLÍMPICA

70

COORDENADORES REGIONAIS

71

Page 66: Eureka 2009

Sociedade Brasileira de Matemática

EUREKA! N°30, 2009

2

XXX OLIMPÍADA BRASILEIRA DE MATEMÁTICA Problemas e Soluções da Primeira Fase

PROBLEMAS – NÍVEL 1 01) Com segmentos de 1 cm de comprimento podemos formar triângulos. Por exemplo, com nove desses segmentos podemos formar um triângulo eqüilátero de lado 3 cm. Com qual número de segmentos a seguir é impossível formar um triângulo? A) 4 B) 5 C) 6 D) 7 E) 8 02) Esmeralda compra cinco latas de azeite a quatro reais e setenta centavos a lata, cinco latas de leite em pó a três reais e doze centavos cada e três caixas de iogurte com seis iogurtes cada caixa ao preço de oitenta centavos por iogurte. Paga com uma nota de cinqüenta reais e quer saber quanto irá receber de troco. Qual das expressões aritméticas a seguir representa a solução para este problema? A) 50 5 (4,70 3,12) 18 0,80− × + + × B) 5 4,70 5 3,12 3 6 0,80 50× + × + × × − C) [ ]5 (4,70 3,12) 3 6 0,80 50− × + + × × +

D) [ ]50 5 (4,70 3,12) 3 6 0,80− × + + × + E) [ ]50 5 (4,70 3,12) 6 0,80− × + + ×

03) Uma pesquisa foi feita entre pessoas de ambos os sexos, em igual número, com a seguinte pergunta: Entre as cores azul, vermelho e amarelo, qual é a cor que você prefere? Cada pessoa apresentou a sua preferência por uma, e só uma, dessas cores. E o resultado da pesquisa aparece nos gráficos abaixo:

Page 67: Eureka 2009

Sociedade Brasileira de Matemática

EUREKA! N°30, 2009

3

Podemos concluir que, em relação ao total de pessoas pesquisadas, a ordem de preferência das cores é: A) I, II, III B) I, III, II C) II, I, III D) II, III, I E) III, II, I 04) O quociente e o resto na divisão de 26097 por 25 são, respectivamente: A) 1043 e 22 B) 1044 e 3 C) 143 e 22 D) 1044 e 22 E) 144 e 3 05) Numa reunião da comunidade do bairro, cada uma das 125 pessoas presentes recebeu um número diferente, a partir do número 1 até o 125. Em dado momento, foi feita uma lista das pessoas com número par e das pessoas com número múltiplo de 3, que deveriam participar de um projeto. Algumas pessoas reclamaram, dizendo que o seu nome aparecia duas vezes na lista. Quantas pessoas apareceram duas vezes na lista? A) 2 B) 6 C) 20 D) 41 E) 62 06) Sobre uma mesa retangular de uma sala foram colocados quatro sólidos, mostrados no desenho. Uma câmera no teto da sala, bem acima da mesa, fotografou o conjunto. Qual dos esboços a seguir representa melhor essa fotografia?

Page 68: Eureka 2009

Sociedade Brasileira de Matemática

EUREKA! N°30, 2009

4

07) Uma classe tem 22 alunos e 18 alunas. Durante as férias, 60% de todos os alunos dessa classe foram prestar trabalho comunitário. No mínimo, quantas alunas participaram desse trabalho? A) 1 B) 2 C) 4 D) 6 E) 8 08) Uma urna contém 2008 cartões. Cada cartão recebeu um número diferente, a partir do número 1 até o 2008. Retiram-se dois cartões ao acaso e somam-se os números dos cartões. Quantos números ímpares diferentes podem ser obtidos dessa maneira? A) 1004 B) 1005 C) 2007 D) 2008 E) 4016 09) Juntando quatro trapézios iguais de bases 30 cm e 50 cm, como o da figura ao lado, podemos formar um quadrado de área 2500 cm2, com um “buraco” quadrado no meio. Qual é a área de cada trapézio, em cm2?

50 cm 45o 45o

30cm

A) 200 B) 250 C) 300 D) 350 E) 400 10) Quantos números pares de três algarismos têm dois algarismos ímpares? A) 20 B) 48 C) 100 D) 125 E) 225

11) Sabe-se que 92

do conteúdo de uma garrafa enchem 65

de um copo. Para

encher 15 copos iguais a esse, quantas garrafas deverão ser usadas? A) 2 B) 3 C) 4 D) 5 E) 6

12) Quantos quadrados têm como vértices os pontos do reticulado ao lado?

A) 6 B) 7 C) 8 D) 9

E) 10

Page 69: Eureka 2009

Sociedade Brasileira de Matemática

EUREKA! N°30, 2009

5

13) A primeira fase da OBM se realiza no dia 14 de junho, um sábado do ano bissexto 2008. Daqui a quantos anos o dia 14 de junho será novamente no sábado? A) 4 B) 5 C) 6 D) 7 E) 8 14) No desenho temos AE = BE = CE = CD. Além disso, α e β são medidas de ângulos.

Qual é o valor da razão αβ

?

A) 53

B) 54

C) 1

D) 45

E) 35

15) Na multiplicação ao lado, alguns algarismos, não necessariamente iguais, foram substituídos pelo sinal *. Qual é a soma dos valores desses algarismos?

A) 17 B) 27 C) 37 D) 47

E) 57

16) Três amigos moram na mesma rua: um médico, um engenheiro e um professor. Seus nomes são: Arnaldo (A), Bernaldo (B) e Cernaldo (C). O médico é filho único e o mais novo dos três amigos. Cernaldo é mais velho que o engenheiro e é casado com a irmã de Arnaldo. Os nomes do médico, do engenheiro e do professor, nessa ordem, são: A) A, B, C B) C, A, B C) B, A, C D) B, C, A E) A, C, B

Page 70: Eureka 2009

Sociedade Brasileira de Matemática

EUREKA! N°30, 2009

6

17) Dois cartões iguais têm a forma de um triângulo retângulo de lados 5 cm, 12 cm e 13 cm. Esmeralda juntou os dois cartões sobre uma folha de papel e, contornando as beiradas com um lápis, obteve uma figura como a ao lado, que está fora de escala. Qual é o perímetro dessa figura?

A) 28 cm B) 35 cm C) 42 cm D) 43 cm E) 60 cm 18) Qual é o maior número de algarismos que devem ser apagados do número de 1000 algarismos 20082008…2008, de modo que a soma dos algarismos restantes seja 2008? A) 130 B) 260 C) 510 D) 746 E) 1020 19) Soninha tem muitos cartões, todos com o mesmo desenho em uma das faces. Ela vai usar cinco cores diferentes (verde, amarelo, azul, vermelho e laranja) para pintar cada uma das cinco partes do desenho, cada parte com uma cor diferente, de modo que não haja dois cartões pintados da mesma forma. Na figura abaixo, por exemplo, os cartões são iguais, pois um deles pode ser girado para se obter o outro. Quantos cartões diferentes Soninha conseguirá produzir?

A) 16 B) 25 C) 30 D) 60 E) 120

Page 71: Eureka 2009

Sociedade Brasileira de Matemática

EUREKA! N°30, 2009

7

20) Três carros com velocidades constantes cada um, na mesma estrada, passam no mesmo momento por Brasilópolis. Ao viajar 100 quilômetros, o carro A passa por Americanópolis, 20 quilômetros à frente do carro B e 50 quilômetros à frente do carro C. Quando o carro B passar por Americanópolis, quantos quilômetros estará à frente do carro C? A) 20 B) 25,5 C) 30 D) 35 E) 37,5

PROBLEMAS – NÍVEL 2 01) Veja o problema No. 14 do Nível 1. 02) Quantos dos números abaixo são maiores que 10?

113 , 74 , 55 , 36 , 27 A) 1 B) 2 C) 3 D) 4 E) 5

03) 1212 é igual a:

A) 66 B) 3212 C) 612 3.2 D) 126 E) 12

12

04) Uma grande empresa possui 84 funcionários e sabe-se que cada funcionário fala pelo menos uma das línguas entre Português e Inglês. Além disso, 20% dos que falam Português também falam Inglês e 80% dos que falam Inglês também falam Português. Quantos funcionários falam as duas línguas? A) 12 B) 14 C) 15 D) 16 E) 18

05) Edmilson, Carlos e Eduardo ganharam um total de R$150,00 lavando carros. Eles ganharam quantidades diferentes de dinheiro. Como eles são muito amigos decidiram dividir o dinheiro ganho em partes iguais. Para isto, Edmilson deu metade do que ganhou para dividir em partes iguais entre Carlos e Eduardo, porém, Carlos tinha muito dinheiro e, portanto, deu R$ 10,00 a cada um dos outros dois. Finalmente, para que cada um tivesse a mesma quantidade de dinheiro, Eduardo deu R$ 2,00 a Edmilson. Quanto Eduardo ganhou antes da divisão? A) R$ 76,00 B) R$ 51,00 C) R$ 23,00 D) R$ 50,00 E) R$ 100,00

06) Nove números são escritos em ordem crescente. O número do meio é a média aritmética dos nove números. A média aritmética dos 5 maiores é 68 e a média aritmética dos 5 menores é 44. A soma de todos os números é: A) 560 B) 504 C) 112 D) 56 E) 70

Page 72: Eureka 2009

Sociedade Brasileira de Matemática

EUREKA! N°30, 2009

8

07) Veja o problema No. 12 do Nível 1. 08) Veja o problema No. 13 do Nível 1. 09) Os algarismos a , b e c são tais que os números de dois algarismos aa , bc e

cb são números primos e 2

aacbbcaa =++ . Se cb < , então bc é igual a: A) 19 B) 17 C) 37 D) 29 E) 59

10) Cinco inteiros positivos edcba ,,,, maiores que um satisfazem as seguintes condições:

275)(243)(203)(155)(128)(

=+++=+++=+++=+++=+++

dcbaeecbadedbacedcabedcba

Quanto vale a soma edcba ++++ ? A) 9 B) 16 C) 25 D) 36 E) 49 11) Em um triângulo ABC foi traçada a altura AH. Sejam M e N pontos sobre os lados AB e AC, respectivamente, tais que HM é perpendicular a AB e HN é perpendicular a AC. Achar MN, sabendo que o perímetro do triângulo órtico do triângulo ABC é igual a 10. Observação: o triângulo órtico de um triângulo é aquele cujos vértices são as interseções das alturas do triângulo com os respectivos lados. Pode-se demonstrar que o incentro (encontro das bissetrizes) do triângulo órtico é sempre igual ao ortocentro (encontro das alturas) do triângulo original. A) 5 B) 6 C) 7 D) 8 E) 9 12) Quantos números inteiros positivos menores que 500 têm exatamente 15 divisores inteiros positivos? A) 0 B) 1 C) 2 D) 3 E) 4

13) Seja )(nP a soma dos algarismos pares do número n . Por exemplo,

.642)1234( =+=P Qual o valor de ?)100(...)3()2()1( PPPP ++++ A) 200 B) 360 C) 400 D) 900 E) 2250

Page 73: Eureka 2009

Sociedade Brasileira de Matemática

EUREKA! N°30, 2009

9

14) De quantas maneiras podemos dividir R$ 10,00 em moedas de 10 centavos e de 25 centavos, se pelo menos uma moeda de cada valor tem que ser usada? A) 15 B) 16 C) 17 D) 18 E) 19

15) Sejam dcba ,,, números inteiros tais que ba 2< , cb 3< , dc 4< . Se

40<d , o maior valor possível de a será: A) 960 B) 959 C) 951 D) 934 E) 927 16) A figura abaixo é um exemplo de um quadrado mágico de ordem 4. A soma dos 4 números em cada linha, coluna e diagonal é 34. Então dizemos que a soma mágica deste quadrado mágico é 34. Suponha que exista um quadrado mágico de ordem 7, formado pelos números inteiros de 1 a 49. Determine sua soma mágica.

16 3 2 13

5 10 11 8

9 6 7 12

4 15 14 1

A) 175 B) 2450 C) 1225 D) 190 E) 100 17) Observe que:

.85841243,131243

,543

22222

2222

222

=+++

=++

=+

Qual o menor valor possível da soma yx + com yx, inteiros positivos tais que ?841243 222222 yx =++++

A) 289 B) 250 C) 425 D) 795 E) 103 18) Um número de três algarismos é 629 vezes menor que a soma de todos os outros números de três algarismos. Este número é: A) 450 B) 785 C) 630 D) 471 E) 525

19) Veja o problema No. 19 do Nível 1.

Page 74: Eureka 2009

Sociedade Brasileira de Matemática

EUREKA! N°30, 2009

10

20) Em um triângulo ABC, ∠ A = 20o e ∠ B = 110o. Se I é o incentro (centro da circunferência inscrita) e O o circuncentro (centro da circunferência circunscrita) do triângulo ABC, qual a medida do ângulo IAO∠ ? A) 20o B) 25o C) 30o D) 40o E) 35o 21) Veja o problema No. 7 do Nível 1.

22) Na figura abaixo os pontos A, B, C são colineares, assim como os pontos D, E, F. As duas retas ABC e DEF são paralelas.

A B C

D E F

A A A1 2 3

Sendo A1, A2 e A3 as áreas das regiões destacadas na figura, podemos afirmar que: A) A2 = 2A1 = 2A3 B) A2 = A1 + A3 C) A2 > A1 + A3 D) A2 < A1 + A3 E) A2

2 = A1.A3 23) O grupo A da última Copa do Mundo de futebol terminou com os seguintes resultados:

Equipe Número de Pontos Áustria 7 Brasil 5

Camarões 4 Dinamarca 0

Sabe-se que Áustria e Camarões levaram apenas 1 gol, cada um. Além disso, Brasil e Dinamarca marcaram apenas 1 gol, cada um, enquanto que Áustria marcou 3 gols. Qual o resultado da partida Áustria × Dinamarca? Observação: no grupo, cada seleção joga com as demais exatamente uma vez e, em cada partida, o time vencedor ganha 3 pontos, o perdedor não ganha nem perde pontos e, em caso de empate, cada time ganha 1 ponto. A) 1 × 0 B) 2 × 1 C) 2 × 0 D) 0 × 0 E) Nada se pode afirmar.

Page 75: Eureka 2009

Sociedade Brasileira de Matemática

EUREKA! N°30, 2009

11

24) Abaixo temos um quadrado mágico multiplicativo, onde o produto dos números em cada linha, coluna e diagonal é o mesmo e igual ao número de quatro dígitos ABCD, onde cada letra representa um dígito e cada casa contém um número inteiro. Se AC representa o número de dois dígitos no centro do quadrado, a soma A + B + C + D vale:

4

AC

C 24

A) 17 B) 18 C) 19 D) 20 E) 21 25) Tenho um cubo de madeira, com três faces vermelhas e três faces azuis. O cubo é cortado em 3×3×3 = 27 cubos menores. Quantos destes cubos menores têm, pelo menos, uma face vermelha e outra azul? A) 6 B) 12 C) 14 D) 16 E) depende de quais faces do cubo são vermelhas e quais são azuis. PROBLEMAS – NÍVEL 3 01) Veja o problema No. 14 do Nível 1. 02) Sendo x = 10–2008, assinale a alternativa que apresenta o maior valor.

A) 1x

B) 1

( 1)x x + C)

x11

11

1

++

D) x E)

xx

x1

+

03) O número inteiro positivo a e o númeroa1

localizam-se na reta da seguinte

maneira:

Qual é a soma desses dois números?

A) 819 B)

809 C)

981 D)

982 E) 9

Page 76: Eureka 2009

Sociedade Brasileira de Matemática

EUREKA! N°30, 2009

12

04) Veja o problema No. 4 do Nível 2 05) Rafael tem 10 cartões. Cada um tem escrito um dos números 3, 8, 13, 18, 23, 28, 33, 48, 53, 68, e todos os dez números aparecem. Qual o menor número de cartões que Rafael pode escolher de modo que a soma dos números nos cartões escolhidos seja exatamente 100? A) 2 B) 3 C) 4 D) 5 E) não é possível obter soma 100 com esses cartões. 06) Em uma pista de corrida, cujo formato é de um polígono regular de n vértices, numerados de 1 até n no sentido anti-horário, existem três pessoas: Nelly, Sônia e Penha, estando inicialmente todas em um mesmo vértice. Em um dado momento elas começam a caminhar pelos lados do polígono. Nelly caminha no sentido anti-horário, enquanto que Sônia e Penha caminham no sentido contrário. Nelly cruza com Sônia pela primeira vez em um vértice e com Penha dois vértices à frente. A velocidade de Nelly é o dobro da velocidade de Sônia e a velocidade de Sônia é o dobro da velocidade de Penha. Quantos vértices tem o polígono? A) 30 B) 60 C) 15 D) 10 E) 6 07) Veja o problema No. 6 do Nível 2. 08) A primeira fase da OBM se realiza no dia 14 de junho, um sábado do ano bissexto 2008. Daqui a quantos anos o dia 14 de junho será novamente no sábado? A) 4 B) 5 C) 6 D) 7 E) 8

09) Veja o problema No. 14 do Nível 2. 10) O inteiro n é tal que n⋅2n possui 2008 divisores a mais que n. A soma dos algarismos de n é igual a: A) 5 B) 7 C) 9 D) 11 E) 12 11) Quantos dos números 2, 3, 5, 7, 11 são divisores de 3714 – 414? A) um B) dois C) três D) quatro E) cinco 12) Veja o Problema No. 25 do Nível 2.

Page 77: Eureka 2009

Sociedade Brasileira de Matemática

EUREKA! N°30, 2009

13

13) O número de soluções reais do sistema

+=+=+=

222

2

2

2

accbba

é igual a: A) 0 B) 1 C) 2 D) 4 E) 8 14) Arnaldo, Bernaldo, Cernaldo e Dernaldo baralharam as 52 cartas de um baralho e distribuíram 13 cartas para cada um. Arnaldo ficou surpreso: “Que estranho, não tenho nenhuma carta de espadas.” Qual a probabilidade de Bernardo também não ter cartas de espadas?

A) !52!26

!39 B)

!39!13!26

C) !52!26!39!39

D) !39!13!26!26

E) !52

!13!39

15) Veja o problema No. 19 do Nível 2.

16) Dado o quadrilátero ABCD tal que ∠CAD = 25°, ∠ACD = 45° e ∠BAC = ∠BCA = 20°, qual o valor do ângulo ∠DBC? A) 40° B) 45° C) 50° D) 55° E) 60° 17) No triângulo PQR isósceles, com PQ = PR = 3 e QR = 2, a tangente à sua circunferência circunscrita no ponto Q encontra o prolongamento do lado PR em X. O valor de RX é:

A) 5

16 B)

512

C) 38

D) 29

E) 49

18) Dado um triângulo ABC de lados AB = 3, BC = 4 e AC = 5. Sejam R1 e R2, respectivamente, os raios da circunferência inscrita e da circunferência com centro

sobre o lado BC que passa por B e é tangente ao lado AC. A razão 2

1

RR

vale:

A) 43

B) 32

C) 23

D) 98

E) 54

19) Qual o número de soluções reais do sistema

1|||| =⋅+⋅ yyxx e 1=+ yx ,

Page 78: Eureka 2009

Sociedade Brasileira de Matemática

EUREKA! N°30, 2009

14

onde x representa a parte inteira de x? A) 0 B) 1 C) 2 D) 4 E) infinitas 20) Um número de quatro dígitos é dito paladino se é múltiplo de 9 e nenhum de seus dígitos é nulo. Quantos números paladinos existem? A) 1284 B) 1024 C) 849 D) 1109 E) 729 21) Considere a função f, definida no conjunto dos números reais e satisfazendo

32)(

+=

xcxxf , para todo x ≠ −3/2. Determine o número de tais funções f para as

quais f (f (x)) = x, para todo x tal que f (f (x)) está bem definida. A) 0 B) 1 C) 2 D) 4 E) infinitas. 22) O brinquedo favorito de Cícero é um cone reto de vidro com 5 cm de altura. Cícero encheu o cone com areia até a altura de 3 cm, como mostrado na figura 1. Em seguida, Cícero fechou a base do cone e virou-o de cabeça para baixo, como indicado na figura 2. A que altura da base do cone, em cm, ficou a marca de areia?

3 cm ?

Figura 2 Figura 1 A) 1 B) 2 C) 3 985 − D) 3 98

E) 5981

3−

23) Veja o problema No. 24 do Nível 2. 24) Considere 10 pessoas, todas de alturas diferentes, as quais devem ficar em fila de tal modo que, a partir da pessoa mais alta, as alturas devem decrescer para ambos os lados da fila (se a pessoa mais alta for a primeira ou a última da fila,

Page 79: Eureka 2009

Sociedade Brasileira de Matemática

EUREKA! N°30, 2009

15

todas as pessoas a partir dela devem estar em ordem decrescente de altura). Obedecendo essas condições, de quantos modos essas pessoas podem ficar em fila? A) 256 B) 768 C) 1260 D) 512 E) 2560 25. Veja o problema No. 10 no Nível 2.

GABARITO NÍVEL 1 (6º. ou 7º. Anos)

1) A 6) E 11) C 16) C 2) C 7) B 12) E 17) C 3) B ou D 8) C 13) C 18) D 4) A 9) E 14) D 19) C ou D 5) C 10) D 15) C 20) E

NÍVEL 2 (8º. ou 9º. Anos)

1) D 6) B 11) A 16) A 21) B 2) C 7) E 12) D 17) A 22) B 3) C 8) C 13) C 18) B 23) B 4) D 9) C 14) E 19) C ou D 24) B 5) C 10) D 15) E 20) C 25) E

NÍVEL 3 (Ensino Médio)

1) D 6) C 11) D 16) C 21) B 2) A 7) B 12) B 17) B 22) C 3) D 8) C 13) E 18) B 23) B 4) D 9) E 14) D 19) C 24) D 5) D 10) A ou B 15) C ou D 20) E 25) D

Page 80: Eureka 2009

Sociedade Brasileira de Matemática

EUREKA! N°30, 2009

16

XXX OLIMPÍADA BRASILEIRA DE MATEMÁTICA Problemas e Soluções da Segunda Fase

PROBLEMAS – NÍVEL 1 – PARTE A (Cada problema vale 5 pontos)

01. Nicanor quer completar o Sudoku ao lado, de modo que em cada linha (fileira horizontal) e cada coluna (fileira vertical) apareçam todos os números de 1 a 6. Qual é a soma de todos os números que faltam para completar o Sudoku?

2

4

6

6 4

3 2

2

5

1

02. A partir das igualdades

,N820072009e

,382457

,281635

,18813

22

22

22

22

⋅=−

⋅==−

⋅==−

⋅==−

podemos escrever 1)N(N4120092 +⋅⋅=− . Qual é o valor de N? 03. Certo banco brasileiro obteve um lucro de R$ 4,1082 bilhões ao final do primeiro semestre de 2008. Esse valor representa um aumento de 2,5% em relação ao resultado obtido no mesmo período do ano passado. Qual é a soma dos dígitos do número inteiro que representa, em reais, o lucro desse banco no primeiro semestre de 2007? 04. A piscina do clube que Esmeralda freqüenta tem a forma de um hexágono (polígono com seis lados), com um ângulo interno de 270º, os demais ângulos de 90º e os quatro lados menores com 12 metros cada. Esmeralda costuma nadar pelo meio da piscina, a partir do ponto A, descrevendo o trajeto representado, na figura, pelo ângulo reto ABC, em que AB = BC.

Page 81: Eureka 2009

Sociedade Brasileira de Matemática

EUREKA! N°30, 2009

17

Certo dia, ela nadou por esse trajeto 4 vezes, isto é, foi e voltou 2 vezes. Quantos metros ela percorreu? 05. Com o dinheiro que Carlinhos tinha, poderia ter comprado 600 gramas de queijo ou 400 gramas de presunto. Usando esse dinheiro, ele resolveu comprar quantidades iguais de presunto e queijo. Quantos gramas de cada item ele comprou? 06. Quantos números inteiros maiores que zero e menores que 100 possuem algum divisor cuja soma dos dígitos seja 5? PROBLEMAS – NÍVEL 1 – PARTE B (Cada problema vale 10 pontos) PROBLEMA 1 Zezinho tem 37 cartões quadrados de lado 6 cm e 21 cartões quadrados de lado 9 cm. Ele quer colar esses cartões lado a lado, sem sobrepô-los nem deixar buracos, formando quadrados maiores. a) Apresente, através de desenhos, duas maneiras diferentes de Zezinho construir um quadrado de lado 27 cm. b) Quantos cartões são necessários para construir o quadrado com a maior área possível? PROBLEMA 2 Para construir o arranjo triangular de letras ao lado, que tem 2008 linhas, obedeceu-se a uma certa regra. a) Quantas vezes a palavra OBM aparece completamente na maior coluna desse arranjo? b) Quantas vezes a letra O aparece no arranjo? PROBLEMA 3 Em Ferius, os pontos do dominó vão de 0 a 7, ao contrário de um dominó comum, em que os pontos vão de 0 a 6. Uma peça do dominó de Ferius é chamada importante se a soma de seus pontos é par. Por exemplo, os seguintes dominós são importantes:

Page 82: Eureka 2009

Sociedade Brasileira de Matemática

EUREKA! N°30, 2009

18

a) Quantas peças diferentes possui o dominó jogado em Ferius? b) Quantas dessas peças são importantes? c) Qual é a soma dos pontos de todas as peças importantes? PROBLEMAS – NÍVEL 2 – PARTE A (Cada problema vale 5 pontos) 01. Sejam x e y números reais positivos satisfazendo as equações 2 2 1x y+ = e

4 4 17 .18

x y+ = Calcule o valor de 1 .xy

02. Um viajante, que se encontrava perdido na floresta, andou 1 metro para o Leste, 2 metros para o Norte, 3 para o Oeste, 4 para o Sul, 5 para o Leste, 6 para o Norte,..., 2006 metros para o Norte, 2007 para o Oeste e 2008 para o Sul. Calcule, em metros, o valor inteiro mais próximo da distância entre as posições inicial e final do viajante. 03. Os números α e β são as raízes da equação 2 1 0.x x− − = Calcule

75 513 β⋅+α⋅ . 04. Em um triângulo ABC, seja D um ponto sobre o lado BC tal que DB = 14, DA = 13 e DC = 4. Sabendo que o círculo circunscrito ao triângulo ADB tem raio igual ao do círculo circunscrito ao triângulo ADC, calcule a área do triângulo ABC. 05. Dado um número natural N, multiplicamos todos os seus algarismos. Repetimos o processo com o número obtido até obtermos um número com um algarismo. Este número será chamado de primitivo de N. Por exemplo, como 3 2 7 42⋅ ⋅ = e 4 2 8,⋅ = concluímos que o primitivo de 327 é 8. Calcule a soma dos algarismos do maior número natural com todos os algarismos diferentes cujo primitivo é ímpar.

Page 83: Eureka 2009

Sociedade Brasileira de Matemática

EUREKA! N°30, 2009

19

PROBLEMAS – NÍVEL 2 – PARTE B (Cada problema vale 10 pontos) PROBLEMA 1 Encontre todos os triângulos retângulos, de lados com medidas inteiras, nos quais a área tem valor numérico igual ao do perímetro. PROBLEMA 2 No quadro negro são escritos os números 2 2 2 2 21 ,2 ,3 ,4 ,...,2008 . Pedro e Igor jogam um jogo onde eles apagam alternadamente um número por vez até sobrarem apenas dois números. Se a diferença entre estes dois números for múltiplo de 2009, Igor vence. Caso contrário, quem vence é Pedro. Sabendo que Pedro é o primeiro a jogar, diga quem possui a estratégia vencedora. Justifique sua resposta. PROBLEMA 3 Seja ABC um triângulo acutângulo com BC = 5. Seja E o pé da altura relativa ao lado AC e F o ponto médio do lado AB. Se BE = CF = 4, calcule a área do triângulo ABC. PROBLEMA 4 Um país tem 8 cidades, A1, A2, ..., A6, B, C, ligadas por rodovias de mão dupla satisfazendo as seguintes condições: B e C são ambas ligadas às cidades A1, A2, ..., A6, mas não são ligadas uma à outra; A1, A2, ..., A6 são ligadas duas a duas. Calcule o número de maneiras distintas de viajar de carro de B a C, sem passar duas vezes por uma mesma cidade. PROBLEMAS – NÍVEL 3 – PARTE A (Cada problema vale 5 pontos) 01. Um trapézio isósceles ABCD, com lados paralelos AB e CD, é tal que a diagonal BD mede 100 m e o ângulo BDC mede 30°. Seja S a área do trapézio em m2. Determine 3.S ⋅ 02. Se x é um número real, denotamos por x o maior inteiro que é menor ou igual a x. Por exemplo, 2 2, 3π= = e 2,1 3.− = − Calcule o valor da soma

4 4 44 41 2 3 4 ... 2008 . + + + + +

03. Um inteiro positivo n é chamado de auto-replicante se os últimos dígitos de 2n formam o número n. Por exemplo, 25 é auto-replicante pois 225 625.= Determine

Page 84: Eureka 2009

Sociedade Brasileira de Matemática

EUREKA! N°30, 2009

20

a soma de todos os números auto-replicantes com exatamente 4 dígitos (isto é, números auto-replicantes n com 1000 9999n≤ ≤ ). 04. Quantas permutações de 1, 2, 3, ..., 9 há com a propriedade de que, para todo 1 9,i≤ < os números que aparecem entre i e i + 1 (onde i pode aparecer tanto antes como depois de i + 1) são todos menores do que i? Por exemplo, 976412358 é uma permutação com esta propriedade. 05. Suponha que α ∈ é raiz de algum polinômio não-nulo com coeficientes racionais. O polinômio minimal de α é o polinômio de menor grau m(x) tal que:

• ( ) 0;m α = • ( )m x é Mônico (isto é, o seu coeficiente líder é 1) e todos os seus

coeficientes são racionais.

Por exemplo, o polinômio minimal de 2 é 2 2.x − Determine o produto dos

coeficientes não nulos do polinômio minimal de 3 327 5 33 27 5 33.− + − + PROBLEMAS – NÍVEL 3 – PARTE B (Cada problema vale 10 pontos) PROBLEMA 1 Determine todos os inteiros positivos m e n tais que

2 161 3nm + = PROBLEMA 2 Determine a quantidade de funções :1,2,3,4,5 1,2,3,4,5f → tais que

( ( )) ( )f f x f x= para todo 1,2,3,4,5.x∈ PROBLEMA 3 Um trapézio ABCD, com lados paralelos AB e CD, está inscrito em uma circunferência de raio 25. Sabe-se que CD é um diâmetro e a altura desse trapézio é 24. Seja E um ponto no arco menor determinado por A e B e sejam F e G os pontos

de interseção de ED e EC com AB, respectivamente. Calcule .AF BGFG⋅

PROBLEMA 4 Em uma matriz 2008 2008× o elemento na linha i e coluna j é o número i + j (as

Page 85: Eureka 2009

Sociedade Brasileira de Matemática

EUREKA! N°30, 2009

21

linhas e colunas são numeradas de 1 a 2008). Escolhem-se 2008 elementos desta matriz de modo que não haja dois elementos escolhidos numa mesma linha ou coluna. Os elementos são multiplicados. Qual o menor produto que se pode obter desta forma? Soluções Nível 1 – Segunda Fase – Parte A

Problema 01 02 03 04 05 06 Resposta 91 1004 12 144 240 34 01.[91] A soma de todos os números do Sudoku completo é igual a 6 vezes a soma dos números em cada linha, ou seja, ( )6 1 2 6 6 21 126× + + + = × =… . A soma dos números que já estão escritos no Sudoku é 35. Logo a soma dos números que faltam para completar o Sudoku é126 – 35 = 91. 02. [1004] Temos:

2 22009 1 4 1 2009 1 2009 1 4 1 2008 2010 4 12008 2010 4 1 1004 1005 1 1004

2 2 2 2

N ( N ) ( )( ) N( N ) N( N )N( N ) N( N ) N

− = ⋅ ⋅ + ⇔ − + = + ⇔ ⋅ = + ⇔+

⇔ ⋅ = ⇔ ⋅ = + ⇔ =⋅

Soluções alternativas: 1a solução Cada linha pode ser associada a um número ímpar e a um múltiplo de 8 da seguinte forma: na linha 1 temos o quadrado de 1121 −⋅= (no lado esquerdo da igualdade) e 8 vezes 1 (no lado direito da igualdade), na linha 2 temos o quadrado de

1223 −⋅= e 8 vezes 2, na linha 3 temos o quadrado de 1325 −⋅= e 8 vezes 3 e assim sucessivamente, até chegarmos à linha N onde temos o quadrado de 2007 2N 1= − e 8 vezes N. Assim, 2N 1 2007 2N 2008 N 1004− = ⇔ = ⇔ = . 2a solução Cada linha pode ser associada um múltiplo de 8 da seguinte forma: na linha 1 temos 8 vezes 1 (no lado direito da igualdade), na linha 2 temos 8 vezes 2, na linha 3 temos 8 vezes 3 e assim sucessivamente, até chegarmos a última linha, onde

temos N820072009 22 ⋅=− , que é a linha 10042

12009=

− , ou seja, N = 1004.

Page 86: Eureka 2009

Sociedade Brasileira de Matemática

EUREKA! N°30, 2009

22

3a solução Temos:

1004NN840162N8)20072009)(20072009(N820072009 22 =⇔⋅=⋅⇔⋅=+−⇔⋅=− 03. [12] Seja x o lucro desse banco no primeiro semestre de 2007, em bilhões de reais. Logo 2 5 4 1082 0 025 4 1082 1 025 4 1082 4 008x , % x , x , x , , x , x ,+ ⋅ = ⇔ + = ⇔ = ⇔ = bilhões de reais, ou seja, o lucro foi de R$ 4008000000,00, cuja soma dos dígitos é 12. 04. [144] A partir das informações dadas, concluímos que na figura ID DE EF FG 12= = = = metros e que A é o ponto médio de ID , ou seja, AD = 6 metros e, da mesma forma, FC = 6 metros. Logo AB = BC = 12 + 6 = 18 metros e, portanto, Esmeralda nadou 144364)1818(4 =⋅=+⋅ metros.

05. [240] Supondo que Carlinhos tem Q reais, o preço do grama de queijo é Q600

e

o preço do grama de presunto é Q400

. Seja m a quantidade, em gramas, de queijo e

de presunto que Carlinhos comprou. Dessa forma: Q Q 1 1 1 400 600 240000m m Q m 1 m 2401 1600 400 600 400 400 600 1000

600 400

× ⋅ + ⋅ = ⇔ + = ⇔ = = = = + +

Portanto ele comprou 240 gramas de cada item. 06. [34] São os múltiplos de 5, que nesse intervalo são 19; os múltiplos de 14, que são 6 (pois o 70 já foi contado); os múltiplos de 23, que são 4; os múltiplos de 32, que são 3 e, finalmente, os múltiplos de 41, que são 2. Note que o único múltiplo de 50 no intervalo, que é o próprio 50, já foi contato nos múltiplos de 5. Portanto ao todo são 19 + 6 + 4 + 3 + 2 = 34 números.

I D

E F

G

Page 87: Eureka 2009

Sociedade Brasileira de Matemática

EUREKA! N°30, 2009

23

Soluções Nível 1 – Segunda Fase – Parte B PROBLEMA 1 a) Os desenhos mostram as duas formas de construção dos quadrados. Elas são as únicas possíveis. De fato, sendo x o número de quadrados de lado 6 cm e y o número de quadrados de lado 9 cm usados para construir um lado de 27 cm, temos:

9 2x6x 9y 27 2x 3y 9 y3−

+ = ⇔ + = ⇔ = Como x e y são inteiros não negativos,

podemos substituir x apenas por 0, 1, 2, 3 ou 4. As únicas soluções para essa situação são x = 0 e y = 3 ou x = 3 e y = 1, representadas nos desenhos. b) Repetindo mais 3 vezes a segunda construção acima, obtém-se um quadrado de lado 54 cm, com a utilização de 36 cartões de lado 6 cm e 20 cartões de lado 9 cm, sobrando apenas 1 cartão de lado 6 cm e 1 cartão de lado 9 cm. Esse quadrado é o maior que se pode construir, usando-se o maior número de cartões, 56 cartões. De fato, como os quadrados construídos com os cartões devem ter lados com medidas inteiras, concluímos que o quadrado maior do que o construído deveria ter lado de 60 cm, pelo menos, já que o cartão menor tem lado 6 cm. Como

2 2 260 54 684 cm− = é maior do que 222 cm11796 =+ , que é a soma das áreas dos quadrados que sobraram, concluímos que realmente o quadrado de lado 54 cm é o maior que se pode construir usando o maior número de cartões. PROBLEMA 2 a) A maior coluna tem 2008 letras e OBM é um bloco de 3 letras. Como

136692008 +⋅= , o número de vezes em que a palavra OBM aparece completamente na maior coluna é 669. b) Da esquerda para a direita, fazendo a contagem ao longo das flechas, a primeira passa por 2008 letras O. Como a segunda inicia 3 linhas abaixo, ela passa por

200532008 =− letras O. Nesse padrão, a próxima passará por 2002 letras O, a seguinte, por 1999, e assim até a última flecha, que passará por 1. Portanto o número de vezes que a letra O aparece no arranjo é

Page 88: Eureka 2009

Sociedade Brasileira de Matemática

EUREKA! N°30, 2009

24

6730152

670)12008(11999200220052008 =⋅+

=+++++ .

PROBLEMA 3

a) Há 28278=

⋅ peças com quantidades diferentes de pontos em cada lado e 8 com

quantidades iguais, ou seja, o dominó de Ferius tem 28 + 8 = 36 peças diferentes. Outra solução: O dominó comum possui 28 peças. Como existem mais 8 novas peças que possuem alguma casa marcando 7 pontos, o dominó de Ferius tem 28 + 8 = 36 peças diferentes. b) Como a soma de um par e um ímpar é ímpar e há 4 quantidades ímpares de pontos (1, 3, 5, 7) e 4 quantidades pares de pontos (0, 2, 4, 6), há 1644 =⋅ peças que não são importantes. Logo existem 201636 =− peças importantes. c) Cada quantidade de pontos aparece exatamente 9 vezes. Assim a soma dos pontos de todas as peças é 252)7321(9 =++++⋅ . A soma dos pontos de todas as peças que não são importantes é 112)7321(4 =++++⋅ , pois cada quantidade de pontos aparece exatamente 4 vezes em peças que não são importantes. Assim, a soma pedida é 140112252 =− . Soluções Nível 2 – Segunda Fase – Parte A

Problema 01 02 03 04 05 Resposta 6 1420 144 108 22 01. De

( )24 4 2 2 2 217 2( ) 1 2( ) ,18

x y x y xy xy= + = + − = −

obtemos 2 1( ) ,36

xy = e daí 1 6.xy

=

02. O deslocamento líquido do viajante na direção Leste-Oeste foi de ( )

502 vezes

1 3 (5 7) ... (2005 2007) ( 2) ( 2) ... ( 2) 1004.− + − + + − = − + − + + − = −

Page 89: Eureka 2009

Sociedade Brasileira de Matemática

EUREKA! N°30, 2009

25

Analogamente, o deslocamento líquido na direção Norte-Sul foi de –1004. Portanto, pelo teorema de Pitágoras a distância entre as posições inicial e final do viajante é 1004 2. Observe agora que, como 2 1,414,≅ temos 1004 2 1419,656.≅ Para ter certeza se estamos usando uma aproximação boa o suficiente, basta checar se 1419,5 1004 2 1420,< < quer dizer, se ( ) 2 2 21419,5 1004 2 1420 .< ⋅ < Mas é fácil efetuar os cálculos e verificar que essas desigualdades realmente se verificam. Logo, a melhor aproximação pedida é 1420 metros. 03. Veja que 1α β+ = e

3 2 2( 1) 2 1,α α α α α α α α= ⋅ = + = + = + 4 3 2(2 1) 2 3 2,α α α α α α α α= ⋅ = + = + = +

5 4 2(3 2) 3 2 5 3.α α α α α α α α= ⋅ = + = + = + Analogamente,

7 4 3 2(5 3)( 1) 5 8 3 13 8.β β β β β β β β= ⋅ = + + = + + = + Portanto, 5 713 5 13(5 3) 5(13 8) 65( ) 79 65 79 144.α β α β α β+ = + + + = + + = + = 04. Como os dois círculos circunscritos são iguais, segue do teorema do ângulo inscrito que ACB ABC∠ =∠ e, com isso, AB = AC.

A

C D

M

B

Seja AM a altura relativa ao lado BC. Como ABC é isósceles de base BC, segue que AM também é mediana, e daí MC = 9. Portanto, MD = 5 e, pelo teorema de

Page 90: Eureka 2009

Sociedade Brasileira de Matemática

EUREKA! N°30, 2009

26

Pitágoras, AM = 12. Finalmente, a área do triângulo ABC é

( )( ) ( )( )1 1 12 18 108.2 2

AM BC = =

05. Para que o primitivo de um número seja ímpar, todos os seus algarismos precisam ser ímpares, pois o produto de um número par por um número qualquer é sempre um número par. Assim, só nos restam os algarismos 1, 3, 5, 7 e 9 para construir o número pretendido. Por outro lado, como os algarismos precisam ser todos diferentes, o número terá, no máximo, 5 algarismos. Contudo, qualquer número com 5 algarismos ímpares e todos distintos tem primitivo 0. De fato, o produto dos números 1, 3, 5, 7 e 9 é 945 e seu primitivo é 0. O maior número com 4 algarismos ímpares e todos diferentes é 9753, mas esse número tem primitivo 0. O número que o antecede e tem seus 4 algarismos ímpares e distintos é 9751, e seu primitivo é 5. Portanto, a soma de seus algarismos é 9 + 7 + 5 + 1 = 22. Soluções Nível 2 – Segunda Fase – Parte B SOLUÇÃO DO PROBLEMA 1: Os catetos do triângulo medem a e b, e a hipotenusa mede c. Como a área e o

perímetro são iguais, temos 1 ,2

ab a b c= + + e daí 1 .2

c ab a b= − − Usando o

teorema de Pitágoras, segue que 2 2 2 2 2 2 2 2 21 1( ) 2 ,

2 4a b ab a b a b ab a b b a a b+ = − − = + + − − +

ou ainda 2 2 2 28 4 4 0.ab a b b a a b− − + = . Dividindo por ab, obtemos ( )( )4 4 8,a b− − = de maneira que a – 4 divide 8. Portanto, os possíveis valores de a são 2, 3, 5, 6, 8 e 12. Determinando os valores de b e c, encontramos os triângulos de lados 5, 12, 13 ou 6, 8, 10. SOLUÇÃO DO PROBLEMA 2: Note que ( ) ( )2 22009 2009 2009 2 ,x x x− − = − um múltiplo de 2009. Assim, sempre que Pedro apagar um número, x2 digamos, basta Igor apagar o número (2009 – x)2. Desse modo, no final restarão dois números cuja diferença é um múltiplo de 2009.

Page 91: Eureka 2009

Sociedade Brasileira de Matemática

EUREKA! N°30, 2009

27

SOLUÇÃO DO PROBLEMA 3: A

D

E F

C B

Seja D o pé da perpendicular baixada de F a AC. Pelo teorema de Pitágoras, segue que 2 2 2 25 4 3.EC BC BE= − = − = Por outro lado, por semelhança de

triângulos temos 1 22

FD BE= = e 2 .AE DE= Portanto,

2 2 2 24 2 2 3,DC CF FD= − = − = e daí 2 3 3,DE = − de maneira que 4 3 6.AE = − Finalmente,

[ ] ( ) ( )1 1 4 3 6 3 4 8 3 6.2 2

ABC AE EC BE= + = − + ⋅ = −

SOLUÇÃO DO PROBLEMA 4: Há duas escolhas envolvidas e que determinam a maneira de viajar de B a C: por quais dentre as cidades 1 6,...,A A devemos passar, e em que ordem. Digamos que escolhamos passar por exatamente k dentre as cidades 1 6,...,A A , com 1 6;k≤ ≤ o

número de modos de escolher as k cidades é 6k

. Por outro lado, após

escolhermos as k cidades, devemos escolher em que ordem vamos visitá-las, o que corresponde a k! possibilidades. Logo, o número de modos de viajar de B a C é

( )6 6

1 1

6 6! 6! 6! 6!! ... 1956.6 ! 5! 4! 0!k k

kk k= =

= = + + + = −

∑ ∑

Soluções Nível 3 – Segunda Fase – Parte A

Problema 01 02 03 04 05 Resposta 7500 9779 9376 256 18

Page 92: Eureka 2009

Sociedade Brasileira de Matemática

EUREKA! N°30, 2009

28

01. Seja P a projeção ortogonal de B sobre .CD

D C

100

B A

P 30°

Temos que 2

CD ABCP −= logo .

2AB CDPD CP AB +

= + = Assim, a área do

trapézio é: 2(100 sen30 ) (100 cos30 ) 2500 3 m

2AB CDS BP BP PD+

= ⋅ = ⋅ = ° ⋅ ° = e portanto

3 7500.S = 02. Observe que para 1i ≥ temos

4 44 4 1 ( 1)n i i n i i n i = ⇔ ≤ < + ⇔ ≤ < + e assim há ( )4 41i i+ − números n tais

que 4 .n i =

Portanto a soma pedida é: ( ) ( ) ( ) ( ) ( ) ( )4 4 4 4 4 4 4 4 4 4 41 2 1 2 3 2 3 4 3 4 5 4 5 6 5 6 2008 6 1 9779.⋅ − + ⋅ − + ⋅ − + ⋅ − + ⋅ − + ⋅ − + =

03. Seja n um inteiro de 4 dígitos. Temos que n é auto-replicante se e somente se

2n n− é divisível por 10000, isto é, 42 | ( 1)n n − e 45 | ( 1)n n − . Como n e n – 1 são primos entre si, temos 4 possibilidades:

• 42 | n e 45 | n

• ( )42 | 1n − e ( )45 | 1n −

• 42 | n e ( )45 | 1n −

• 42 | ( 1)n − e 45 | .n A primeira possibilidade implica que 410 | ,n o que é impossível pois 1000 9999.n≤ ≤ Da mesma forma, a segunda não ocorre.

Page 93: Eureka 2009

Sociedade Brasileira de Matemática

EUREKA! N°30, 2009

29

Na terceira possibilidade, de ( )45 | 1n − temos que 625 1n k= + para algum k inteiro e que 625 1 0(mod16) 1 0(mod16) 15(mod16)k k k+ ≡ ⇔ + ≡ ⇔ ≡ Assim, 15 16k = + para algum inteiro e ( )625 15 16 1 9376 10000n = + + = + E como 1000 9999,n≤ ≤ a única possibilidade é 9376.n = Finalmente, para a quarta possibilidade, temos que n = 625k, k inteiro, e que

1 0(mod16) 1(mod16)n k− ≡ ⇔ ≡ . Assim, 1 16 ,k = + inteiro, e 625(1 16 ) 625 10000n = + = + . Como 1000 9999,n≤ ≤ não há soluções neste caso. Logo o único número auto-replicante de 4 dígitos é 9376. 04. Da propriedade, decorre que 9 só pode aparecer ou como primeiro ou como último elemento da permutação e que os elementos de 1 a 8 formam uma permutação com a mesma propriedade. Assim, o número pedido é o dobro do número de permutações de 1, 2,...,8 com a mesma propriedade. Da mesma forma, o número de permutações de 1, 2,.., 8 com a propriedade é o dobro do número de permutações de 1, 2,.., 7 com a propriedade. Repetindo o raciocínio, concluímos que o número pedido é portanto 82 256.=

05. Seja 3 327 5 33 27 5 33.α = − + − + Temos

( ) ( )( )

3 3 3 3 3

33 3 5 4 33

27 5 33 27 5 33 3 27 5 33 27 5 33 27 5 33 27 5 33

54 3 96 54 2 3

α

α α α α

= − + − + − − + ⋅ + ⋅ − + − +

⇒ = − − ⋅ ⇒ + = − ⋅

Agora faça 318 .y α= Temos

( )3 6 618 54 2 3y y+ = − ⋅

( )3 3 3 2 23 2 9 35 27 0 ( 1)( 8 27) 0y y y y y y y y⇔ + = − ⇔ + + + = ⇔ + + + =

Como ,α e portanto y, são reais e 2 8 27 0y y+ + = não tem raízes reais,

concluímos que y = –1 e portanto 3 18α = − (pasmem!). Assim, α é raiz do polinômio 3 18 0,x + = que é o polinômio minimal de α já que 3 18 0x + = não possui raízes racionais.

Page 94: Eureka 2009

Sociedade Brasileira de Matemática

EUREKA! N°30, 2009

30

Soluções Nível 3 – Segunda Fase – Parte B SOLUÇÃO DO PROBLEMA 1: Olhando a equação módulo 7, temos: 2 3 ,nm ≡ porém 2m só poderá ser congruente a 0,1,2,4 enquanto que se n for ímpar 3n só poderá ser congruente a 3, 5, 6, então n deverá ser par. Logo existe 0n ∈ tal que 02 .n n= Voltando à equação original temos:

( )( )0 0 0 02 22 2161 3 3 161 3 3 161.n n n nm m m m+ = ⇔ − = ⇔ − + = Como m e n são

inteiro positivos, logo o módulo de ( )03n m− é menor que ( )03 ,n m+ e como

( )03n m− é positivo e 161 7 23,= ⋅ então temos as opções:

• 03 1n m− = e 0 03 161 3 81n nm+ = ⇔ = e 080 4m n= ⇔ = e 80 8m n= ⇔ = e 80m =

• 03 7n m− = e 0 03 23 3 15n nm+ = ⇔ = e 8.m = Não há solução inteira. Logo m = 80 e n= 8 é a única solução. SOLUÇÃO DO PROBLEMA 2: Para que ( )( ) ( )f f x f x= então a imagem de f deverá só conter pontos fixos. Utilizando esse fato temos:

• Com 5 pontos fixos na imagem teremos 1 função possível.

• Com 4 pontos fixos na imagem teremos 5

4 201

=

funções

• Com 3 pontos fixos na imagem teremos 253 90

2

⋅ =

funções

• Com 2 pontos fixos na imagem teremos 352 80

3

⋅ =

funções

• Com 1 ponto fixo na imagem teremos 451 5

4

⋅ =

funções

logo o total de funções f satisfazendo ( ( )) ( )f f x f x= igual a 196.

Page 95: Eureka 2009

Sociedade Brasileira de Matemática

EUREKA! N°30, 2009

31

SOLUÇÃO DO PROBLEMA 3:

D C

B

O

A F G

E

Como ABE ADE≅ (ambos enxergam o arco AE ) temos que FBE FDA∆ ∆∼ e portanto

FB BEFD DA

= (1)

Analogamente, das semelhanças ,EBG ACG∆ ∆∼ AEG CBG∆ ∆∼ e AEF DBF∆ ∆∼ obtemos respectivamente

BG EBCG AC

= (2)

AE AGCB CG

= (3)

AE AFDB DF

= (4)

Assim, utilizando o fato que ABCD é isósceles (de modo que AD = BC e BD = AC) temos

(2) e (4) 1AF BG AE DF CG EBFG FG DB AC⋅ ⋅ ⋅

= ⋅ ⋅

2 (1) e (3)

2 2

1 ( )( )AE CG DF EB AD AG BFAC FG AC FG

⋅ ⋅ ⋅= = ⋅

2 ( )( )AD AF FG BG FGAC FG

+ + =

Page 96: Eureka 2009

Sociedade Brasileira de Matemática

EUREKA! N°30, 2009

32

2 ( )AD FG AF FG BG AF BGAC FG

+ + + ⋅ =

2AD AF BGABAC FG

⋅ = ⋅ +

Em suma, temos 2AF BG AD AF BGAB

FG AC FG⋅ ⋅ = ⋅ +

2

2 2

AF BG AD ABFG AC AD⋅ ⋅

⇔ =−

Utilizando o fato de que ABCD é isósceles com base CD = 50 e altura 24, aplicando Pitágoras várias vezes é fácil calcular AB = 14, AD = 30, AC = 40.

Assim, 18AF BGFG⋅

= .

SOLUÇÃO DO PROBLEMA 4: Vamos mostrar que o menor produto é obtido quando tomamos os elementos da diagonal principal. Neste caso, o produto é dado por

2008(1 1)(2 2)(3 3)...(2008 2008) 2 2008!+ + + + = ⋅ Suponha que todos os elementos (1, 1), (2, 2),..., (i – 1, i – 1) tenham sido escolhidos mas que os elementos nas i– ésimas linha e colunas sejam (i, j) e (k, i) com j e k maiores ou iguais a i + 1. Vamos mostrar que trocando estes dois elementos por (i, i) e (k, j) obtemos um produto menor. De fato, para isto devemos mostrar que

( )( ) ( )( )i i j k i j i k+ + < + + 22 ( ) ( )i j k i j k i jk⇔ + < + + +

2 ( ) 0i j k i jk⇔ − + + > ( )( ) 0i j i k⇔ − − >

O que é verdade, já que 0i j− < e 0.i k− <

Page 97: Eureka 2009

Sociedade Brasileira de Matemática

EUREKA! N°30, 2009

33

XXX OLIMPÍADA BRASILEIRA DE MATEMÁTICA Problemas e Soluções da Terceira Fase

NÍVEL 1 (6o. e 7o. Anos) PROBLEMA 1 Um quadrado de lado 12 foi dividido em sete regiões retangulares que não se sobrepõem, conforme a figura. Uma delas é um quadrado de vértice C, cuja área é metade da área de cada um dos dois retângulos vizinhos; outra é um quadrado de vértice A, cuja área é metade da área de cada um dos dois retângulos vizinhos.

a) Mostre que o quadrilátero destacado é um quadrado. b) Calcule a área do quadrado destacado. PROBLEMA 2 Esmeralda escolhe um número inteiro positivo qualquer e realiza a seguinte operação com ele: cada um de seus algarismos é trocado pelo seu sucessor, com exceção do 9, que é trocado por 0. Em seguida, os eventuais zeros que aparecem à esquerda são eliminados. Por exemplo, ao se realizar a operação no número 990003953 obtém-se 1114064 (note que os dois zeros à esquerda gerados pelos dois primeiros algarismos 9 foram eliminados). A operação é repetida até que se obtenha 0. Por exemplo, começando com 889, obtemos a seqüência de números

889, 990, 1, 2, 3, 4, 5, 6, 7, 8, 9, 0 a) Apresente a sequência de números quando o primeiro número é 2008.

A B

C D

Page 98: Eureka 2009

Sociedade Brasileira de Matemática

EUREKA! N°30, 2009

34

b) Mostre que, independente do número inicial, após uma quantidade finita de operações Esmeralda obtém 0. PROBLEMA 3 Jade tem n peças iguais 13× e quer utilizá-las para cobrir um tabuleiro n×3 , sendo n um inteiro positivo. Por exemplo, para n = 4 ela pode cobrir o tabuleiro da seguinte maneira:

a) Determine de quantas maneiras Jade pode fazer a cobertura para n = 1, 2, 3, 4, 5, 6, 7. b) De quantas maneiras Jade pode cobrir o tabuleiro para n = 15? PROBLEMA 4 Considere o seguinte hexágono:

Com cópias desse polígono podemos cobrir todo o plano, sem sobreposições, como mostra a figura a seguir.

a) É possível cobrir o plano com cópias de um pentágono regular?

Page 99: Eureka 2009

Sociedade Brasileira de Matemática

EUREKA! N°30, 2009

35

Observação: um polígono é regular quando todos os seus lados são de mesma medida e todos os seus ângulos internos são iguais. b) Seja ABCDE um pentágono com todos os lados iguais e tal que a medida do ângulo interno nos vértices A e B são 100)ˆ( =Am e 80)ˆ( =Bm . Mostre como é possível cobrir todo o plano com cópias desse pentágono, sem sobreposições. PROBLEMA 5 Vamos chamar de garboso o número que possui um múltiplo cujas quatro primeiras casas de sua representação decimal são 2008. Por exemplo, 7 é garboso pois 200858 é múltiplo de 7 e começa com 2008. Observe que

728694200858 ×= . a) Mostre que 17 é garboso. b) Mostre que todos os inteiros positivos são garbosos.

TERCEIRA FASE – NÍVEL 2 (8o. e 9o. Anos) PRIMEIRO DIA PROBLEMA 1 Em cada casa de um tabuleiro nn× , colocamos um dos números 1,2,3,4, de modo que cada casa tem exatamente uma casa vizinha com o mesmo número. É possível fazer isso quando a) 2007=n ? b) 2008=n ? Observação. Duas casas são vizinhas se possuem um lado em comum. PROBLEMA 2 Seja P um pentágono convexo com todos os lados iguais. Prove que se dois dos ângulos de P somam 180 graus, então é possível cobrir o plano com P, sem sobreposições. PROBLEMA 3 Prove que existem infinitos inteiros positivos n tais que

n

n 15 2 −−

é um inteiro.

Page 100: Eureka 2009

Sociedade Brasileira de Matemática

EUREKA! N°30, 2009

36

TERCEIRA FASE – NÍVEL 2 (8o. e 9o. Anos) SEGUNDO DIA PROBLEMA 4

Mostre que se p,q são inteiros positivos primos tais que qpqpr

++

=22

é inteiro,

então r é primo. PROBLEMA 5 Seja ABC um triângulo acutângulo e O, H seu circuncentro e ortocentro, respectivamente. Sabendo que

,2

OBBHAB==

calcule os ângulos do triângulo ABC. PROBLEMA 6 Sendo A um conjunto de números inteiros, definimos S(A) como o conjunto formado pelas somas de dois elementos, não necessariamente distintos e D(A) como o conjunto formado pelas diferenças de dois elementos, não necessariamente distintos. Por exemplo, se A = 1, 2, 3, 10 então S(A) = 2, 3, 4, 5, 6, 11, 12, 13, 20 e D(A) = –9, –8, –7, –2, –1, 0, 1, 2, 7, 8, 9. Mostre que existe um conjunto finito A tal que S(A) tem no máximo 1097 elementos e D(A) tem no mínimo 10100 elementos. TERCEIRA FASE – NÍVEL 3 (Ensino Médio) PRIMEIRO DIA PROBLEMA 1 Vamos chamar de garboso o número que possui um múltiplo cujas quatro primeiras casas de sua representação decimal são 2008. Por exemplo, 7 é garboso pois 200858 é múltiplo de 7 e começa com 2008. Observe que

728694200858 ×= . Mostre que todos os inteiros positivos são garbosos. PROBLEMA 2 Sobre uma reta há um conjunto S de 6n pontos. Destes, 4n são escolhidos ao acaso e pintados de azul; os 2n demais são pintados de verde. Prove que existe um segmento que contém exatamente 3n pontos de S, sendo 2n pintados de azul e n pintados de verde.

Page 101: Eureka 2009

Sociedade Brasileira de Matemática

EUREKA! N°30, 2009

37

PROBLEMA 3 Sejam x, y, z reais quaisquer tais que x + y + z = xy + yz + zx. Encontre o valor mínimo de

111 222 ++

++

+ zz

yy

xx

TERCEIRA FASE – NÍVEL 3 (Ensino Médio) SEGUNDO DIA PROBLEMA 4 Seja ABCD um quadrilátero cíclico e r e s as retas simétricas à reta AB em relação às bissetrizes internas dos ângulos CAD∠ e CBD∠ , respectivamente. Sendo P a interseção de r e s e O o centro do círculo circunscrito a ABCD, prove que OP é perpendicular a CD. PROBLEMA 5 Prove que para quaisquer inteiros a > 1 e b > 1 existe uma função f dos inteiros positivos nos inteiros positivos tal que ( ( ))f a f n b n⋅ = ⋅ para todo n inteiro positivo. PROBLEMA 6 O profeta venusiano Zabruberson enviou a seus discípulos uma palavra de 10000 letras, sendo cada uma delas A ou E: a Palavra Zabrúbica. Seus seguidores passaram a considerar, para 100001 ≤≤ k , cada palavra formada por k letras consecutivas da Palavra Zabrúbica uma palavra profética de tamanho k. Sabe-se que há no máximo 7 palavras proféticas de tamanho 3. Determine o número máximo de palavras proféticas de tamanho 10. SOLUÇÕES – TERCEIRA FASE – NÍVEL 1 (6o. e 7o. Anos) PROBLEMA 1 SOLUÇÃO DE LUCAS CAWAI JULIÃO (CAUCAIA – CE) a) Vamos chamar o lado do quadrado de vértice C de x, e o lado do quadrado de vértice A de y. Como os retângulos que estão vizinhos a esses quadrados têm o dobro da área deles, então eles irão ter a largura com a mesma medida dos quadrados e comprimento, igual ao dobro do lado do quadrado. Veja a figura:

Page 102: Eureka 2009

Sociedade Brasileira de Matemática

EUREKA! N°30, 2009

38

A B 2y x y

y

2y

x

D 2x x C

x

2x

2x y

2x – y

Podemos ver que um lado do quadrado maior mede 3x. Para calcularmos o lado do quadrilátero central, basta retirarmos o que não pertence a ele. Logo, retiraremos x y+ . Mas isso ocorrerá dos dois lados, então os dois lados do quadrilátero destacado são iguais a 2x – y. Assim temos que ele é um quadrado. b) Como um lado do quadrado maior é 12, e já havíamos falado que também é igual a 3x. Logo x = 4. Mas também podemos perceber que a medida 2x é

equivalente a 3y. Como x = 4, então 83

y .=

Agora, como o lado do quadrado destacado é 2x – y, então sua área é ( )22x y .− Substituindo x e y, e resolvendo temos que a área do quadrado destacado

é 2569

.

PROBLEMA 2 SOLUÇÃO DE LARA VIANA DE PAULA CABRAL e RAFAEL RODRIGUES ROCHA DE MELO (FORTALEZA – CE) a) A sequencia é 2008, 3119, 4220, 5331, 6442, 7553, 8664, 9775, 886, 997, 8, 9, 0 b) Independente do dígito que ocupa a 1ª posição do número, após uma certa quantidade de operações, ele chegará a 9 e, basta mais uma operação para ele chegar a 0, que “desaparecerá”, e o número ficará assim com um dígito a menos. Em seguida, independente do dígito que agora ocupa a 2ª posição, após uma certa quantidade de operações ele também chegará a 9 e, logo depois, a 0, que também “desaparecerá”, e o número terá assim outro dígito a menos. Continuando esse processo até o número ter um único dígito, esse dígito também chegará a 9 e, depois, a 0, encerrando o processo.

Page 103: Eureka 2009

Sociedade Brasileira de Matemática

EUREKA! N°30, 2009

39

PROBLEMA 3 SOLUÇÃO OFICIAL DA BANCA Seja fn o número de maneiras possíveis de cobrir o tabuleiro 3 n× . Se a primeira coluna é coberta por uma peça vertical, falta cobrir um tabuleiro ( )3 1 .n× − Senão,

começamos com três peças na horizontal, e falta cobrir um tabuleiro ( )3 3 .n× − Assim, temos 1 3 ,n n- n-f f f= + para todo 4.n ≥ Como claramente temos 1 1f ,=

2 1f = e 3 2f ,= temos 4 3 1 5 4 2 6 5 3 7 6 43 4 6 9f f f , f f f , f f f , f f f ,= + = = + = = + = = + =

8 7 5 9 8 6 10 9 7 11 10 813 19 28 41f f f , f f f , f f f , f f f ,= + = = + = = + = = + = 12 11 9 60f f f ,= + =

13 12 10 14 13 1188 129f f f , f f f= + = = + = e, finalmente, 15 14 12 189f f f .= + = Assim, as respostas são: a) 1, 1, 2, 3, 4, 6, 9, respectivamente. b) De 189 maneiras. PROBLEMA 4 SOLUÇÃO OFICIAL DA BANCA a) Não é possível. Para que seja possível cobrir o plano com uma figura, em cada vértice determinado pelas figuras que a cobrem a soma dos ângulos internos deve ser 180º ou 360º:

Todo pentágono pode ser cortado em três triângulos, de modo que a soma de seus ângulos internos é 5401803 =⋅ . Assim, cada ângulo interno de um pentágono

regular é 1085

540= . Como 1082180108 ⋅<< e 10843601083 ⋅<<⋅ ,

não é possível cobrir o plano com cópias de um pentágono regular.

b) Note que, como 180)ˆ()ˆ( =+ BmAm , EA e BC são paralelos, de modo que EABC é um losango. Assim CE = DE = CD e CDE é um triângulo equilátero.

Soma 180º Soma 360º

Page 104: Eureka 2009

Sociedade Brasileira de Matemática

EUREKA! N°30, 2009

40

Assim é possível cobrir o plano com o pentágono ABCDE, como mostra a figura a seguir:

PROBLEMA 5 SOLUÇÃO ADAPTADA DA SOLUÇÃO DE GABRIEL YASHIMI BARRÓN TOYAMA (SÃO PAULO – SP) a) Observe que 200800 dividido por 17 tem resto 13. Assim, 200804 é múltiplo de 17 e, portanto, 17 é garboso. Na verdade, 17 tem infinitos múltiplos começados por 2008. b) Seja x a quantidade de algarismos de um número inteiro positivo y qualquer. Considere o resto m da divisão de 2008 10x⋅ por y. Temos 0 1m y ,≤ ≤ − e portanto 1 y m y.≤ − ≤ Como y tem x algarismos, 10xy ,< e logo 1 10xy m y .≤ − ≤ < Assim, y – m tem no máximo x algarismos, e portanto ( )2008 10x y m⋅ + − começa sua

representação decimal por 2008. Como 2008 10x y z m,⋅ = ⋅ + para algum inteiro z, ( ) ( )2008 10 1x y m y z⋅ + − = ⋅ + é múltiplo de y, e portanto y é garboso.

SOLUÇÕES – TERCEIRA FASE – NÍVEL 2 (8o. e 9o. Anos) PROBLEMA 1 SOLUÇÃO DE DANIEL DOS SANTOS BOSSLE (PORTO ALEGRE – RS) Perceba que a distribuição dos números no tabuleiro forma dominós 2 × 1, pois a cada casa está associada exatamente uma casa vizinha com o mesmo número. Logo, para que todos os dominós se encaixem, deve haver um número par de casas no tabuleiro. Assim, é impossível cobrir um tabuleiro 2007 × 2007.

… …

A B

C

D

E

Page 105: Eureka 2009

Sociedade Brasileira de Matemática

EUREKA! N°30, 2009

41

Por outro lado, é possível cobrir um 2008 × 2008. Uma solução é a seguinte, bastando repetir o padrão até o fim:

1 1 2 2 3 3 4 4 2 2 3 3 4 4 1 1 3 3 4 4 1 1 2 2 4 4 1 1 2 2 3 3

Assim, as respostas são: a) Não b) Sim PROBLEMA 2 SOLUÇÃO DE JOÃO LUCAS CAMELO SÁ (FORTALEZA – CE) Suponha que os ângulos suplementares sejam adjacentes. Vamos chamá-los de A e B e os outros de C , D e E . Observe a montagem a seguir:

B A

A B B

B B

C

A

E

D

C

D C

D D

C

C B

A

E E

E

A A

E C

s

r

Como 180 (5 2) 540A B C D E+ + + + = ° − = ° e 180 ,A B+ = ° temos que

360 .C D E+ + = ° Logo, é possível encaixar os pentágonos desta maneira, em “faixas”. Ao encaixarmos faixa sobre a outra pelas retas r e s da figura, poderemos cobrir o plano inteiro. Temos agora que analisar o caso quando os suplementares (dessa vez A e C ) não são adjacentes. Sendo B o ângulo do vértice entre A e C , e D e E os outros ângulos, temos a seguinte configuração: (Lembrando que 360B D E+ + = ° )

Page 106: Eureka 2009

Sociedade Brasileira de Matemática

EUREKA! N°30, 2009

42

A

B C

D

E B

C

D

E C A B

A

E

B D

A E

D

C

B

D

E B

E

D =

Hexágono α

Vamos mostrar que podemos agrupar vários “hexágonos” α de modo a cobrir o plano. Basta seguir as faixas abaixo: Faixas:

B D

E

D

B

E

B D

E

D

B

E

B D

E

D

B

E

D D

D D

D D

D D

Como os ângulos de fora valem 360 B E D° − − = e os da ponta também, é possível encaixar, cobrindo todo o plano.

Page 107: Eureka 2009

Sociedade Brasileira de Matemática

EUREKA! N°30, 2009

43

PROBLEMA 3 SOLUÇÃO DE JOÃO LUCAS CAMELO SÁ (FORTALEZA – CE) Seja p um primo 3≥ e diferente de 5. Temos

2 2 2( 1) ( 1) ( 1)5 1 5 1 (5 1) (5 1) .2 2 2

p p p p

p p p

− − − −− − − += = Analisando módulo p, pelo pequeno

Teorema de Fermat, 1 15 1(mod ) 5 1 0(mod )p pp p− −≡ ⇔ − ≡ e 1 15 1(mod 2) 5 1 0(mod 2).p p− −≡ ⇒ + ≡

Assim, 15 1p

p

− − é inteiro e 15 12

p− + é inteiro ⇒ 25 1n

n

− − é inteiro quando n = 2p.

Como existem infinitos primos p, existem infinitos n que satisfazem a condição do enunciado. PROBLEMA 4 SOLUÇÃO DE JOÃO LUCAS CAMELO SÁ (FORTALEZA – CE)

Suponha 2 2 22

2p q qp q q r q rp q q+

= ⇒ = = ⇒ = ⇒+

é primo.

Caso contrário, 2 2 2 2 2 2

22 2 | 2 .p q p q q qp q p q qp q p q p q+ − +

= = − + ∈ ⇒ ++ + +

Analogamente, 2| 2 .p q p+ Como 2 2 2 2, ( , ) 1 (2 ,2 ) 2( , ) 2.p q p q p q p q≠ = ⇒ = = Logo 2 2| (2 ,2 ) | 2 2.p q p q p q p q+ ⇒ + ⇒ + ≤ Mas , 2,p q ≥ absurdo. Logo, p q= , e portanto r é primo.

Obs.: João Lucas utilizou a notação ( , ) ( , ).a b mdc a b= PROBLEMA 5 SOLUÇÃO DE MARIA CLARA MENDES SILVA (PIRAJUBA – MG)

B C

A 30°

45°

30° α = 15° 15° = α

45° 150°

120°

M H

O

Page 108: Eureka 2009

Sociedade Brasileira de Matemática

EUREKA! N°30, 2009

44

O circuncentro é equidistante dos 3 vértices.

.2

ABBH OC OA OB= = = =

2 .AB OB= Aí 2 2 2 22AB OB OB OA OAB= = + ⇔ ∆ é retângulo em O pela recíproca do Teorema de Pitágoras. Como ,OA OB= ele também é isósceles e

45O AB OBA= = ° . Seja M o ponto médio de AC. OM é perpendicular a AC, e

temos que .2 2

BH OAOM = = Aí AO é o dobro de OM, logo ( ) 0,5sen M AO = e

como 0, , 30 .2

M AO M AO ∈ = °

π Logo 30 ,OCM = ° já que COA∆ é isósceles.

Assim 180 60 120COA = ° − ° = ° e 360 90 120 150 .COB = ° − ° − ° = °

Finalmente 180 150 152

° − °= = °α . Os ângulos são:

30 45 75 ,30 15 45° + ° = ° ° + ° = ° e 45 15 60 .° + ° = ° PROBLEMA 6 SOLUÇÃO OFICIAL DA BANCA Considere o conjunto 0,1,3.C = Temos ( ) 0,1,2,3,4,6S C = e

( ) 3, 2, 1,0,1,2,3.D C = − − − Assim, S(C) tem 6 elementos, enquanto D(C) tem 7. Vamos agora, para cada inteiro positivo n, considerar o conjunto nA dos naturais com no máximo n algarismos na base 7, todos pertencentes a C, isto é,

1

0( ) 7 ; ,0 1 .

nj

n j jj

A a a C j n−

=

= ⋅ ∈ ≤ ≤ − ∑ Dados

1

07

nj

jj

a a−

=

= ⋅∑ e 1

07

nj

jj

b b−

=

= ⋅∑ em

nA , com , ,j ja b C∈ para 0 1,j n≤ ≤ − temos ( )1

07

nj

j jj

a b a b−

=

+ = + ⋅∑ e

( )1

07 .

nj

j jj

a b a b−

=

− = − ⋅∑ Assim, temos 1

0( ) 7 , ( ),0 1

nj

n j jj

S A u u S C j n−

=

= ⋅ ∈ ≤ ≤ − ∑

e 1

0( ) 7 , ( ),0 1 .

nj

n j jj

D A v v D C j n−

=

= ⋅ ∈ ≤ ≤ − ∑

Como S(C) tem 6 elementos entre 0 e 6, e a representação em base 7 é única, S( nA )

tem exatamente 6n elementos. Por outro lado, como 1

0

7 1 3 7 ,2

n nj

j

=

−= ⋅∑ temos

Page 109: Eureka 2009

Sociedade Brasileira de Matemática

EUREKA! N°30, 2009

45

7 1 7 1( ) : , ( )2 2

n n

n nD A m m D A − −

+ = + ∈ =

( )1 1

0 03 7 , ( ),0 1 7 , 0,1,2,3,4,5,6,0 1

n nj j

j j j jj j

v v D C j n r r j n− −

= =

= + ⋅ ∈ ≤ ≤ − = ⋅ ∈ ≤ ≤ − = ∑ ∑

0,1,2,...,7 1 ,n= − pois todo inteiro entre 0 e 7 1n − pode ser representado na base 7,

usando os algarismos 0, 1, 2, 3, 4, 5 e 6. Assim, ( )nD A tem 7n elementos.

Como 5 100

6 5 6 120 97 100 12020

10 106 10 7 ,6 10 10 7 ,2 2

< < < < < < < e portanto o conjunto

120 ,A A= que tem 1203 elementos, certamente satisfaz as condições do enunciado. SOLUÇÕES – TERCEIRA FASE – NÍVEL 3 (ENSINO MÉDIO) PROBLEMA 1 SOLUÇÃO DE CUSTÓDIO M. B. SILVA Seja n um inteiro positivo. Como n é inteiro finito, 10 ,kn < para algum k. Seja 10 2008kp n q= ⋅ + − , onde q n< é o resto da divisão de 10 2008k ⋅ por n. Assim, 10kn q− < e portanto p começa com 2008 e é múltiplo de n. PROBLEMA 2 SOLUÇÃO DE RAFAEL SUSSUMU YAMAGUTI MIADA (SÃO PAULO – SP) Considere que os pontos são numerados de 1 a 6n. Sabe-se que, para 1 3 1,b n≤ ≤ + um segmento de b até 3 1n b+ − contém exatamente 3n pontos e será representado como 3 1.b n b→ + − Como os pontos devem ser consecutivos, pode-se formar 3n + 1 segmentos (1 3 ;2 3 1;3 3 2,...,3 1 6 )n n n n n→ → + → + + → . Vamos analisar a variação do número de pontos verdes de 3 1b n b→ + − até

1 3 .b n b+ → + Considere que em 3 1b n b→ + − há z pontos verdes. Pode acontecer: z pontos → z – 1 pontos: Sai um ponto verde e não entra outro ponto verde no

segmento.

z pontos → z pontos: Sai um ponto verde e entra outro ponto verde no segmento.

z pontos → z pontos: Não sai um ponto verde e não entra outro ponto

Page 110: Eureka 2009

Sociedade Brasileira de Matemática

EUREKA! N°30, 2009

46

verde no segmento.

z pontos → z + 1 pontos: Não sai um ponto verde e entra outro ponto verde no segmento.

(quantidade de pontos verdes) (em relação ao segmento anterior) Maior variação: 1 ponto para mais ou para menos. Considere então os pontos de 1 → 3n e 3n + 1→ 6n (usando o fato de que há 2n pontos verdes e 4n pontos azuis). Se em 1 → 3n há n + k pontos verdes, em 3n + 1 > 6n haverá n – k pontos verdes. Além disso, em 1 → 3n haverá 2n – k pontos azuis e em 3n – 1 → 6n haverá 2n+ k pontos azuis. Temos os seguintes casos: a) para k = 0: é verdadeiro na primeira e última sequência (1→ 3n e 3n + 1 → 6n) (verdadeiro!). b) para k < 0: deve aumentar o número de pontos verdes de 1→ 3n a 3n + 1 → 6n, porém com a máxima variação entre cada sequência é 1 ponto e n + k < n < n – k , conclui-se que existe a → a + 3n – 1 talque o número de pontos verdes é igual a n (verdadeiro!). O caso k > 0 é análogo. Como há 3n pontos na sequência a → a + 3n – 1 e os pontos são verdes ou azuis, pode-se qualificar o fato de que existe uma sequência a → a + 3n – 1 tal que há n pontos verdes e 2n pontos azuis como verdadeiro. PROBLEMA 3 SOLUÇÃO DE RÉGIS PRADO BARBOSA (FORTALEZA – CE) Para (x, y, z) = (–1, –1, 1), temos

x + y + z = –1 – 1 + 1 = –1 xy + yz + zx = (–1)(–1) + (–1)1 + (–1)1 = –1

e

2 2 2

1 1 1 11 1 1 2 2 2 2

x y z .x y z

+ + = − − + = −+ + +

Provaremos que 21

− é o mínimo, ou seja, sendo x + y + z = xy + yz + zx

mostraremos que 21

111 222 −≥+

++

++ z

zy

yx

x.

A desigualdade é equivalente a

Page 111: Eureka 2009

Sociedade Brasileira de Matemática

EUREKA! N°30, 2009

47

2 2 2 2 2

2 2 2 2 2 2 2 2 2 2 2 2 2 2 2 2

2 2 2 2 2 2 2 2 2

2 1 1 1 1 1

2 1

2 2 2 1 0

cic

cic

cic sim

x( y )( z ) ( x )( y )( z )

( xy z xy xz x ) ( x y z x y y z z x x y z )

( x y x xy z x ) x y x y z

+ + ≥ − + + +

⇔ + + + ≥ − + + + + + + +

⇔ + + + + + + ≥

∑ ∑

(usamos as anotações cic∑ e

sim∑ para denotar soma cíclica e soma simétrica

respectivamente). Mas

2 2 2

cic cic

xy z xyz( xy yz zx ) xyz( x y z ) x yz= + + = + + =∑ ∑ e

( ) ( ) ( )( )( ) ( )

2 2 2 2 2 2 2

2

3 3

3

3 3

sim

x y x y xy x z x z y z yz xyz xyz

xy x y z yz x y z zx x y z xyz

xy yz zx x y z xyz x y z xyz

= + + + + + + −

= + + + + + + + + −

= + + + + − = + + −

Assim, a desigualdade é equivalente a

( ) ( )22 2 2 2 2 2 22 2 2 6 1 0cic

x y x x yz x x y z xyz x y z+ + + + + + − + + ≥∑

Agora montemos quadrados: 2222 )(2 yzxzyxyzx −=+− 2222 )(2 zxyzxxyzy −=+− 2222 )(2 xyzyxxyzz −=+−

22 )1(1)(2)( +++=++++++ zyxzyxzyx 22222 )()(2)( xyzzyxzyxzyxxyzzyx +++=++++++

Observando que ( )2

cicx yz xyz x y z= + +∑ , a desigualdade é equivalente a

0)()1()()()( 22222 ≥++++++++−+−+− xyzzyxzyxxyzzxyyzx ,

que é verdadeira pois 02 ≥A para todo A real.

Logo o mínimo da soma dada é 21

− .

PROBLEMA 4 SOLUÇÃO DE MARCO ANTONIO LOPES PEDROSO (SANTA ISABEL – SP) Para termos as bissetriz de C AD e CBD determinados não precisa saber as posições dos pontos C e D, basta a posição do ponto médio do arco DC , que

Page 112: Eureka 2009

Sociedade Brasileira de Matemática

EUREKA! N°30, 2009

48

vamos chamar de L. Perceba também que a mediatriz de CD é perpendicular a CD e passa por O; desse modo o nosso problema passa a ser provar que P também está na mediatriz de CD. Mas já sabemos que L está na mediatriz de CD, então na realidade queremos provar que O, P, L são colineares (agora podemos esquecer o C e o D e pensar só no L). Então nosso problema passa a ser:

B

P

A

L

α α

β β

O

Provar que O,P,L são colineares.

Perceba que LA é a bissetriz externa do PAB∆ relativa ao vértice A; e LB é a bissetriz externa do PAB∆ relativa ao vértice B. Logo L é o ex-incentro do PAB∆ relativo a P; desse modo PL é bissetriz do ângulo APB.

É natural pensar no incentro I do PAB;∆ como PL é bissetriz de APB então P, I, L são colineares.

B

P

A

L

α α

β β

O

I 90° – β 90° – β 90° – α

90° – α

Page 113: Eureka 2009

Sociedade Brasileira de Matemática

EUREKA! N°30, 2009

49

Como bissetriz interna e externa de um ângulo são perpendiculares então IA AL;⊥

assim como LB BI ,⊥ desse modo temos 90LAI LBI= ° = ⇒ o quadrilátero LAIB é inscritível, e seu centro está no centro da hipotenusa do LAI .∆ Logo O está no ponto médio de LI (pois é o centro da circunferência que passa por L, A, B). Então L, O, I são colineares, e como já provamos que P, I, L são colineares então P, O, L são colineares, como queríamos demonstrar. PROBLEMA 5 SOLUÇÃO DE GABRIEL LUIS MELLO DALALIO (S.J. DOS CAMPOS – SP) Seja nS a sequência crescente dos inteiros positivos não múltiplos de a e nR a sequência crescente dos inteiros positivos não múltiplos de b. Definindo : * *:f →

se ( ) onde é tal que

( ) se com ( ) onde é tal quese com ( ) ( )

k k

i i

a |n, f n =R , k S = n

f n = n = ak, k *, b|k, f n b S i R k, n = abj, j *, f n ab f j

∈ = ⋅ = ∈ = ⋅

Devemos provar que a recursão acaba, mas de fato, como ao passar pelo terceiro caso precisamos do valor da função em um número j < n , já que n = abj, e portanto alguma hora a recursão cai em algum dos dois primeiros casos. Vamos provar que ( ( ))f af n bn= para todo n inteiro positivo. Temos os seguintes casos: 1) ( ( )) ( ), ondek ka | n f af n = f a R S = n,⇒ e temos

( )k kf a R b S =bn= ⋅ , donde ( ( ))f a f n bn= quando a | n . 2) a |n . Temos dois subcasos:

2.i) n = ak, b | k ⇒ f (a f (n)) = f ( af (ak)) = f (ab . Si), onde Ri = k, e temos

f (ab Si) = abf(Si) = abRi = bak = bn ⇒ f(af(n)) = bn quando n = ak e b | k .

2.ii) n = abj, j *∈ ⇒ f (a f (n)) = f (af (abj)) = f (ab· a f( j)) = ab· f(af (j)).

Se ( ( )) ( ( ))ab | j, f af j = bj f af n ab bj bn.⇒ = ⋅ = Se *,ab | j, j=abi,i∈ i < j. Podemos supor, por indução, que f(a f(j)) = bj, donde ( ( )) ( ( )) f af n = ab f af j ab bj bn, c.q.d.⋅ = ⋅ = PROBLEMA 6 SOLUÇÃO OFICIAL DA BANCA Seja f(n) o número de palavras proféticas de tamanho n.

Page 114: Eureka 2009

Sociedade Brasileira de Matemática

EUREKA! N°30, 2009

50

Temos (1) 2, (2) 4f f≤ ≤ e (3) 7.f ≤ Assim, há uma palavra XYZ de três letras X,Y,Z pertencentes a A,E que não é profética. Para 1,n ≥ uma palavra proféticas de tamanho n + 3 pode ser de três tipos (no máximo): - uma palavra profética de tamanho n + 2 seguida da letra U A, E∈ distinta de Z, - uma palavra profética de tamanho n + 1 seguida de TZ, onde T A,E∈ é a letra distinta de Y ou uma palavra profética de tamanho n seguida de SYZ, onde S A,E∈ é a letra distinta de X. Assim, ( 3) ( 2) ( 1) ( )f n + f n + + f n + + f n≤ para todo 1;n ≥ logo,

(4) 13, (5) 24, (6) 44, (7) 81, (8) 149, (9) 274f f f f f f≤ ≤ ≤ ≤ ≤ ≤ e (10) 504.f ≤ Vamos agora ver que é possível que haja 504 palavras proféticas de tamanho 10. Para isso observamos inicialmente que há 504 palavras de tamanho 10 que não têm três letras E consecutivas. Para 1,n ≥ uma palavra de tamanho n + 3 sem 3 E´s seguidos pode ser de três tipos, todos distintos: uma palavra sem 3 E´s seguidos de tamanho n + 2 seguida da letra A, uma palavra sem 3 E´s seguidos de tamanho n + 1 seguida de AE ou uma palavra sem 3 E´s seguidos de tamanho n seguida de AEE. Isso mostra que, se g(n) é o número de palavras de n letras, todas A ou E, sem 3 E´s consecutivos, então g(n + 3) = g(n + 2) + g(n + 1) + g(n) para todo

1.n ≥ Como g(1) = 2, g(2) = 4 e g(3) = 7, segue que g(10) = 504. Agora; como 11 504 10000,⋅ < basta listar todas essas palavras, colocar uma letra A no final de cada uma delas e concatená-las, completando com letras A até obtermos uma palavra de10.000 letras para concluir.

Page 115: Eureka 2009

Sociedade Brasileira de Matemática

EUREKA! N°30, 2009

51

XXX OLIMPÍADA BRASILEIRA DE MATEMÁTICA Problemas e Soluções da Primeira Fase Nível Universitário

PROBLEMA 1 Determine todos os valores inteiros de n para os quais a equação 3 13 0x x n− + = possua três raízes inteiras. PROBLEMA 2 Considere as retas de equações paramétricas

( , , ) (0,0,1)( , , ) (1,2,0) (1,0,0)( , , ) (1,1,1) (0,1,0)( , , ) (1,0,0) (1,1,1)

x y z tx y z tx y z tx y z t

= ⋅= + ⋅= + ⋅= + ⋅

Quantas retas intersectam simultaneamente as 4 retas acima? PROBLEMA 3 Esmeralda passeia pelos pontos de coordenadas inteiras do plano. Se, num dado momento, ela está no ponto (a, b), com um passo ela pode ir para um dos seguintes pontos: (a +1, b), (a –1, b), (a,b + 1) ou (a, b – 1). De quantas maneiras Esmeralda pode sair do (0, 0) e andar 2008 passos terminando no (0,0)? PROBLEMA 4 Suponha que existem duas matrizes inversíveis n n× , A e B, diferentes da matriz identidade I e satisfazendo as relações

7

1 2

A IABA B−

=

=

Mostre que existe um inteiro 0k > tal que kB I= e determine o menor k com esta propriedade. PROBLEMA 5 Dizemos que uma hipérbole cobre um ponto se este pertence a uma das duas regiões infinitas por ela determinada que contêm os focos. Qual o menor número de hipérboles necessárias para cobrir todos os pontos do plano?

Page 116: Eureka 2009

Sociedade Brasileira de Matemática

EUREKA! N°30, 2009

52

PROBLEMA 6

Seja 0

nn

k n

kP sennπ

≤ ≤

=

∑ .

Calcule 1lim n n

n

P Pn

+

→∞.

SOLUÇÕES PRIMEIRA FASE – NÍVEL UNIVERSITÁRIO PROBLEMA 1 Sejam , ,α β γ as três raízes do polinômio. As relações de Girard implicam que

0s = + + =α β γ e 13,p = + + = −αβ βγ γα logo 2 2 2 2 2 26.s p+ + = − =α β γ As únicas possibilidades para , , : 4, 3, 1+ − −α β γ ou 4, 3, 1.− + + Logo 12.n = − = ±αβγ PROBLEMA 2 PRIMEIRA SOLUÇÃO Sejam (0,0, ), (1 ,2,0), (1,1 ,1)A a B b C c= = + = + e (1 , , )D d d d= + pontos genéricos, um sobre cada uma das 4 retas dadas. Esses pontos são colineares se, e somente se, a matriz

0 0 11 2 0 1

1 1 1 11 1

ab

Mc

d d d

+ = + +

tem posto 2. Subtraindo a primeira linha de M das demais obtemos a matriz equivalente

0 0 11 2 0

1 1 1 01 0

ab a

Mc a

d d d a

+ − = + − + −

que tem posto 2 se, e somente se 21

1 1ab

c a−

+ = =+ −

e 1 .1 1

d d adc a

−+ = =

+ −

Três dessas quatro igualdades nos permitem expressar b, c e d em função de a:

Page 117: Eureka 2009

Sociedade Brasileira de Matemática

EUREKA! N°30, 2009

53

1 2 1, , ;1

ab c da a a

−= = =

− a quarta, então, equivale a 22 2 0,a a− − = equação que

possui duas soluções reais. Logo há duas retas que intersectam simultaneamente as 4 retas dadas. SEGUNDA SOLUÇÃO As coordenadas de Plücker das quatro retas são:

1 : 0,0,1 0,0,0r

2 : 1,0,0 0,0, 2r −

3 : 0,1,0 1,0,1r −

4 : 1,1,1 0, 1,1r −

Qualquer solução : , , , ,x y z x y zr d d d p p p tem que ser ortogonal às quatro retas.

Resolvendo o sistema linear, temos que : 2 , ,0 ,2 , ;r − +α β α β α β α finalmente,

como 0,d pr r⋅ = temos que ter

2 2 3 173 2 02

− ±+ − = ⇔ =

ββ αβ αα

logo existem duas retas que intersectam as quatro retas dadas. PROBLEMA 3 Cada movimento de subida ( )↑ deva ser compensado por um movimento de

descida ( )↓ , e cada movimento para a esquerda ( )← deve ser compensado por um

movimento para a direita ( ).→ Assim, se fizermos k movimentos ↑ , temos que

fazer também k movimentos ,↓ 1004 k− movimentos ←e 1004 k− movimentos .→

Para cada k, o número de caminhos é, portanto, igual ao número de anagramas com 4 letras distintas, duas aparecendo k vezes e as outras duas, 1004 k− vezes cada. Logo a resposta é

1004

0

2008!! !(1004 )!(1004 )!k

Rk k k k=

=− −∑

Page 118: Eureka 2009

Sociedade Brasileira de Matemática

EUREKA! N°30, 2009

54

1004

0

2008! 1004!1004!1004!1004! ! !(1004 )!(1004 )!k k k k k=

= ⋅− −∑

21004

0

2008 1004.

1004 k k=

=

Considere agora um conjunto de n meninos e n meninas. De quantas maneiras

podemos escolher um grupo de n crianças? Por um lado, a resposta é 2

.

nn

Por outro lado, se escolhermos k meninos 2 n n n

k n k k

= − maneiras de formar

um grupo. Logo 2

0

2n

k

n nk n=

=

e portanto 22008.

1004R

=

SEGUNDA SOLUÇÃO

Esmeralda tem 22008

1004

maneiras de escolher dois conjuntos de 1004 passos dentre

os 2008 passos que andará: o conjunto X dos passos para cima ou para a direita ( ) ou ↑ → e o conjunto Y dos passos para baixo ou para a direita ( ) ou .↓ →

Essas escolhas determinam unicamente todos os passos: O conjunto dos passos para a direita será ,X Y∩ para a esquerda será ,c cX Y∩ para cima cX Y∩ e para baixo cX Y∩ (onde cX e cY denotam os complementares de X e Y, respectivamente). Se ,X Y k∩ = teremos 1004 , 1004c cX Y k X Y k∩ = − ∩ = −

e .c cX Y k∩ = Assim, a resposta é 22008.

1004

PROBLEMA 4 Note que ( ) ( ) ( )2 24 2 1 2 1 1 1 2 2 .B B ABA AB A A ABA A A BA− − − − −= = = = =

De forma análoga,

Page 119: Eureka 2009

Sociedade Brasileira de Matemática

EUREKA! N°30, 2009

55

8 3 3 16 4 4 32 5 5 64 6 6 128 7 7, , , , ,B A BA B A BA B A BA B A BA B A BA B− − − − −= = = = = = logo 127 .B I=

Suponha agora que existe 0 127k< < tal que ;kB I= como 127 é primo, o m.d.c. entre 127 e k vale 1. Pelo Teorema de Bézout, existem a, b inteiros tais que 127 1;a kb+ = então

( ) ( )1 127 127 .a ba kb kB B B B B I+= = = ⋅ =

Isso é uma contradição, pois .B I≠ Logo o menor valor de k é 127. Nota: Não é necessário exibir exemplos de tais matrizes A e B, mas tais exemplos existem. Podemos fazer n = 127, enumerar uma base de 127 como 0 1 126, ,...,e e e e definir A e B por 2 (mod127)j jAe e= e 2 1(mod127) ,0 126.j jBe e j+= ≤ ≤

PROBLEMA 5 Como toda hipérbole tem duas assíntotas não paralelas, dadas duas hipérboles, sempre existe pelo menos um ponto comum a uma assíntota de cada uma delas. Esse ponto não é coberto por qualquer uma das duas hipérboles, logo é impossível cobrir todo o plano com apenas duas hipérboles. As seguintes três hipérboles cobrem todo o plano:

2 2 1x y− = 2 2( 2) 1y x− − = 2 2( 2) 1y x+ − =

De fato, para qualquer 2( , ) ,x y ∈ vale pelo menos das seguintes desigualdades: 2 2 1,x y> + 2 2( 1) 1x y< − − ou 2 2( 2) 1.x y< + − Com efeito,

( ) ( ) ( )22 2 2 2max 2 1, 2 1 2 1 4 3 1.y y y y y y− − + − = + − = + + > +

Assim o número mínimo de hipérboles necessárias para cobrir todos os pontos do plano é 3. PROBLEMA 6 Observe inicialmente que

( ) ( )0

1 1 .nn nP sen xdx P

n n− < < +∫

ππ π

Defina 0

.nnI sen xdx= ∫

π Integrando por partes, temos que, para 2,n >

1

0

nnI sen xsenxdx−= =∫

π

Page 120: Eureka 2009

Sociedade Brasileira de Matemática

EUREKA! N°30, 2009

56

1 2

0 0cos ( 1) cos ( cos )n nsen x x n sen x x x dx− − = − − − − = ∫

ππ

( ) 2 2

01 (1 )nn sen x sen x dx−= − − =∫

π

( ) 21 ( 1)n nn I n I−= − − −

e portanto 21 ;n n

nI In −

−= daí segue que

2

lim 1.n

nn

II→∞

= Como, para todo n,

2 1 ,n n nI I I− −≥ ≥ temos 1

lim 1.n

nn

II→∞

=

Como 1 2I = e 2 ,2

I =π temos que para todo 0,k ≥

2 1 2 2(2 )!! (2 1)!!2, ,

(2 1)!! (2 2)!!k kk kI I

k k+ ++

= ⋅ = ⋅+ +

π

onde 0!! ( 2 ).kn n k>= −∏ Assim, 2 1 2 22 (2 1)lim(2 1) lim 2 ,

2 2k kk k

kk I Ik+ +→∞ →∞

++ = =

+π π e

2 2 2 32 (2 2)lim(2 2) lim 2 ,

2 3k kk k

kk I Ik+ +→∞ →∞

++ = =

+π π ou seja, 1lim 2 ,k n nnI I→∞ + = π donde

1 12

2lim lim .n n n n

k k

P P nI In

+ +

→∞ →∞= =

π π

Obs.: Alternativamente, pela aproximação de Stirling,

2

(2 1)!! (2 )!(2 )!! (2 )!!k k

k k−

= =

2

(2 )! ~2 !k

k

k=

2 2 1

2 2 1

(2 ) 4 (1 ( ))~ ~4 2 (1 ( ))

k k

k k k

k e k O kk e k O k

− −

− −

++

ππ

321~ O k

k−

+ π

e portanto 322~ .nI O n

n−

+

π Mas isso implica 2~ (1)n O+π

e portanto

1 2lim .n nn

P Pn

+→∞ =

π

Page 121: Eureka 2009

Sociedade Brasileira de Matemática

EUREKA! N°30, 2009

57

XXX OLIMPÍADA BRASILEIRA DE MATEMÁTICA Problemas e Soluções da Segunda Fase Nível Universitário

PRIMEIRO DIA PROBLEMA 1

Seja :nf → dada por 2008

2( ) ,2008xf x x nx= + − para cada ,n∈ e seja nm o

valor mínimo assumido por .nf Determine α ∈ tal que o limite lim n

n

mnα→∞

existe e

é não-nulo, e calcule esse limite (para esse valor de α ). PROBLEMA 2

No 3 , considere a elipse ε1 definida pelas equações 0x = e 2 241 41 80 36 36 81 0,y z yz y z+ − + + − = e a elipse ε2 definida pelas equações 0y = e 2 271 41 40 18 36 81 0.x z xz x z+ − + + − = Prove que existe uma única

superfície cônica de revolução no 3 que intersecta o plano 0x = em ε1 e o plano

0y = em ε2, e determine a interseção dessa superfície com o plano z = 0. PROBLEMA 3

Mostre que existem 1 2, ,...,a a ∈ tais que a série 1

nn

na x

=∑ converge para todo

x∈ e, definindo 1

( ) ,nn

nf x a x

=

=∑ temos:

i) f é uma bijeção de em que satisfaz ( ) 0, .f x x> ∀ ∈ ii) ( )f = A , onde | ( )p x= ∈ ∃A α polinômio com coeficientes inteiros tal que ( ) 0p α = é o conjunto dos algébricos reais. SEGUNDO DIA PROBLEMA 4 Seja 2[0,1] [0,1]Q = × ⊂ um quadrado de lado 1 e :f Q → uma função contínua e positiva. Prove que é possível dividir Q em duas regiões 1R e 2R de

Page 122: Eureka 2009

Sociedade Brasileira de Matemática

EUREKA! N°30, 2009

58

mesma área, separadas por um segmento de reta, tais que

1 2( , ) ( , ) .R Rf x y dxdy f x y dxdy=∫ ∫

PROBLEMA 5 Prove que não existe uma matriz 7 7,× 1 , 7( ) ,ij i jA a ≤ ≤= com 0,1 , 7ija i j≥ ≤ ≤ cujos autovalores (contados com multiplicidade) são: 6, –5, –5, 1, 1, 1, 1. PROBLEMA 6

Prove que 2 2 21 1

1 1 , 0.( ) 2n nn n

λ λλ λ

∞ ∞

= =

< ∀ ≥+ +∑ ∑

SOLUÇÕES SEGUNDA FASE – NÍVEL UNIVERSITÁRIO PRIMEIRO DIA PROBLEMA 1 SOLUÇÃO DE EDUARDO POÇO (SÃO PAULO – SP) Seja 2007

nx n.= Como nm é o mínimo de nf , então:

2007 220071( ) 1 , .2008n nm f x n n n n ≤ = − + ∀ ∈

Seja agora 2008

( ) .2008nxg x nx= −

Temos ( ) ( )n nf x g x , x ,≥ ∀ ∈ e sendo nk o valor mínimo de ng :

( ) ( )n n n n nf x g x k , x m k , n≥ ≥ ∀ ∈ ⇒ ≥ ∀ ∈

Calculando nk : ( )ng x mínimo ( ) 0ng´ x⇒ = ⇒

2007 2007 200710 12008nx n x n k n n ⇒ − = ⇒ = ⇒ = −

Assim: 1 1 21 1

2007 2007 20071 11 12008 2008nn m n n

+ + − ≤ ≤ − +

Como 2 112007 2007

,< + devemos ter 1 200812007 2007

,= + =α e o limite é

Page 123: Eureka 2009

Sociedade Brasileira de Matemática

EUREKA! N°30, 2009

59

1 200712008 2008

.− = −

PROBLEMA 2 SOLUÇÃO OFICIAL DA BANCA Sejam ( )0 0 0x , y ,z o vértice do cone e ( )a,b,c um vetor não nulo na direção do eixo

do cone. Se ( )x, y,z é um ponto do cone, existe um ângulo θ tal que o ângulo

entre os vetores ( )0 0 0x x , y y ,z z− − − e ( )a,b,c é sempre θ ou ,−π θ e em

particular o módulo de seu cosseno é igual a cos .θ Assim,

( ) ( ) ( )( ) ( ) ( )

0 0 0

2 2 22 2 20 0 0

a x x b y y c z z

a b c x x y y z z

− + − + −=

+ + ⋅ − + − + −

( ) ( )( ) ( )

0 0 0

0 0 0

a,b,c x x ,y y ,z zcos

a,b,c x x ,y y ,z z⋅ − − −

=⋅ − − −

θ ,

e podemos escrever a equação do cone como

( ) ( ) ( )( ) ( ) ( ) ( )( )2 2 2 220 0 0 0 0 0 0a x x b y y c z z d x x y y z z ,− + − + − − − + − + − = onde

2 2 2d cos . a b c ,= + +θ a qual pode ser escrita na forma 2 2 2 0Ax By Cz Dxy Exz Fyz Gx Hy Iz J .+ + + + + + + + + = Como, fazendo x = 0, obtemos

uma equação da elipse ε1, podemos supor (ajustando o módulo de (a, b, c), e trocando os sinais, se necessário) que B = 41, C = 41, F = –80, H = 36, I = 36 e

J = –81, e como, fazendo y = 0, obtemos uma equação da elipse ε2, teremos A = 71, E = –40 e G = 18 (note que os coeficientes de z2, z e o coeficiente constante são

os mesmos nas equações dadas de ε1 e ε2). Assim, basta determinar D. Comparando as equações, devemos ter 2 2 2 2 2 271 41 2 40a d , b d c d , ac− = − = − = =− e 2 80bc=− ou 2 2 2 2 2 271 41 2 40a d , b d c d , ac− =− − = − =− = e 2 80bc .= Nos dois casos, b = 2a, donde 2 2 2 2b d a d ,− ≥ − o que não acontece no primeiro caso. Assim, ocorre o segundo caso, e portanto 22D ab b .=− =− Como 2 2 2 2b d c d ,b c ,− = − = e portanto

22 2 80 402 2bc

D b b bc .=− =− =− =− =− =− Em particular, a interseção do cone com o

plano z = 0 é dada pela equação 2 271 41 40 18 36 81 0x y xy x y ,+ − + + − = e logo é uma elipse. Obs.: Fazendo ( ) ( ) ( ) ( )0 0 0 1 2 2 10 2 10 2 10x ,y ,z , , , a,b,c , ,= = e d = 9 obtemos a

equação desejada (após trocar os sinais).

Page 124: Eureka 2009

Sociedade Brasileira de Matemática

EUREKA! N°30, 2009

60

PROBLEMA 3 SOLUÇÃO OFICIAL DA BANCA Construiremos uma função que satisfaz as condições do enunciado da forma

( )1

( ) n nn

f x x c g x ,∞

=

= + ⋅∑ onde as funções ng têm derivada limitada na reta real e as

constantes nc convergem a 0 muito rápido, de tal forma que

( ) 1

1 12n n ng sup g x ,x , n ,+≤ ∈ < ∀ ≥ o que garantirá que ( )1 3

2 2f´ x , x ,< < ∀ ∈

e portanto f será uma bijeção crescente de em . Os conjuntos e A são subconjuntos enumeráveis densos de . Podemos enumerá-los como nx ,n= ∈ e A ny ,n .= ∈ Para cada 1n ≥ construiremos um conjunto nB ⊂ com n elementos, com 1 1n nB B , n+⊂ ∀ ≥ e tomaremos

( ) ( )n

nb B

g x sen x b ,∈

= −∏ que é limitada e tem derivada limitada em . Tomamos

1 0B = e 1 0c .=

Como ( ) ( ) ( ) ( )1 1 1 1 u = =2 2 2 2

sen cos v sen u v sen u v , sen u sen v cos u v cos u v ,⋅ + + − ⋅ − − +

( ) ( ) + sen x a cos a sen x sen a cos x, cos x a cos a cos x sen a sen x,+ = ⋅ ⋅ + = −

( ) = sen u sen u− − e ( )cos = cos u u,− podemos escrever ( )ng x como

( ) ( ) ( ) ( )( )0

nn n

k kk

r sen kx s cos kx .=

+∑ Como, para todo x ,∈ ( )( )

2 1

0

1

2 1 !

k k

k

xsen x

k

+∞

=

−=

+∑ e

( )( )

2

0

1

2 !

k k

k

xcos x ,

k

=

−=∑ temos que ( ) ( )

0

n mn m

mg x a x ,

=

=∑ onde os ( )nma são tais que existe

uma constante 0nk > tal que ( ) ! 0n mm na k n m , m .≤ ∀ ≥

Como, para todo

23

3

1 !3

m

mk

mm , m k ,>

≥ ≥ >

∏ temos que

( )2 3 2 3

2 2 21 63!

3

mmn m m m m m

m n n nm na m k n m m k n m k ,

m ⋅ ⋅ ≤ ⋅ ⋅ < ⋅ ⋅ = ⋅

donde

( ) 2 0n mmm

lim a m .→∞

⋅ = Em particular, existe 1nj ≥ tal que ( ) 21n mm na m , m j .< ∀ ≥

Page 125: Eureka 2009

Sociedade Brasileira de Matemática

EUREKA! N°30, 2009

61

Definimos ( )0

1n

nn mm j

t max a≤ <

= + e ( ) 21

1 122 nn jnn

n nn

w min , .t jsup g x ,x+

= ⋅ ⋅⋅ ∈

Temos então ( )2

1 1 02

nn m n mw a , m ,n .

m⋅ < ∀ ≥ ≥

⋅ Escolheremos as constantes nc

satisfazendo sempre n nc w< (sempre escolhemos nc depois de já ter escolhido o conjunto nB , e logo já tendo determinado a função ng ). Teremos então

( ) ( )1 1

mn n m

n m

f x x c g x a x ,∞ ∞

= =

= + ⋅ = ⋅∑ ∑ onde ( )1 1

1

1 nn

n

a c a∞

=

= +∑ satisfaz

( )1 1

1 1

11 1 22

nn n

n na w a

∞ ∞

= =

< + ⋅ < + =∑ ∑ e

( ) ( )2 2

1 1 1

1 1 22

n nm n m n m n m m

n n na c a w a , m .

m m

∞ ∞ ∞

= = =

≤ ⋅ < ⋅ < = ∀ ≥⋅∑ ∑ ∑ Em particular

1

mm

ma x

=

⋅∑

convergirá para todo x .∈ Escolheremos agora os conjuntos nB e as constantes 1 2nc ,n ,− ≥ recursivamente.

Para n par, tomamos o menor k tal que 1k nx B ,−∉ e definimos 1n n kB B x .−= ∪

Como π é irracional, ( )1 0n kg x .− ≠ Assim, como A é denso em , podemos

escolher ( )1 1 1n n nc w ,w− − −∈ − tal que ( ) ( )1 11 1

k m m k n n km n

x c g x c g x− −≤ < −

+ ⋅ + ⋅ ∈∑ A, e tal

que, se 2y a k ,= + π com 0a , k \ ,∈ ∈ então ( )1 1

m mm n

y c g y≤ ≤ −

+ ⋅ ∉∑ A (de fato

o conjunto dos 1nc − ∈ tais que a última condição falha é enumerável – usamos aqui o fato de π ser transcendente).

Seja agora 3n ≥ ímpar. Seja ( ) ( )2

21

n

n m mm

f x x c g x .−

−=

= + ⋅∑ Temos que

( )11 2 3 2nf´ x , x ,−< < ∀ ∈ e logo 2nf − é uma bijeção crescente de em .

Considere agora o menor r∈ tal que ( ) 2 1r n ny f b ,b B .− −∉ ∈ Temos, por

construção, que ( )12n rf y−

− não é da forma 2a k ,+ π com 1na B ,k .−∈ ∈ Assim,

( )( )11 2 0n n rg f y ,−− − ≠ e portanto podemos escolher ( )1 1 1n n nc w ,w− − −∈ − tal que, se

( ) ( )1

11

n

n m mm

f x x c g x ,−

−=

= + ⋅∑ ( )11n rf y ,−− ∈ e tal que, se 2y a k ,= + π com

0a ,k \ ,∈ ∈ então ( ) ( )11 1

n m mm n

f y y c g y−≤ ≤ −

= + ⋅ ∉∑ A. Tomamos então

Page 126: Eureka 2009

Sociedade Brasileira de Matemática

EUREKA! N°30, 2009

62

( ) 11 1n n n rB B f y .−− −= ∪ Ao final dessa construção é claro que ( )f x ∈A para todo

x∈ e ( )1f y− ∈ para todo y∈A. Obs.: Se não quisermos usar o fato de π ser transcendente podemos trocar as funções ( )sen x c− por ( )( )sen x c ,−πα com α transcendente. SEGUNDO DIA PROBLEMA 4 SOLUÇÃO DE RENATO REBOUÇAS DE MEDEIROS (S.J. DOS CAMPOS – SP) Lema: Se um segmento de reta divide o retângulo em duas regiões 1R e 2R da mesma área, então o segmento passa pelo centro do retângulo, que é (0,5, 0,5), no caso. Prova: De fato, se o segmento dividisse igualmente a área do retângulo e não passasse pelo seu centro, aconteceria:

A

B

M (0,5, 0,5) Segmento que passa pelo centro ( )AB

Segmento que não passa pelo centro ( )AB

Nessa situação cada metade limitada por AB teria área 12

, enquanto cada metade

limitada por A B também teria essa mesma área (por semelhança de triângulos, A B divide igualmente) e o triângulo AA´B teria área nula, absurdo. Assim, tais

segmentos passam pelo centro do retângulo. y

x 0

Ө M

1

1

Page 127: Eureka 2009

Sociedade Brasileira de Matemática

EUREKA! N°30, 2009

63

Girando um segmento (de comprimento variável) em torno do centro do retângulo (M), graças ao lema, obtêm-se todas as possibilidades de dividir a área em duas regiões de mesma área. Orientando esse segmento, é possível denominar por 1A a integral sobre a região à direita da seta ( )1R e por 2A a integral sobre a região à esquerda da seta, que será

2R . Além disso, ( )1 1A A= θ e ( )2 2A A ,= θ sendo θ o ângulo no sentido anti-horário entre o segmento e um eixo horizontal por M. Resumindo, ( ) ( )

1

1R

A f x, y dxdy= ∫θ e ( ) ( )2

1R

A f x, y dxdy.= ∫θ

Outra constatação é que ( ) ( )1 2 0A A=π e ( ) ( )2 1 0A A ,=π pois as regiões 1R e

2R de 0=θ são trocadas para .=θ π Ainda, tem-se que

( ) ( ) ( )1 2Q

A A f x, y dxdy.+ = ∫θ θ

Como *f : Q +→ é contínua, há indícios de que ( )1A θ e, conseqüentemente,

( )2A θ são funções contínuas em θ em todo o intervalo [ ]0,π em que as funções

1A e 2A merecem análise. Provaremos este fato no final da solução. Como as funções ( )1A θ e ( )2A θ têm soma constante e trocam de valor entre

0=θ e ,=θ π além de serem contínuas, então as duas assumem o mesmo valor para algum θ , com 0 .≤ <θ π De fato, basta ver que, se ( ) ( )1 20 0A A≠ , então a função ( ) ( ) ( )1 2G A A= −θ θ θ no mesmo domínio de 1A e 2A tem um sinal em 0=θ e o sinal contrário em =θ π ,

pois ( ) ( ) ( )1 20 0 0G A A= − e ( ) ( ) ( ) ( ) ( ) ( )1 2 1 20 0 0G A A A A G .= − = − − = − π π π Então G tem uma raiz entre 0 e π , por ser contínua e pelo Teorema do Valor Intermediário. Nessa raiz, digamos ´,θ têm-se as condições desejadas. Se

( ) ( )1 20 0A A ,= as condições já são satisfeitas para o segmento inicial.

Resta provar que ( )1A θ é contínuo em θ .

Para isso, basta ver que, por f ser contínua em [ ] [ ]0 1 0 1, , ,× e ainda positiva, assume um valor máximo M e um valor mínimo m. Sendo assim,

( ) ( ) ( )2

1 12

2A A M m

+ ∆ − ≤ − ⋅ ⋅

θ θ θ θ (pois e na figura são 2

2≤ ).

Page 128: Eureka 2009

Sociedade Brasileira de Matemática

EUREKA! N°30, 2009

64

∆Ө

Assim, 0,∀ >ε para 2 01

.M m

⋅= >

− +εδ Temos ( ) ( )0 1 1 0A A− < ⇒ − <θ θ δ θ θ ε

e assim 1A é contínua, permitindo aplicar o Teorema do Valor Intermediário para G como feito acima. PROBLEMA 5 SOLUÇÃO DE FABIO DIAS MOREIRA (RIO DE JANEIRO – RJ) Se os autovalores de A são (6, – 5, –5, 1, 1, 1, 1), os autovalores de 3B A= são (216, –125, – 125, 1, 1, 1, 1). Por outro lado, se ( )ijX x= e ( )ijY y= são matrizes

com 0, 0,ij ijx y≥ ≥ então a mesma propriedade vale para Z = XY : de fato, 7

10.ij ij kj

kz x y

=

= ≥∑ Logo 0 1 , 7ijb i j≥ ∀ ≤ ≤ e daí Tr 11 77... 0.B b b= + + ≥

Mas Tr 216 125 125 1 1 1 1 30,B = − − + + + + = − contradição. Portanto não existe nenhuma matriz A com a propriedade pedida. PROBLEMA 6 SOLUÇÃO OFICIAL DA BANCA Começamos com a expressão de senx como produto infinito:

2

2 21

1 .n

xsenx xn

=

= −

∏ π

Sabendo que essa igualdade vale para todo x∈ , como os dois lados definem

funções analíticas em (lembramos que ),2

ix ixe esenxi

−−= a igualdade vale para

todo .x∈ Fazendo x iy= −π obtemos

Page 129: Eureka 2009

Sociedade Brasileira de Matemática

EUREKA! N°30, 2009

65

( ) 2

2 21 1

1 12

y y

n n

senh y e e x ysenx x iyi i n n

− ∞ ∞

= =

− = = = − = +

∏ ∏π ππ

ππ

donde

( )2

21

1 ,n

ysenh y yn

=

= +

∏π π para todo y∈ , e, em particular, para todo y > 0.

Aplicando logaritmos, obtemos ( )2

21

log log log log 1 , 0.n

ysenh y y yn

=

= + + + ∀ >

∑π π

Derivando, temos:

(*)( ) 2 2

1

cos ( ) 1 2 , 0.n

h y y ysenh y y y n

=

= + ∀ >+∑ππ

π

Derivando novamente, obtemos, para todo y > 0,

( )( )( )( ) ( )( )

( )( ) ( )

2 222 2

2 2 22 2 2 2 21

( ) cos ( ) 1 2 4n

senh y h y yy y nsenh y senh y y n

=

− − = = − + − + +

∑π π ππ

π π

Portanto, ( ) ( )( )

2 2

2 22 2 22 21 1

1 12 2 , 0.n n

y yy n y senh yy n

∞ ∞

= =

− = − ∀ > + + ∑ ∑ π

π

O lado direito dessa igualdade é positivo para todo y > 0, de fato, isso equivale a ( ) , 0;senh y y y> ∀ >π π os dois lados dessa última desigualdade coincidem para

y = 0, e a derivada do lado esquerdo, que é ( ) cos2

y ye eh y− +

=

π π

π π π é maior

que ,π que é a derivada do lado direito, para todo y > 0.

Portanto temos, para todo y > 0, ( )

2

22 2 2 21 1

1 2 .n n

yy n y n

∞ ∞

= =

>+ +

∑ ∑

Essa desigualdade obviamente também vale para y = 0. Finalmente, tomando y = λ , concluímos que a desigualdade do enunciado vale para todo 0.≥λ

Page 130: Eureka 2009

Sociedade Brasileira de Matemática

EUREKA! N°30, 2009

66

XXX OLIMPÍADA BRASILEIRA DE MATEMÁTICA Resultado – Nível 1 (6o. e 7o. Anos)

NOME CIDADE – ESTADO PRÊMIO Guilherme Renato Martins Unzer São Paulo – SP Ouro Francisco Markan Nobre de Souza Filho Fortaleza – CE Ouro Rafael Rodrigues Rocha de Melo Fortaleza – CE Ouro Liara Guinsberg São Paulo – SP Ouro Pedro Augusto de Paula Barbosa Belo Horizonte – MG Ouro Matheus Silva Lima Bragança Paulista – SP Ouro Vinícius Canto Costa Salvador – BA Prata Michel Rozenberg Zelazny São Paulo – SP Prata Vinicius Luiz Ferreira Belo Horizonte – MG Prata Lucas Cawai Julião Pereira Caucaia – CE Prata Mário de Mello Figueiredo Neto Petrópolis – RJ Prata Henrique Vieira G. Vaz São Paulo – SP Prata Elias Brito Oliveira Brasília – DF Prata Igor Albuquerque Araujo Belo Horizonte – MG Prata Luis Fernando Veronese Trivelatto Cascavel – PR Prata Daniel Lima Santanelli Rio de Janeiro – RJ Prata Glauber Lima da Cunha Júnior Fortaleza – CE Bronze Tiago Sueda Limone Jundiaí – SP Bronze Leyberson Pereira Assunção Fortaleza – CE Bronze João Marcos Carnieleto Nicolodi Florianópolis – SC Bronze Fellipe Sebastiam da Silva P. Pereira Recife – PE Bronze Lucas Cardoso Zuccolo São Paulo – SP Bronze Maria Clara Cardoso São Paulo – SP Bronze Daniel Vincent Cacsire Garibay São Carlos – SP Bronze Rafael Tedeschi Eugênio Pontes Barone Araçatuba – SP Bronze Gabriel Nogueira Coelho de Togni de Souza Rio de Janeiro – RJ Bronze Thomas Akio Ikeda Valvassori Mogi das Cruzes – SP Bronze William Cechin Guarienti Porto Alegre – RS Bronze Gabriel Yoshimi Barrón Toyama Brasília – DF Bronze Daniel Behrens Cardoso Salvador – BA Bronze Ricardo Vidal Mota Peixoto Vassouras – RJ Menção Honrosa Jardel da Silva Pires Santos Dumont – MG Menção Honrosa Felipe Mendes de Holanda Lins Recife – PE Menção Honrosa Daniel Shinji Hoshi São Paulo – SP Menção Honrosa Júlio César de Barros Santo André – SP Menção Honrosa Gabriel Queiroz Moura Teresina – PI Menção Honrosa Lucas Carvalho Daher Anápolis – GO Menção Honrosa Daiana Luna Rio de Janeiro – RJ Menção Honrosa Filipe Santana do Vale Salvador – BA Menção Honrosa Rosane Thiemi Toma Gundim Campo Grande – MS Menção Honrosa Guilherme de Oliveira Rodrigues Fortaleza – CE Menção Honrosa Breno Soares da Costa Vieira Jaboatão dos Guararapes – PE Menção Honrosa Lara Viana de Paula Cabral Fortaleza – CE Menção Honrosa Nicolas Chiu Ogassavara São Paulo – SP Menção Honrosa Juliana Amoedo Amoedo Plácido Salvador – BA Menção Honrosa Bruno Cordeiro de Macedo São Paulo – SP Menção Honrosa Arthur Schott Lopes Curitiba – PR Menção Honrosa Bruno Costa Silva Rio de Janeiro – RJ Menção Honrosa Alessandro Augusto Pinto de Oliveira Pacanowski Rio de Janeiro – RJ Menção Honrosa Vitória Carolina Rondon Pereira Jacareí – SP Menção Honrosa Jorge Luiz Soares Pereira Rio de Janeiro – RJ Menção Honrosa Pedro Carvalho da Fonseca Guimarães Cachoeira Paulista – SP Menção Honrosa Guilherme Ryu Odaguiri Kobori São Paulo – SP Menção Honrosa Lorena Marroni Carvalho Amparo – SP Menção Honrosa Luiz Akyhito Miyazaki Serra Negra – SP Menção Honrosa Marcos Felipe Nunes Lino Ribeiro São Paulo – SP Menção Honrosa Gustavo Souto Henriques Campelo João Pessoa – PB Menção Honrosa João Vitor Fernandes Paiva Rio de Janeiro – RJ Menção Honrosa Fábio Kenji Arai São Paulo – SP Menção Honrosa Roberto Tadeu Abrantes de Araújo Rio de Janeiro – RJ Menção Honrosa Lucas Butschkau Vida Pinhais – PR Menção Honrosa João Ribeiro Pacheco Salvador – BA Menção Honrosa

Page 131: Eureka 2009

Sociedade Brasileira de Matemática

EUREKA! N°30, 2009

67

Nível 2 (8o. e 9o. Anos) Nome Cidade - Estado Prêmio João Lucas Camelo Sá Fortaleza – CE Ouro Gustavo Haddad F. e Sampaio Braga S. J. dos Campos – SP Ouro Gabriel Militão Vinhas Lopes Fortaleza – CE Ouro Rubens Cainan Sabóia Monteiro Fortaleza – CE Ouro Maria Clara Mendes Silva Pirajuba – MG Ouro Marla Rochana Braga Monteiro Fortaleza – CE Ouro Otávio Araújo de Aguiar Fortaleza – CE Prata Caíque Porto Lira Fortaleza – CE Prata Tuane Viana Pinheiro Rio de Janeiro – RJ Prata Carlos Henrique de Andrade Silva Fortaleza – CE Prata Otávio Augusto de Oliveira Mendes Pilar do Sul – SP Prata Rafael Kazuhiro Miyazaki São Paulo – SP Prata Vinicius Cipriano Klein Venda do Imigrante – ES Prata Daniel dos Santos Bossle Porto Alegre – RS Prata Mateus Braga de Carvalho Teresina – PI Prata Luiz Henrique Vieira Leão Rio de Janeiro – RJ Prata Kayo de França Gurgel Fortaleza – CE Prata Davi Coelho Amorim Fortaleza – CE Bronze Rodolfo Rodrigues da Costa Fortaleza – CE Bronze Rafael Henrique dos Santos Santa Cruz do Rio Pardo – SP Bronze Ivan Tadeu Ferreira Antunes Filho Lins – SP Bronze Lucas Cordeiro Herculano Fortaleza – CE Bronze Breno Leví Corrêa Campo Belo – MG Bronze Tábata Cláudia Amaral de Pontes São Paulo – SP Bronze Marina Pessoa Mota Fortaleza – CE Bronze Marcos Massayuki Kawakami São Paulo – SP Bronze Bruno Ferri de Moraes São Paulo – SP Bronze Tiago Leandro Estevam Dias Rio de Janeiro – RJ Bronze Gabriel Pacianotto Gouveia São Paulo – SP Bronze Filipe José Oliveira Sabóia Fortaleza – CE Bronze Daniel Prince Carneiro São Lourenço – MG Bronze Bruno Moraes Moreno Porto Nacional – TO Bronze Renan Fernandes Moreira Taubaté – SP Menção Honrosa Murilo Dória Guimarães São Paulo – SP Menção Honrosa Lucas Nishida Pedreira – SP Menção Honrosa Leonardo Ferreira Patrício Rio de Janeiro – RJ Menção Honrosa Victor Kioshi Higa São Paulo – SP Menção Honrosa Marina de Moura Faleão Recife – PE Menção Honrosa Lucas Almeida Pereira de Lima Recife – PE Menção Honrosa Arthur Ribeiro Notaro Recife – PE Menção Honrosa Pedro Mendonça de Lima Goiânia – GO Menção Honrosa Rafael Ferreira Antonioli S.B. do Campo – SP Menção Honrosa Lucas Okumura Ono São Paulo – SP Menção Honrosa Vinicius Affonso de Carvalho São Paulo – SP Menção Honrosa Julio Barros de Paula Taubaté – SP Menção Honrosa João Francisco Goes Braga Takayanagi São Paulo – SP Menção Honrosa Débora Barreto Ornellas Salvador – BA Menção Honrosa Luis Henrique Kobayashi Higa Campo Grande – MS Menção Honrosa Vítor Gabriel Barra Souza Juiz de Fora – MG Menção Honrosa Pedro Ivo Coêlho de Araújo Caucaia – CE Menção Honrosa Guilherme Cherman Perdigão de Oliveira Rio de Janeiro – RJ Menção Honrosa Cesar Nobuo Moniwa Ishiuchi Campinas – SP Menção Honrosa Letícia Dias Mattos Contagem – MG Menção Honrosa Raul Aragão Rocha Recife – PE Menção Honrosa Tiago de Ávila Palhares Brasília – DF Menção Honrosa Gabriel Leal Teixeira de Souza Rio de Janeiro – RJ Menção Honrosa

Page 132: Eureka 2009

Sociedade Brasileira de Matemática

EUREKA! N°30, 2009

68

Nível 3 (Ensino Médio) Nome Cidade - Estado Prêmio Henrique Pondé de Oliveira Pinto São Paulo – SP Ouro Régis Prado Barbosa Fortaleza – CE Ouro Marcelo Tadeu de Sá Oliveira Sales Salvador – BA Ouro Guilherme Philippe Figueiredo São Paulo – SP Ouro Marcelo Matheus Gauy S.J. do Rio Preto – SP Ouro Gabriel Luís Mello Dalalio S.J. dos Campos – SP Ouro Renan Henrique Finder São Paulo – SP Prata Alfredo Roque de Oliveira Freire Filho Salvador – BA Prata Rafael Tupynambá Dutra Belo Horizonte – MG Prata Marco Antonio Lopes Pedroso Santa Isabel – SP Prata Thiago da Silva Pinheiro São Paulo – SP Prata Eduardo Queiroz Peres Jundiaí – SP Prata Davi Lopes Alves de Medeiros Fortaleza – CE Prata Ricardo Turolla Bortolotti Rio de Janeiro – RJ Prata Alex Atsushi Takeda Londrina – PR Prata Deborah Barbosa Alves São Paulo – SP Prata Marlen Lincoln da Silva Fortaleza – CE Bronze Hugo Fonseca Araújo Rio de Janeiro – RJ Bronze Rafael Parpinel Cavina São Paulo – SP Bronze Thiago Ribeiro Ramos Varginha – MG Bronze Rafael Horimoto de Freitas São Paulo – SP Bronze Rafael Alves da Ponte Fortaleza – CE Bronze Robério Soares Nunes Ribeirão Preto – SP Bronze Gustavo Lisbôa Empinotti Florianópolis – SC Bronze Henrique Hiroshi Motoyama Watanabe São Paulo – SP Bronze Ivan Guilwon Mitoso Rocha Fortaleza – CE Bronze José Airton Coêlho Lima Filho Fortaleza – CE Bronze Hudson do Nascimento Lima Fortaleza – CE Bronze Hanon Guy Lima Rossi São Paulo – SP Bronze Ricardo Bioni Liberalquino Maceió – AL Bronze Luiz Filipe Martins Ramos Rio de Janeiro – RJ Bronze Illan Feiman Halpern São Paulo – SP Bronze Matheus Secco Torres da Silva Rio de Janeiro – RJ Bronze Matheus Araújo Marins Rio de Janeiro – RJ Menção Honrosa Paulo Cesar Neves da Costa Brasília – DF Menção Honrosa Leonardo Pereira Stedile São Paulo – SP Menção Honrosa Jonas Rocha Lima Amaro Fortaleza – CE Menção Honrosa Matheus Barros de Paula Taubaté – SP Menção Honrosa Rafael Sussumu Yamaguti Miada Valinhos – SP Menção Honrosa Rafael Alves da Silva Teresina – PI Menção Honrosa James Jun Hong São Paulo – SP Menção Honrosa Joas Elias dos Santos Rocha Muribeca – SE Menção Honrosa Gelly Whesley Silva Neves Fortaleza – CE Menção Honrosa João Mendes Vasconcelos Fortaleza – CE Menção Honrosa Marilia Valeska Costa Medeiros Fortaleza – CE Menção Honrosa Júlio Cézar Batista de Souza Salvador – BA Menção Honrosa José Cabadas Duran Neto Salvador – BA Menção Honrosa Thiago Saksanian Hallak São Paulo – SP Menção Honrosa Luiz Eduardo Schiller Rio de Janeiro – RJ Menção Honrosa Esdras Muniz Mota Fortaleza – CE Menção Honrosa Fábio Luís de Mello São Paulo – SP Menção Honrosa Ana Beatriz Prudêncio de Almeida Rebouças Fortaleza – CE Menção Honrosa Victorio Takahashi Chu São Paulo – SP Menção Honrosa Isabella Amorim Gonçalez Maceió – AL Menção Honrosa Grazielly Muniz da Cunha Fortaleza – CE Menção Honrosa José Leandro Pinheiro Deputado Irapuan Pinheiro – CE Menção Honrosa Antônio Deromir Neves Silva Júnior Fortaleza – CE Menção Honrosa

Page 133: Eureka 2009

Sociedade Brasileira de Matemática

EUREKA! N°30, 2009

69

Nível Universitário Nome Cidade - Estado Prêmio Fábio Dias Moreira Rio de Janeiro – RJ Ouro Rafael Daigo Hirama S.J. dos Campos – SP Ouro Guilherme Rodrigues Nogueira de Souza São Paulo – SP Ouro Eduardo Poço São Paulo – SP Ouro Ramón Moreira Nunes Fortaleza – CE Ouro Renato Rebouças de Medeiros S.J. dos Campos – SP Prata Thiago Costa Leite Santos São Paulo – SP Prata Raphael Constant da Costa Rio de Janeiro – RJ Prata André Linhares Rodrigues Campinas – SP Prata Edson Augusto Bezerra Lopes Fortaleza – CE Prata Marcelo de Araújo Barbosa S.J. dos Campos – SP Prata Levi Máximo Viana Rio de Janeiro – RJ Prata Thomás Yoiti Sasaki Hoshina Rio de Janeiro – RJ Prata Felipe Gonçalves Assis Campina Grande – PB Prata Fernando Nascimetno Coelho S.J. dos Campos – SP Prata Marcos Victor Pereira Vieira S.J. dos Campos – SP Prata Reinan Ribeiro Souza Santos Aracajú – SE Bronze Mateus Oliveira de Figueiredo S.J. dos Campos – SP Bronze Caio Ishizaka Costa S.J. dos Campos – SP Bronze Paulo Sérgio de Castro Moreira S.J. dos Campos – SP Bronze Pedro henrique Milet Pinheiro Pereira Rio de Janeiro – RJ Bronze Willy George do Amaral Petrenko Rio de Janeiro – RJ Bronze Jorge Henrique Craveiro de Andrade Rio de Janeiro – RJ Bronze Leandro Farias Maia Fortaleza – CE Bronze Alysson Espíndola de Sá Silveira S.J. dos Campos – SP Bronze Leonardo Ribeiro de Castro Carvalho São Paulo – SP Bronze Rafael Montezuma Pinheiro Cabral Fortaleza – CE Bronze Rafael Sampaio de Rezende Fortaleza – CE Bronze Luty Rodrigues Ribeiro S.J. dos Campos – SP Bronze Leandro Augusto Lichtenfelz Florianópolis – SC Bronze Sidney Cerqueira Bispo dos Santos Filho S.J. dos Campos – SP Bronze André Jorge Carvalho São Paulo – SP Menção Honrosa Jordan Freitas Piva Rio de Janeiro – RJ Menção Honrosa Juan Raphael Diaz Simões São Paulo – SP Menção Honrosa Eduardo Fischer Encantado – RS Menção Honrosa Adenilson Arcanjo de Moura Júnior Fortaleza – CE Menção Honrosa Kellem Correa Santos Rio de Janeiro – RJ Menção Honrosa Roberto Akiba de Oliveira São Paulo – SP Menção Honrosa José Armando Barbosa Filho S.J. dos Campos – SP Menção Honrosa Daniel Lopes Alves de Medeiros S.J. dos Campos – SP Menção Honrosa Vitor Humia Fontoura Rio de Janeiro – RJ Menção Honrosa Bruno da Silva Santos Belford Roxo – RJ Menção Honrosa Luca Mattos Möller S.J. dos Campos – SP Menção Honrosa Marcelo Salhab Brogliato Rio de Janeiro – RJ Menção Honrosa José Marcos Andrade Ferraro São Paulo – SP Menção Honrosa Evandro Makiyama São Paulo – SP Menção Honrosa Antonia Taline de Souza Mendonça Rio de Janeiro – RJ Menção Honrosa Diego Andrés de Barros Lima Barbosa Rio de Janeiro – RJ Menção Honrosa Gabriel Ponce São Carlos – SP Menção Honrosa Luiz Paulo Freire Moreira Fortaleza – CE Menção Honrosa Felipe Rodrigues Nogueira de Souza São Paulo – SP Menção Honrosa Igor Magalhães Oliveira Maceió – AL Menção Honrosa Rafael Ghussn Cano Campinas – SP Menção Honrosa

Page 134: Eureka 2009

Sociedade Brasileira de Matemática

EUREKA! N°30, 2009

70

AGENDA OLÍMPICA

XXXI OLIMPÍADA BRASILEIRA DE MATEMÁTICA

NÍVEIS 1, 2 e 3 Primeira Fase – Sábado, 06 de junho de 2009

Segunda Fase – Sábado, 12 de setembro de 2009 Terceira Fase – Sábado, 17 de outubro de 2009 (níveis 1, 2 e 3)

Domingo, 18 de outubro de 2009 (níveis 2 e 3 - segundo dia de prova).

NÍVEL UNIVERSITÁRIO Primeira Fase – Sábado, 12 de setembro de 2009

Segunda Fase – Sábado, 17 e Domingo, 18 de outubro de 2008

XV OLIMPÍADA DE MAIO 09 de maio de 2009

XX OLIMPÍADA DE MATEMÁTICA DO CONE SUL 14 a 20 de abril de 2009 Mar del Plata – Argentina

L OLIMPÍADA INTERNACIONAL DE MATEMÁTICA 10 a 22 de julho de 2009

Bremen – Alemanha

XVI OLIMPÍADA INTERNACIONAL DE MATEMÁTICA UNIVERSITÁRIA 25 a 30 de julho de 2009

Budapeste, Hungria

XXIV OLIMPÍADA IBEROAMERICANA DE MATEMÁTICA 17 a 27 de setembro de 2009

Querétaro, México ♦

XII OLIMPÍADA IBEROAMERICANA DE MATEMÁTICA UNIVERSITÁRIA

Page 135: Eureka 2009

Sociedade Brasileira de Matemática

EUREKA! N°30, 2009

71

COORDENADORES REGIONAIS

Alberto Hassen Raad (UFJF) Juiz de Fora – MG Américo López Gálvez (USP) Ribeirão Preto – SP Amarísio da Silva Araújo (UFV) Viçosa – MG Andreia Goldani FACOS Osório – RS Antonio Carlos Nogueira (UFU) Uberlândia – MG Benedito Tadeu Vasconcelos Freire (UFRN) Natal – RN Carlos Alexandre Ribeiro Martins (Univ. Tec. Fed. de Paraná) Pato Branco – PR Carmen Vieira Mathias (UNIFRA) Santa María – RS Claus Haetinger (UNIVATES) Lajeado – RS Cleonor Crescêncio das Neves (EDETEC) Manaus – AM Cláudio de Lima Vidal (UNESP) S.J. do Rio Preto – SP Denice Fontana Nisxota Menegais (UNIPAMPA) Bagé – RS Edson Roberto Abe (Colégio Objetivo de Campinas) Campinas – SP Eduardo Tengan (USP) São Carlos – SP Élio Mega (Grupo Educacional Etapa) São Paulo – SP Eudes Antonio da Costa (Univ. Federal do Tocantins) Arraias – TO Fábio Brochero Martínez (UFMG) Belo Horizonte – MG Florêncio Ferreira Guimarães Filho (UFES) Vitória – ES Francinildo Nobre Ferreira (UFSJ) São João del Rei – MG Genildo Alves Marinho (Centro Educacional Leonardo Da Vinci) Taguatingua – DF Ivanilde Fernandes Saad (UC. Dom Bosco) Campo Grande – MS Jacqueline Rojas Arancibia (UFPB)) João Pessoa – PB Janice T. Reichert (UNOCHAPECÓ) Chapecó – SC João Benício de Melo Neto (UFPI) Teresina – PI João Francisco Melo Libonati (Grupo Educacional Ideal) Belém – PA Jose de Arimatéia Fernandes (UFPB) Campina Grande – PB José Luiz Rosas Pinho (UFSC) Florianópolis – SC José Vieira Alves (UFPB) Campina Grande – PB José William Costa (Instituto Pueri Domus) Santo André – SP Krerley Oliveira (UFAL) Maceió – AL Licio Hernandes Bezerra (UFSC) Florianópolis – SC Luciano G. Monteiro de Castro (Sistema Elite de Ensino) Rio de Janeiro – RJ Luzinalva Miranda de Amorim (UFBA) Salvador – BA Mário Rocha Retamoso (UFRG) Rio Grande – RS Marcelo Rufino de Oliveira (Grupo Educacional Ideal) Belém – PA Marcelo Mendes (Colégio Farias Brito, Pré-vestibular) Fortaleza – CE Newman Simões (Cursinho CLQ Objetivo) Piracicaba – SP Nivaldo Costa Muniz (UFMA) São Luis – MA Osnel Broche Cristo (UFLA) Lavras – MG Osvaldo Germano do Rocio (U. Estadual de Maringá) Maringá – PR Raul Cintra de Negreiros Ribeiro (Colégio Anglo) Atibaia – SP Ronaldo Alves Garcia (UFGO) Goiânia – GO Rogério da Silva Ignácio (Col. Aplic. da UFPE) Recife – PE Reginaldo de Lima Pereira (Escola Técnica Federal de Roraima) Boa Vista – RR Reinaldo Gen Ichiro Arakaki (UNIFESP) SJ dos Campos – SP Ricardo Amorim (Centro Educacional Logos) Nova Iguaçu – RJ Sérgio Cláudio Ramos (IM-UFRGS) Porto Alegre – RS Seme Gebara Neto (UFMG) Belo Horizonte – MG Tadeu Ferreira Gomes (UEBA) Juazeiro – BA Tomás Menéndez Rodrigues (U. Federal de Rondônia) Porto Velho – RO Valdenberg Araújo da Silva (U. Federal de Sergipe) São Cristovão – SE Vânia Cristina Silva Rodrigues (U. Metodista de SP) S.B. do Campo – SP Wagner Pereira Lopes (CEFET – GO) Jataí – GO